ORTHOPEDIC MCQS BANK HAND AND WRIST 1B

145) A 32-year-old carpenter complains of progressively worsening wrist pain for the last 2 months. He denies any recent history of trauma to the wrist or hand. An MRI is obtained and a representative image is provided in Figure A. Which of the following surgical interventions is thought to be effective for this condition by inciting a local vascular healing response?

 

 

 

 

  1. Wrist fusion

  2. Ulnar shortening osteotomy

  3. Distal radius core decompression

  4. Proximal row carpectomy

  5. Scapholunate ligament reconstruction CorreCt answer: 3

This clinical scenario and imaging studies are consistent with Kienbock's disease, avascular necrosis of the lunate, in the pre-collapse stage. Core decompression of the distal radius is an accepted treatment for Kienbock's disease. The procedure creates a local vascular healing response facilitating vascular recovery prior to collapse and degeneration of the lunate. Other acceptable interventions include revascularization with a pedicled graft and joint leveling procedures such as a radial shortening osteotomy. The radial shortening osteotomy is ideal for patients with negative ulnar variance who experience greater loads through the radiolunate fossa.

 

Sherman et al performed a cadaveric study demonstrating minimal change in the distribution of force between the radiocarpal fossa and ulnocarpal fossa following core decompression of the distal radius.

 

Illarramendi et al reviewed 22 cases of Kienbock's treated with radial and ulnar metaphyseal core decompression. No surgical complications occurred, and 20 of 22 reported satisfactory clinical outcomes while one patient

developed intercarpal arthritis.

 

Incorrect Answers:

  1. Proximal row carpectomy and wrist fusion would be options for the collapsed and degenerative lunate.

  2. Ulnar shortening osteotomy and scapholunate ligament reconstruction are incorrect as they do not address the pathology of Kienbock's.

  1. Proximal row carpectomy and wrist fusion would be options for the collapsed and degenerative lunate.

  2. Ulnar shortening osteotomy and scapholunate ligament reconstruction are incorrect as they do not address the pathology of Kienbock's.

 

 

 

  1. A 30-year-old female undergoes arthroscopy for a chronically painful right wrist that failed to improve with 4 months of immobilization and NSAIDS. Her clinical examination revealed point tenderness dorsally over the lunate but no tenderness elsewhere in the wrist. A picture from the procedure is shown in Figure A where 'R' identifies the distal radius, 'L' the lunate, and '*' represents a chondral flap. The articular surface of the lunate is stable to probing. A radiograph and MRI image of the patients wrist are shown in Figures B and C respectively. What is the most appropriate next step in treatment?

     

     

     

     

     

     

     

     

    1. Continue Immobilization and NSAIDS

    2. Radial shortening osteotomy

    3. Proximal row carpectomy

    4. Scaphotrapeziotrapezoid fusion

    5. Wrist fusion CORRECT ANSWER: 2

    The patients clinical presentation and radiographs are consistent with Stage 2 Kienbock's disease in the setting of negative ulnar variance. Radial shortening osteotomy is the most appropriate treatment option listed for Stage 2 disease which is defined as lunate sclerosis without significant collapse. Shortening osteotomy can alter DRUJ contact pressures leading to remodeling, especially in the presence of a Tolat Type II DRUJ, such as that shown in the radiographs. However, this remodeling has been shown to occur without the development of arthritis, and therefore is not a contraindication to this procedure.

     

    This patients radiographs shows some slight sclerosis of the lunate and negative ulnar variance, and the MRI shows diffuse edema and early osteonecrosis of the lunate. The arthroscopic image shows a cartilage flap with a stable base left on the lunate. Based on these images, the patient has Stage 2 disease and should be treated with a joint leveling procedure; or radial shortening osteotomy in this case.

     

    Sltusky et al provide a review article which focuses on the methodology behind

    a normal arthroscopic wrist examination and discusses some of the more standard arthroscopic procedures along with the expected outcomes.

     

    Bain et al review the arthroscopic staging of Kienbock's disease, and state that this techinique is a valuable assessment tool which allows for not only classification of Kienbock's disease, but also may guide treatment.

     

    Schuind et al. provide a review of the pathogenesis of Kienbock's. They conclude that the natural history of the condition is not well known, and the symptoms do not correlate well with the changes in shape of the lunate and the degree of carpal collapse. They also state that there is no strong evidence to support any particular form of treatment.

     

    Illustration A shows a table which outlines the Stages of Kienbock's Disease.

     

    Illustration B shows a table which outlines the general treatment options for each stage of Kienbock's Disease.

     

    Incorrect Answers:

    Answer 1: Immobilization and NSAIDS is indicated in Stage I disease or as a first line of treatment for Stage 2, which this patient has failed.

    Answer 3: Proximal row carpectomy is indicated in Stage 3B. Answer 4: STT Fusion is indicated in Stage 3B.

    Answer 5: Wrist fusion is indicated in Stage 4.

     

     

     

     

     

     

     

     

     

  2. Cubital tunnel syndrome is caused by compression of the ulnar nerve between what two structures as it passes posterior to the medial epicondyle?

    1. Osborne's ligament and the MCL

    2. MCL and Arcade of Struthers

    3. Osborne's ligament and the intermuscular septum

    4. MCL and medial head of the triceps

    5. Ulnar and humeral heads of the flexor carpi ulnaris muscle

    CORRECT ANSWER: 1

    The ulnar nerve passes posterior to the medial epicondyle and medial to the olecranon, then enters the cubital tunnel. The roof of the cubital tunnel is primarily made up of Osborne's ligament, and the floor consists of the medial collateral ligament.

    These soft tissue structures can cause narrowing of the tunnel, especially with elbow flexion, leading to ulnar nerve compression and cubital tunnel syndrome. This is shown in Illustration A. The Arcade of Struthers is a band of deep fascia that attaches to the intermuscular septum and covers the ulnar nerve 8 cm proximal to the medial epicondyle. The intramuscular septum is continuous from the medial epicondyle to the coracobrachialis muscle. The ulnar nerve travels through the two heads of the FCU distal to the cubital tunnel. These anatomic landmarks are shown in Illustration B.

     

    Morrey evaluated 26 patients with post-traumatic contracture of the elbow who were treated with either operative release alone, or operative release and distraction arthroplasty. Twenty-four (96%) of the patients had improved elbow function and two had persistent ulnar neuritis treated with nerve transposition.

     

    Cheung et al discuss the various surgical approaches to the elbow and the indications for each.

     

    Video V is an educational lecture that discusses common nerve entrapment diagnosis and managment.

     

     

     

     

     

     

     

     

     

  3. A 42-year-old male diesel mechanic presents to your clinic 3 days after he was power washing automotive parts with a high-pressure solvent and accidently hit the tip of his finger with the spray gun. A clinical photo is shown in Figure A. What is the most appropriate first line of treatment?

     

     

     

     

    1. Irrigation and debridement at the bedside using a digital block

    2. Immediate debridement in the operating room

    3. A dose of IV antibiotics in the ER, followed by a 10 day oral course

    4. Immediate finger tip amputation

    5. Arrange for follow-up with a hand specialist

    CORRECT ANSWER: 2

    This patient suffered a high-pressure injection injury to his finger. These are uncommon soft tissue trauma injuries of the hand which are frequently underestimated. The prognostic factors are the type, amount and temperature of the material and the pressure of injection. Most of these hand injuries need to undergo immediate formal operative debridement and foreign body removal. This ideally should be done under a formal operative setting to decrease infection risk, and to allow for microvascular repair if needed.

    Delayed treatment can lead to further soft tissue damage and ultimately amputation.

     

    The article by Bekler et al looked at the results of 14 surgically treated high-pressure injection injuries of the hand with a minimum of two years follow-up. Ten of the injuries required formal operative debridement and foreign body removal. Six required reconstructive microsurgical procedures and one underwent digital tip amputation. They concluded that high-pressure injection injury to the hand is a significant problem, which can easily lead to serious sequelae and, even, amputation.

     

     

  4. Which of the following patients with Dupuytren's contracture would benefit the most from dermatofasciectomy and full-thickness skin grafting opposed to traditional fasciectomy?

    1. 70-year-old sedentary male with small finger involvement isolated to the MCP joint

    2. 50-year-old male systems analyst with ring and small finger involvement limited to the MCP joints

    3. 65-year-old female golfer with ring and small finger involvement including MCP and PIP joints

    4. 40-year-old female stenographer with middle, ring, and small finger involvement including MCP and PIP joints with 50 and 55 degree contractures of ring and small finger MCP joints, respectively

    5. None of the above as no difference in outcome has been demonstrated between the two procedures

    CorreCt answer: 5

     

    Dermatofasciectomy and full-thickness grafting has not demonstrated superior finger range of motion, recurrence rate, or patient satisfaction in comparison with traditional fasciectomy.

     

    The main reference from Ullah et al conducted a prospective randomized study of 84 Dupuytren's cases treated with fasciectomy alone or dermatofasciectomy with full-thickness skin grafting. The question was whether the overlying skin needed to be excised. No difference in clinical outcome or recurrence rate was discovered.

     

    Roy et al reviewed 79 cases of advanced Dupuytren's treated with radical fasciectomy (but preservation of the overlying skin) and then adding full-thickness skin grafting to the open areas once the fingers were extended. They found their results of fasciectomy to be similar to those published for dermofasciectomy.

     

     

     

  5. When surgically treating a trigger finger in a child, what structure may need to be released in addition to the A-1 pulley?

    1. One or both limbs of the sublimis tendon

    2. A-4 pulley

    3. Lumbrical origin

    4. Dorsal interosseous insertion

    5. Anomalous insertion of the MCP joint collateral ligament

    CorreCt answer: 1

     

    Unlike adults, release of the A-1 pulley in a pediatric trigger finger alone may not resolve triggering symptoms. Trigger finger in the child may be associated with a more proximal decussation of the FDS tendon, nodules in either the FDS or FDP tendon, a thickened A-2 pulley, or a tight A-3 pulley. Cardon et al looked at 16 pediatric pts with 18 trigger fingers and found that 6 fingers continued to trigger after A-1 pulley release. The sublimis decussation and A-3 pulley were found to be the most common cause of this persistent triggering. Bae et al looked at 23 pediatric trigger fingers and found that triggering was noted to occur at the level of the FDS tendon decussation in half the cases.

    The conclusion was made that all pediatric trigger fingers should be treated with A-1 pulley release and resection of a single FDS tendon slip. Illustration A shows normal decussation of the FDS tendon near the level of the A2 pulley.

    The FDS decussation may be found to be more proximal in pediatric trigger fingers, necessitating release.

     

     

     

     

     

     

  6. A 72-year-old female complains of progressive weakness with grasp and key pinch in her left hand. Physical exam of the hand is significant for decreased sensation on the volar aspect of the fourth and fifth digits. Dorsal sensation throughout the hand is normal. A clinical photo displaying bilateral key pinch is shown in Figure A. What is the most likely cause of compression?

     

     

     

    1. Accessory head of the FPL

    2. Flexor carpi ulnaris

    3. Osborne's ligament

    4. Ganglion within Guyon's canal

    5. Anconeus epitrochlearis

    CORRECT ANSWER:

    4

    Compression of the ulnar nerve within Guyon's canal, termed ulnar tunnel syndrome, is most commonly caused by a ganglion cyst. A lack of dorsal ulnar sensory deficit helps differentiate entrapment here from at the elbow because the dorsal ulnar cutaneous nerve branches proximal to Guyon's canal. The clinical photo demonstrates Froment's sign where the FPL is used to substitute for the weakened adductor pollicis resulting in flexion of the thumb at the interphalangeal joint, and MCP joint hyperextension. The AIN can be compressed by the accessory head of the FPL (Gantzer's muscle) which results in loss of FPL, index FDP and PQ motor function and no sensory deficits. Ulnar nerve compression at Osborne's ligament, the two heads of the FCU, or by the anconeus epitrochlearis will classically result in volar and dorsal ulnar sensory loss of the affected hand.

     

     

     

  7. Which of the following congenital hand deformities displayed in figures A-E is more prevalent in patients of African-American ancestry?

     

     

     

     

     

     

     

     

     

     

     

     

     

    1. Figure A

    2. Figure B

    3. Figure C

    4. Figure D

    5. Figure E CORRECT ANSWER: 2

    Image B is consistent for postaxial polydactyly, which is more prevalent in patients of African-American ancestry.

     

    The cohort study by Woolf found the incidence of postaxial polydactyly in African americans is 12.42 per 1,000 (1.2%) compared to the Caucasian incidence of 0.91 per 1,000 (0.09%). If postaxial polydactyly is found in a patient of Caucasian ancestry then further workup for underlying syndromes (chondroectodermal dysplasia or Ellis-van Creveld syndrome) is needed.

     

    The article by Orioli is a case-control study that hypothesizes that a sex-linked recessive modifier gene occurs more frequently in African americans and this gene then promotes the autosomal dominant polydactyly gene.

     

    Incorrect Answers: Constriction band syndrome or amniotic band syndrome is a type of pseudosyndactyly (Figure A) and is not the result of failure of differentiation during embryogenesis, but a result of injury by bands after the fingers are formed. Preaxial polydactyly (Figure C) is more common in caucasians and is usually sporadic except for triphalangism which is associated with Holt-Oram and Fanconi's Anemia. Syndactyly (Figure D) is defined as an abnormal interconnection between adjacent digits and syndactyly variations are associated with Apert syndrome and Poland syndrome. Macrodactyly (Figure E) represents overgrowth of all structures of the involved digit and is associated with neurofibromatosis and Klippel-Trenaunay-Weber syndrome.

     

     

  8. A 24-year-old male cuts his left middle finger with a knife while chopping vegetables. Physical exam reveals a zone 2 flexor tendon laceration. He undergoes a 2-strand core suture repair with epitendinous suture. This particular repair is strong enough for each of the following rehabilitation protocols EXCEPT:

    1. Kleinert protocol

    2. Duran protocol

    3. Synergistic motion protocol

    4. Low force and low tendon excursion passive range of motion

    5. Early digit active range of motion protocol

      CORRECT ANSWER: 5

      Early active range of motion protocols are thought to decrease adhesions but risk rerupture or gap formation.

       

      Strickland et al notes that the generation of muscle forces to either assist digit flexion or perform “place and hold” exercises require at least a 4-strand core suture with epitendinous repair. This patient only had a 2-strand repair.

       

      The Kleinert and Duran protocols are both forms of low force and low tendon excursion programs, that include passive digit flexion range of motion. Kleinert includes a dorsal block splint with the wrist in 45° of flexion and elastic bands secured to the patient’s nails and a more proximal attachment point. Once the interphalangeal joints are actively fully extended, recoil of the elastic bands flexes them down passively. The Duran protocol utilizes the other hand to passively flex the affected DIP and PIP joints and a higher amount of patient compliance is needed. Synergistic motion regimens allow passive digit flexion combined with active wrist extension, followed by active digit extension coupled with active wrist flexion to produce low forces and high tendon excursions at the involved digit.

       

       

       

  9. Creation of a Stener lesion, as found in Gamekeeper's thumb, requires combined tears of the proper and accessory ulnar collateral ligaments in order for the ligament to be displaced by the adductor aponeurosis. Which of the following most accurately describes the role

    these ulnar collateral ligaments (PCL/ACL) play in thumb MCP joint stability?

    1. PCL is primary restraint to radial deviation with MCPJ in flexion, ACL provides restraint to radial deviation with MCPJ in extension

    2. PCL is primary restraint to radial deviation with MCPJ in extension, ACL provides restraint to radial deviation with MCPJ in extension

    3. ACL is primary restraint to ulnar deviation with MCPJ in flexion, PCL provides restraint to ulnar deviation with MCPJ in extension

    4. ACL is primary restraint to radial deviation with MCPJ in flexion, PCL provides restraint to radial deviation with MCPJ in extension

    5. PCL is primary restraint to ulnar deviation with MCPJ in flexion, ACL provides restraint to radial deviation with MCPJ in extension

    CorreCt answer: 1

     

    The proper ulnar collateral ligament(PCL) runs from the metacarpal head to the volar aspect of proximal phalanx and resists radial stress with the thumb MCPJ in flexion. The accessory ulnar collateral ligament(ACL) lies palmar to the proper ligament, and inserts onto the volar plate. The volar plate and ACL function as the principle restraints to radial stress with the thumb MCPJ in extension.

     

    The function of the ulnar collateral ligaments is shown in Illustration A.(Please note the distal phalanx of the thumb has been removed in Illustration A.) A Stener lesion is described by displacement of the distal end of the completely ruptured UCL such that it comes to lie superficial and proximal to the adductor aponeurosis. This is shown in Illustration B.

     

    Thrikannad and Wolff report a case of distal pull-off of the ulnar collateral ligament (UCL) of the thumb MCPJ with two fracture fragments. They identify the need to look for a second fragment of bone in these injuries, where an apparently undisplaced fracture is noted at the base of the proximal phalanx. They suggest that this second fragment probably indicates the location of the distal end of the UCL and may identify a Stener lesion. A radiographic example from their paper is shown in Illustration C.

     

    Newland, in his review article on Gamekeeper's Thumb, states that criteria for judging what constitutes a complete tear vary from 15 deg to 45 deg difference with respect to the opposite side. He goes on to state, however, that many authors choose an absolute value of >35 degrees of joint laxity compared to the contralateral side when judging a tear to be complete or incomplete. When an complete tear is identified, surgical repair is recommended.

     

     

     

     

     

     

     

     

     

  10. Which of the following hand injuries is most appropriately treated with a volar advancement (Moberg) flap closure?

     

     

     

     

     

     

     

     

     

     

     

     

     

     

    1. Figure A

    2. Figure B

    3. Figure C

    4. Figure D

    5. Figure E CORRECT ANSWER: 4

    Figure D shows a volar thumb defect which can be best covered with a Moberg advancement volar flap (if < 2 cm). FDMA (1st dorsal metacarpal artery) and

    neurovascular island flaps are typically used to cover larger soft tissue defects of volar aspect of the thumb. FDMA (1st dorsal metacarpal artery) flaps can also be used for dorsal thumb wounds as shown in Figure B. The cross-finger flap is a useful heterodigital flap for digital wounds with primarily volar tissue loss (Figure A). Additionally, several articles have advocated secondary intention healing even if bone is exposed as discussed in the 2009 OITE question #48. The thenar flap is useful for volar defects of the index and middle fingers (Figure C). Figure E represents a ring avulsion injury and it is treated with vessel repair if there is inadequate circulation and the bone, tendon, and nerve components are intact. Amputation of the digit is chosen if there is inadequate circulation concomitant with bone, tendon, or nerve injury.

     

    The referenced articles by Martin and Hynes are review articles discussing the treatment options available for digit injuries. Illustration A shows the planned incisions for a moberg advancement flap on a volar thumb defect and Illustration B shows the completed Moberg.

     

     

     

     

     

     

     

     

  11. A 4-year-old child has flattened facial features, wide set eyes, and the hand deformity pictured in Figure A. Which of the following is the most likely diagnosis?

     

     

     

     

    1. Apert's syndrome

    2. Multiple epiphyseal dysplasia

    3. Cleidocranial dysplasia

    4. Noonan syndrome

    5. Achondroplasia CORRECT ANSWER: 1

    Apert's syndrome is a congenital disorder causing deformity of the skull, face, hands, and feet. It affects 1/80,000 children. An autosomal dominant mode of inheritance exists, but the majority of new cases are sporadic. Early fusion of the cranial and facial suture lines (craniofacial synostosis) results in a variety of skull and facial deformities. The primary deformity of the hands and feet is severe syndactyly, often with fusion of the digits. The index, middle, and ring fingers are affected most often. Cognitive function may be normal or moderately disabled.

     

    Rebelo et al reviews the hand deformities of 170 patients with Apert's syndrome and their clinical outcomes following surgical treatment.

     

    Al-Qattan et al review the complex syndactyly of the hand associated with Apert's syndrome and suggests a new classification scheme.

     

    Illustrations A-C are further examples of the face, hand, and foot deformities associated with Apert's syndrome

     

     

     

     

     

     

     

     

     

     

     

  12. A 38-year-old woman complains of a painful finger mass of 4 months duration. A photograph of the mass is provided in Figure A. The decision is made to proceed with surgical excision. Which of the following is an advantage of surgical excision with joint debridement as opposed to aspiration?

     

     

     

     

    1. Reduced rate of infection of the DIP joint

    2. Less post-procedure pain

    3. Improved DIP range of motion

    4. Decreased risk of mass recurrence

    5. Reduced risk of metastasis from seeding the mass into the joint

    CorreCt answer: 4

     

    Figure A demonstrates a mucous cyst. This benign mass originates from the DIP joint, and is secondary to arthritis. It may be treated with aspiration or surgical excision. However, recurrence occurs frequently with aspiration.

    Debridement of any osteophytes from the DIP joint is crucial to preventing recurrence with surgical excision. Rizzo et al retrospectively evaluated the results of 154 mucous cysts treated with either aspiration or surgery.

    Aspiration resulted in a 40% recurrence rate. There were zero recurrences with surgical excision and joint debridement.

     

     

     

  13. Figure A depicts a child with a congenital abnormality. Which of the following is true regarding this condition?

     

     

     

     

    1. Circumferential trunk involvement is more common than distal extremities involvement

    2. Risk factors include late gestation (>44 weeks) and high birth weight (>3500g)

    3. Incomplete circumferential bands not directly interfering with lymphatic circulation should be resected

    4. There is a strong correlation with anterolateral tibial bowing

    5. Complete circumferential bands that interfere with lymphatic drainage can be treated with band excision and z-plasty.

    CorreCt answer: 5

     

    The image and vignette describe a patient with constriction band syndrome (CBS). In the case of lymphatic obstruction or vascular compromise, the treatment of CBS is band excision. There are many terms used to describe this phenomenon. However, the etiology is the entanglement of fetal parts in the amniotic membrane.

     

    Foulkes et al reviewed 71 cases of congenital constriction band syndrome (CCBS). They found the average patient had three involved limbs, with a predilection for distal, central digits of the upper extremity. There was a strong correlation with abnormal gestation and clubfoot. Treatment included distraction osteogenesis and free osteocutaneous transfer.

     

    Goldfarb et al reviewed amniotic constriction band syndrome (ABS), highlighting its association with annular constriction of multiple extremities. They classified ABS into classic (disruptions and deformations) and non-classic (malformations). ABS is due to disruptions (amputations, acrosyndactyly), deformations (oligohydraminos, scoliosis, talipes equinovarus) and malformations (body-wall defects, cleft lip/palate). As there is moderate overlap between the classic and nonclassic, additional research into the underlying cause is being investigated.

     

    Green described a one-stage release of circumferential constriction bands in three patients. The advantages of this technique are the decreased need for anesthesia and subsequent procedures as well as facilitating postoperative care.

     

    Kawakura et al reviewed the intrinsic and extrinsic theories of (CBS). The most common manifestations are distal extremity involvement, intrauterine amputations and acrosyndactyly. Excision of bands and mobilization of subcutaneous adipose tissue as described by Upton is seen in Illustration A.

     

    Incorrect Answers:

     

    Answer 1: Distal extremities are more affected than the trunk

    Answer 2: Risk factors include low birth weight (<2500g), prematurity (<37wks), maternal drug exposure, trauma during pregnancy and attempted fetal termination during the first trimester.

    Answer 3: Shallow bands that do not interfere with circulation or lymphatic

    drainage do not need to be released.

    Answer 4: There is a strong correlation with clubfoot, not anterolateral bowing.

     

     

     

     

     

     

  14. A 25-year-old female is involved in a motorcycle collision and presents with the injuries seen in Figures A through D. What is the best option for definitive management of the injuries seen in Figure D?

     

     

     

     

     

     

     

     

     

     

     

    1. Open reduction and internal fixation

    2. Closed reduction and casting

    3. External fixation

    4. Immediate therapy

    5. Removable splint CORRECT ANSWER: 1

    The patient presents with mutliple injuries including a subtalar dislocation (Figure A), femoral shaft fracture (Figure B), tibia shaft fracture (Figure C) and multiple metacarpal shaft fractures (Figure D). Multiple metacarpal shaft fractures are best managed with open reduction and internal fixation as nonoperative management is associated with loss of motion, asynchronous grasp and decreased grip strength.

     

    Souer and Mudgal retrospectively reviewed their experience treating patients with multiple metacarpal fractures utilizing hand-specific implants. They argue that rigid internal fixation of multiple metacarpal fractures allows for early mobilisation and tendon excursion, and found excellent results in 18 of 19 patients with a 230 degree total arc of motion.

     

    Kawamura and Chung review fixation options for treating unstable oblique phalangeal and metacarpal fractures. They found low complication rates regarding tendon adhesion and stiffness with published studies examing dorsal plating of oblique metacarpal fractures as the extensor tendons are less adherent to bone at the level of the metacarpal.

     

    Incorrect Answers:

    Answer 2. Closed reduction and casting would lead to stiffness due to immobilization

    Answer 3. External fixation would bind the extensor mechanism and would not allow for early motion

    Answer 4. Immediate therapy, although beneficial, would be difficult to accomplish without rigid fixation

    Answer 5. Removeable splinting would not facilitate early motion and and would likely lead to loss of metacarpal length and deformity as the stabilizing effect of the adjacent metacarpals is lost with multiple fractures

     

     

     

  15. A 22-year-old gymnast with known ligamentous laxity has been treated in the hand therapy clinic for 6 months for left wrist pain and discomfort. Radiographs of her left wrist are seen in Figures A and B. Which of the following physical exam findings would be most diagnostic for midcarpal instability?

     

     

     

    1. Radial wrist pain with sudden ulnar deviation

    2. Tenderness to palpation distal to the ulnar styloid

    3. Pain and a clunk on ulnar to radial deviation of the wrist while pressure is held on the scaphoid

    4. Pain in the lunate with volar directed pressure on the dorsum of hand

    5. Pain and a clunk with axial and palmarly directed forces as the wrist is moved from neutral to ulnar deviation

    CorreCt answer: 5

     

    The clinical situation is consistent with midcarpal instability. The most common finding on physical examination is a clunk as the wrist is moved from a neutral position and forearm pronation to ulnar deviation with an axial and palmarly directed load. Carpal instability is complex condition marked by abnormal kinematics in the carpus. Carpal instability dissociative (CID) is marked by intrinsic ligamentous disruption. Carpal instability non-dissociative (CIND) is marked by extrinsic ligamentous disruption between carpal rows or between the proximal row and distal radius. Included in CIND is midcarpal instability (MCI) and radiocarpal instability. Radiographs typically show a mild VISI deformity or no abnormalities as in Figures A and B. Videofluorscopy is diagnostic as the proximal row assumes a volar, flexed position, then snaps into extension the wrist is moved into ulnar deviation.

     

    Lichman et al provided an overview and historical perspective of carpal instability. Carpal instability is divided into dissociative and non-dissociative. They concluded that there are several causes and patterns of carpal instability leading to carpal subluxation. An in-depth understanding is required for proper treatment.

    Apergis et al described 14 cases of midcarpal instability treated with ligamentous reefing of the midcarpal joint and or the radiolunate joint. They reported excellent results in eight cases, good in five cases, and fair in one case.

     

    Incorrect answers:

    Answer 1: A positive Finkelstein's maneuver is radial-sided pain with a clinched thumb and ulnar deviation of the wrist

    Answer 2: A positive Fovea sign is found with pain on palpation distal to the ulnar styloid

    Answer 3: Watson's scaphoid shift is pain and a clunk on ulnar to radial deviation of the wrist while pressure is held on the scaphoid tubercle.

    Answer 4: Describes pain in the lunate as in Keinbock's.

     

     

     

     

  16. A 2-year-old child has a flexion deformity of the interphalangeal joint of his thumb as seen in Figure A. Surgical correction of this deformity places what structure most at risk as it crosses the surgical field?

     

     

     

     

    1. Princeps pollicis artery

    2. Ulnar digital nerve

    3. Oblique pulley

    4. Ulnar digital artery

    5. Radial digital nerve CORRECT ANSWER: 5

    The patient in the scenario has a trigger thumb. Surgical correction of this condition requires the release of the A1 pulley. The A1 pulley is seen at the red arrow in Illustration A. During the dissection, the radial digital nerve crosses

    the operative field and is at risk. It must be identified and protected.

     

    Bae described the etiology, natural history and surgical indications in treatment of pediatric trigger thumbs. Currently, the literature does not advocate for or against surgical management. He concluded that further longterm research will help guide evidence-based treatment.

     

    Baek at al described the natural history of pediatric trigger thumbs. They concluded that trigger thumbs in children will resolve without treatment in

    >60% of patients. In patients who do not have full resolution, the flexion deformity can be expected to gradually improve with time.

     

     

     

     

     

     

  17. A 39-year-old male sustained an index finger injury 6 months ago and has failed eight weeks of splinting. A radiograph taken at the time of injury is shown in Figure A, and a current radiograph is shown in Figure B. Which of the following is true regarding open reduction and screw fixation of this injury?

     

     

     

    1. High risk of symptomatic implant

    2. Immobilization of the distal interphalangeal joint is required for 2 weeks post-operatively

    3. High rates of post-operative infection are common

    4. Open reduction via an approach through the nail bed leads to significant post-operative nail deformity

    5. Range of motion of the DIP joint in the affected finger is usually less than 10 degrees post-operatively

    CorreCt answer: 1

     

    Open reduction and internal fixation of distal phalanx fracture non-unions frequently requires the post-operative removal of the fixation implant after complete fracture healing.

     

    Chim et al followed 14 patients with non-union of fractures of the shaft of the distal phalanx who were treated with open reduction and screw fixation. The implants required removal in 13/14 patients, and the mean post-operative range of motion of the DIP joints was 56 degrees. No immobilization was required postoperatively, and bone grafting was only necessary in two patients with severely comminuted fractures. Finally, the authors recommended approaching the fracture through the nailbed for the best exposure, and found no postoperative nail growth complications. Postoperative infections were not common in their series.

    Mejis et al describe two patients with non-unions of the thumb distal phalanx treated with a single compression screw using a minimally invasive approach. Both patients healed their fractures using this technique.

     

     

     

  18. A 7-year-old boy sustains a ring finger injury after falling from his bike. The fingernail has been torn transversely beneath the eponychium and the surgeon has removed the nail as shown in Figure

    A. Radiographs are shown in Figure B. What is the next best step in management?

     

     

     

     

     

     

     

    1. Irrigation and debridement with alumafoam placement and immobilization

    2. Irrigation and debridement followed by percutaneous pinning and immobilization

    3. Irrigation and debridement followed by reduction, nail bed repair and immobilization

    4. Betadine soaks at home three times daily with intermittent alumafoam

      splint placement and immobilization

    5. Alumafoam splint placement and immobilization

    CORRECT ANSWER: 3

    The clinical presentation is consistent with a physeal separation and a nail bed injury. This is also called a Seymour fracture which is a juxta-epiphyseal fracture of the distal phalanx. Treatment of a nail bed avulsion and physeal separation is irrigation and debridement, physeal reduction, nail bed repair and immobilization. The primary goals are to achieve a stable, viable nail and good cosmetic results.

     

    Inglefield at al retrospectively reviewed 19 children with 22 nail bed injuries. Early operative repair led to good to excellent results in 91% of patients. They concluded that repair of the nail bed at the time of injury is superior to secondary correction.

     

    Fassler reviewed fingertip injuries, providing recommendations for treatment based on degree of soft tissue loss, bone exposure, feasibility for flap coverage and the presence or absence of mitigating systemic conditions. He also concluded that the outcome of nail bed injuries is dependent on the severity of injury to the germinal matrix.

     

    Illustration A shows the makeup of the terminal phalanx. Illustration B and C show a Seymour Fracture before and after irrigation and debridement and reduction.

     

     

     

     

     

     

     

     

     

     

     

     

  19. A 46-year-old homeless IV drug abuser presents with the hand infection shown in Figure A, which developed after sustaining a superficial laceration. Cultures are taken during operative irrigation

    and debridement, and he is started on antibiotic therapy. Based on the patients history, what is the most common pathogen in this setting?

     

     

     

     

    1. Herpes simplex virus

    2. Candida albicans

    3. Escherichia coli

    4. Eikenella corrodens

    5. Methicillin-resistant staphylococcus aureus

    CORRECT ANSWER: 5

    Figure A shows an abscess over the metacarpophalangeal joint of the thumb. Infections with these characteristics in IV drug abusers are most commonly caused by MRSA, and can affect any portion of the hand.

     

    Imahara et al retrospectively reviewed 159 hand infections treated in the operating room over an 11-year period. The examined data included known risk factors for MRSA, including human immunodeficiency virus infection, diabetes mellitus, intravenous drug use, incarceration, and homelessness. Intravenous drug use was the only independent risk factor for CA-MRSA infections.

     

    Boucher et al examined the trends in both nosocomial and community-associated MRSA infections and explored recent studies of the mechanisms that allow S. aureus to become resistant to currently available drugs.

     

    Incorrect Answers:

    1. Herpes simplex virus can cause Herpetic whitlow, as shown in Illustration A, typically presents on the fingers health care workers exposed to a carriers mouth. Usually, this infection appears as small ulcers or vesicles, and operative debridement is contraindicated.

    2. Candida albicans is a more rare hand infection typically associated with chronic paronychia, as shown in Illustration B.

    3. Escherichia coli is a less common cause of abscess formation in the hand.

    4. Eikenella is usually associated with "fight-bite" infections on the dorsal aspect of the MCP joint, and does not commonly occur after superficial lacerations. It can also rarely occur in IV drug users who clean their needles with saliva, as Eikenella is part of the normal oral flora. An example of an Eikenella infection is shown in Illustration C.

     

     

     

     

     

     

     

     

     

  20. Which of the following dorsal wrist compartments is incorrectly paired with its contents?

    1. Compartment 6: Extensor carpi ulnaris

    2. Compartment 5: Extensor digiti minimi

    3. Compartment 4: Extensor digiti communis, posterior interosseous nerve

    4. Compartment 3: Extensor pollicus longus, extensor carpi radialis longus

    5. Compartment 1: Abductor pollicus longus, extensor pollicus brevis

    CORRECT ANSWER: 4

    The dorsum of the wrist is subdivided into six compartments. The PIN is the only nerve found in the dorsal compartments. It is consistently found on the base of the fourth compartment. Anatomic structures within each compartment are:

     

    Compartment 1: Abductor pollicus longus, extensor pollicus brevis. Compartment 2: Extensor carpi radialis longus and brevis Compartment 3: Extensor pollicus longus

    Compartment 4: Extensor digiti communis, extensor indicis propius, posterior interosseous nerve

    Compartment 5: Extensor digiti minimi Compartment 6: Extensor carpi ulnaris

     

    Iwamoto et al performed a cadaveric study of the six compartments and septa. They determined that the 1/2 compartment had the thickest radial surface while the 3/4 septa was the thinnest. The fifth dorsal compartment had the lowest resistance to failure.

     

    Palmer et al also performed a cadaveric study on the extensor retinaculum. They concluded that the retinaculum provides a check rein to bowstringing and should be repaired during surgery.

     

    Grutter et al injected the PIN and AIN in fresh-frozen cadavers. In the first group, the AIN and PIN were injected via a single injection 1cm ulnar and 3cm proximal to Lister's tubercle. For group 2 (PIN alone), the injection site was 1cm ulnar to the proximal aspect of Lister's tubercle. Their technique led to accurate delivery of anesthetic to the PIN and AIN in 100% of the cadavers.

     

    Illustration A shows the 6 dorsal wrist compartments. Illustration B shows the injection site in the Grutter article.

     

     

     

     

     

     

     

     

     

  21. Figures A through E depict various conditions affecting the pediatric hand and wrist. For which of the depicted conditions is temporary scaphotrapeziotrapezoidal pinning most indicated?

     

     

     

     

     

     

     

     

     

     

     

     

     

    1. A

    2. B

    3. C

    4. D

    5. E

     

    CorreCt answer: 4

     

    Temporary scaphotrapeziotrapezoidal (STT) pinning is indicated for treatment of Kienbocks disease in adolescents as shown in Figure D. The radiograph shows increased density and slight lunate collapse. The result is a decrease in radiolunate contact stress while increasing the load on the radioscaphoid articulation. STT pinning is not indicated in any of the conditions explained below.

     

    Ando et al retrospectively reviewed the results of six adolescents treated with temporary scaphotrapezoidal (ST) pinning. All patients had an increase in wrist flexion/extension arc, strength, and lunate intensity on MRI from their preoperative baseline.

     

    Shigematsu et al published a case study on a single 11-year-old patient with wrist pain at rest and with use who was treated with temporary scaphotrapeziotrapedoidal (STT) pinning and cast immobilization for 8 weeks. Both wrist ROM and grip strength improved. Lunate revascularization was also

    seen on subsequent MRI.

     

    Incorrect Answers:

    Answer 1,2,3: Radial clubhand, scaphoid fracture, and hypoplastic thumb are not treated with temporary scaphotrapeziotrapezoidal pinning.

    Answer 5: Gymnast’s wrist is a distal radius physeal injury due to repetitive axial loading. Plain films will show physeal widening and hazy irregularity. The condition is not treated with temporary scaphotrapeziotrapezoidal pinning.

     

     

     

  22. Figure A depicts a child with syndactyly. Following surgical treatment, the most common complication involves which of the following?

     

     

     

     

    1. Nail plate

    2. Nail bed

    3. Web commissure

    4. Radial digital nerve

    5. Radial digital artery CORRECT ANSWER: 3

    Web creep, the most common complication of this procedure, is the distal migration of the web commissure seen in surgically corrected syndactyly patients. It is caused by abnormal scar tissue formation and increasing growth of underlying osseous structures. Informing parents of this complication preoperatively is advised.

     

    Deunk et al reviewed the long-term results of 27 patients treated with either STSG or FTSG at 21 years. The STSG group had increased flexion and

    extension lags but decreased finger abduction and increased graft breakdown. The FTSG had higher rates of web creep, hyperpigmentation and hair presence. The authors did not favor one technique over the other.

     

    Ricterman et al performed a radiographic analysis of web height in children. They were able to radiographically determine a standard web height quantification system using anatomic landmarks. This technique serves as the foundation for measuring web creep in syndactlyly.

     

    Illustration A is a clinical photo demonstrating web creep as a late complication.

     

     

     

     

     

     

  23. The physical exam finding demonstrated on the patient's right hand in the video (Figure V) is found with neuropathy of which of the following nerves?

    1. Musculocutaneous nerve

    2. Anterior Interosseious Nerve (AIN

    3. Radial nerve

    4. Ulnar nerve

    5. Median nerve CORRECT ANSWER: 4

    The video demonstrates Froment's sign on the patient's right hand, which is characterized by interphalangeal (IP) flexion during attempted key pinch, and is found in patients with ulnar neuropathy. Therefore it can be found with ulnar nerve compression in the cubital tunnel (Cubital Tunnel Syndrome) or in Guyon's Canal (Ulnar Tunnel Syndrome).

    Froment's sign is performed by having the patient pinch a piece of paper with the thumb IP joint extended against resistance (pulling paper away). It should be done with both hands side by side to compare them to each other.

     

    In a hand with a ulnar neuropathy, adductor pollicis (ulnar n.) is deficient, and can not flex the MCP joint to give pinch strength with an extended IP joint. The thumb compensates by recruiting the FPL (median n.) to flex the IP joint to give pinch strength. The result is, in a positive Froment's sign, the IP joint will flex (buckle) to try to give increased strength to the pinch.

     

    Illustration V shows a demonstration of the Froment's sign.

     

     

     

  24. A 24-year-old racquetball player presents after accidentally striking his racket against the wall during a match three months ago. He is tender to palpation over the hypothenar mass, and his pain is aggrevated by grasping. A radiograph and CT scan of his wrist are shown in Figures A and B. Which of the following treatment methods has been definitively shown in the literature to have a favorable outcome, and a high chance to return to pre-injury activities in patients with this injury?

     

     

     

     

     

     

    1. Activity restriction and continued monitoring

    2. Open reduction and internal fixation

    3. Casting for 6 weeks, followed by physical therapy

    4. Corticosteroid injection and immediate return to play

    5. Surgical excision CORRECT ANSWER: 5

    The patients history and imaging are consistent with a subacute hook of the hamate fracture. This is demonstrated by the carpal tunnel view radiograph in Figure A, and confirmed by the CT scan of the wrist in Figure B. CT scan of the wrist is usually indicated to definitively diagnose these fractures. Current literature supports the most favorable results and ability to return to pre-injury activities with excision of the fracture fragment. There is little available literature reporting the results of open reduction and internal fixation of these fractures.

    Rettig et al review traumatic wrist injuries in athletes. With regards to treatment of hook of the hamate fractures, they state that ORIF and excision are the two viable treatment options in athletes. Of these, the literature supports fragment excision, which has an average return to sport time of 7-10 weeks.

     

    Welling et al determined which wrist fractures are not diagnosed with initial radiography, using CT as a gold standard and identified specific fracture patterns. In their series, they found that only 40% of hamate fractures were diagnosed on plain radiography, suggesting that CT should be considered after a negative radiographic finding if clinically warranted.

     

     

     

  25. A child is seen in the pediatric orthopedic hand clinic for evaluation of a congenital deformity. A clinical photograph and radiograph are seen in Figures A and B. What is the next best step in this child's evaluation to rule out an associated autosomal-recessive lethal condition?

     

     

     

     

    1. Cardiac ultrasound and renal ultrasound

    2. Cardiac ultrasound, barium swallow and MRI

    3. LFTs, CBC and cardiac ultrasound

    4. Cardiac ultrasound, peripheral blood smear and MRI

    5. CBC, peripheral blood smear and chromosomal breakage analysis

    CORRECT ANSWER: 5

    The clinical and radiographic images depict a patient with radial club hand. This is associated with a number of congenital anomalies including Fanconi’s Anemia (FA), thrombocytopenia absent radius (TAR), Holt-Oram syndrome, VACTERL syndrome, and VATER syndrome. Although all these congenital anomalies are important to recognize and treat, none is more life-threatening than FA. FA is an autosomal-recessive condition resulting in aplastic anemia

    and eventual death. The typical presentation is between 6-9 years of age. It is the most common inherited form of aplastic anemia. Genetic testing will reveal increased chromosomal breakage. A CBC will show decreased leukocytes, red blood cells and platelets. Of the choices above, it is the only one which requires bone marrow transplantation for survival.

     

    DeKerviler et al reviewed many clinical and radiological features of FA. Congenital malformations affect multiple systems including the radial aspect of the forearm as well as the urinary system. They recommended ultrasound and imaging modalities for early detection of FA.

     

    Alter described hematologic disorders manifest in the pediatric upper extremity. Pediatric orthopedic surgeons may be the first to detect FA, Diamond-Blackfan anemia and TAR. As such, an understanding of the syndromes and inheritance patterns may aid in early detection and aid in future genetic counseling.

     

     

     

  26. A 39-year-old male presents with longstanding right wrist pain. He has failed conservative measures including prolonged immobilization. His radiographs and MRI are seen in figures A and B. Which of the following options is an accepted treatment option?

     

     

     

     

    1. Ulnar shortening osteotomy

    2. TFCC repair

    3. Radius core decompression

    4. Arthroscopic lunate chondroplasty and debridement

    5. Scapholunate ligament reconstruction

    CORRECT ANSWER: 3

    The patient in the clinical scenario has Kienbock's disease. Treatment options include a joint leveling procedure, or radius core decompression, which is thought to incite a local vascular healing response in the lunate.

     

    Sherman et al did a biomechanical study reviewing distal radius core decompression for Kienbock's disease. Although the procedure has good clinical outcomes for this disease process, their findings did not show any biomechanical explanation for these good outcomes.

     

    Illarramendi et al reviewed results of curettage of the distal radius and ulna metaphyseal bone through small cortical windows for the treatment of Kienbock's disease. They concluded that the decompression procedure had good results without any complications. Most patients had improvement in pain and were able to return to work.

     

    Incorrect Answers:

    Answer 1: Kienbock's disease is commonly associated with ulnar negative variance which is thought to lead to increased forces on the lunate leading to this disease. Therefore a ulnar shortening osteotomy would not be appropriate.

    Answer 2,4,5: Are not treatment options for this disease process.

     

     

     

  27. A 7-month-old boy has radial longitudinal deficiency. The initial work-up is negative for any cardiac, hematopoetic or renal abnormalities. He has good active elbow flexion and no other deformities. A clinical image and radiograph are seen in Figures A and

    B. Surgical management should include which of the following?

     

     

     

    1. Definitive splinting and stretching

    2. Pollicization of the index finger

    3. ECRB and ECRL transfers to the ulna

    4. Free fibula transfer to the forearm

    5. Ulna centralization and possible tendon transfers

    CORRECT ANSWER: 5

    The image and vignette are consistent with non-syndromic radial longitunidal deficiency (RLD). He has a viable thumb with good active elbow flexion, therefore the treatment of choice is centralization and tendon transfers to re-establish balance across the wrist. The goal of centralizing the carpus on the ulna is to improve reach and to stabilize tendons and muscle balance across the wrist.

     

    The decision for surgery is based on the range of motion of the elbow. In the case of a stiff elbow, a centralization should not be performed as doing so would prohibit the patient from reaching his/her face for dentition and feeding.

     

    RLD is commonly associated with Thrombocytopenia absent radius (TAR), Holt-Oram and Fanconi’s anemia. Classification of RLD is based on the Bayne and Klug system and takes into account the amount of remaining radius present:

    Type I: short distal radius

    Type II: short distal radius with residual growth plates Type III: small proximal radius

    Type IV: absent radius

     

    Incorrect Answers:

    Answer 1: For advanced cases (types III/IV), surgery is recommended Answer 2: The patient in the scenario has a thumb so pollicization is not indicated

    Answer 3: This is not a surgical option

    Answer 4: MTPJ transfers have been explored but not free fibula

     

     

  28. A 25-year-old right-hand dominant construction worker suffers an industrial injury as seen below. He is hemodynamically stable and his only injury is to the limb below. In terms of replantation of the affected limb, which of the following is true?

     

     

     

     

     

     

     

     

     

     

     

     

    1. Transpositional microsurgery offers the best results

    2. Replantation is contraindicated as the injury is through flexor zone II

    3. Replantation is contraindicated because of the extent of injury

    4. Heterotopic transplantation would offer the best function

    5. Anatomic replantation of hand offers the best results

    CORRECT ANSWER: 5

    The clinical scenario and images depict a through the palm amputation with the digits intact. The injury is a clean amputation with minimal avulsion. Thus, anatomic replantation of the entire hand is indicated. Digit transposition refers to using the salvageable digits and replanting them on the functionally important positions in the hand. Transpositional replantation is not indicated in the above scenario and would be more appropriate for a multidigit amputation as shown in Illustration A.

     

    Soucacos et al reviewed their results of transpositional digital microsurgery in

    34 patients. They found that transposition of a digit to the most functional part of the hand lead to a 2-point discrimination of 10-14mm in transposed digits and equivalent functionality of transposed digits with anatomically replanted digits.

     

    Schwabegger et al presented a case series of 13 patients with multiple digit amputations. The primary goal of surgery was function and secondly, cosmesis. They found the results of transpositional replantation similar to conventional replantation.

     

     

     

     

     

     

  29. A 68-year-old female office assistant reports left thumb pain that has progressively worsened over the past 2 years. She is left hand dominant and reports difficulty with opening jars and holding a coffee cup. On examination of the left hand she has a positive thumb carpometacarpal grind test and has a fixed deformity at the thumb metacarpalphalangeal joint. Figure A demonstrates the left hand grasping an object and Figure B shows a radiograph of the left thumb. What is the most appropriate next step in treatment?

     

     

     

     

     

     

    1. Carpometacarpal joint fusion and metacarpophalangeal joint volar capsulodesis

    2. Carpometacarpal joint resection arthroplasty and metacarpophalangeal joint volar capsulodesis

    3. Carpometacarpal joint resection arthroplasty and metacarpophalangeal joint fusion

    4. Carpometacarpal joint resection arthroplasty and temporary metacarpophalangeal joint percutaneous pin fixation

    5. Carpometacarpal joint fusion and metacarpophalangeal joint fusion

    CORRECT ANSWER: 3

    The patients history, examination, and images are consistent with thumb CMC (basilar) joint arthritis with associated MCP joint arthritis. At the MCP joint there is hyperextension of the thumb metacarpophalangeal (MCP) joint and adduction involving the first web space of the hand (Z deformity). Arthrodesis of the MCP joint is the treatment of choice when thumb MCP hyperextension

    exceeds 40°, the deformity is not passively correctable, or advanced degenerative changes are noted to affect the articulation.

     

    The review article by Armbruster and Tan state that when MCP joint hyperextension is:

    0° to 10°= Surgical intervention is not necessary when MCP hyperextension is less than 10°.

    10° to 20°= Percutaneous pinning of the MCP joint in 25° to 35° of flexion for 3-4 weeks may be performed independently or as an adjunct to EPB transfer. 20° to 40°= Capsulodesis of the volar aspect of the MCP joint is recommened to provide a check rein for hyperextension and Sesamoidesis has also been investigated as an adjunctive procedure.

     

    Cooney et al performed a Level 4 review of their CMC arthroplasty patients and found 15 patients with 17 revision arthroplasties in the treatment of mechanical pain related to instability or bone impingement. The revisions included soft-tissue interposition alone or soft-tissue interposition with ligament reconstruction and found that this provided satisfactory patient outcomes in more than 75% of the cases.

     

    Illustration A depicts the forces accounting for the observed adduction and hyperextension deformities. The arrowhead indicates the direction of subluxation of the base of the thumb metacarpal (due to incompetent volar beak ligament). The arrow represents the force vector of the EPB potentiating the MCP hyperextension deformity

     

     

     

     

     

  30. A 44-year-old male factory worker presents with a 7-month history of pain and paresthesias involving the palmar aspect of the left thumb, index finger, long finger, and the radial half of the ring finger. He reports that this often occurs at night when trying to go to sleep. He has a history of anemia and obstructive sleep apnea. Percussion over the volar wrist crease produces electric sensation distally in the hand and wrist flexion with the elbow in extension produces thumb paresthesias within 18 seconds. Figure A demonstrates a radiograph of the left hand. A sensory nerve conduction velocity test shows a distal sensory latency of 5.7 ms. Which of the following is the most appropriate next step in management?

     

     

     

     

    1. Phonophoresis and 6-week course of Vitamin B6 (pyridoxine)

    2. Occupational therapy with wrist massage and activity modification

    3. Wrist splinting

    4. 1-month course of nonsteroidal anti-inflammatory drugs [NSAIDs] and physical therapy

    5. 1-month course of bumetanide, smoking cessation, and physical therapy

    CORRECT ANSWER: 3

    The patients history, examination, and nerve conduction velocity tests (normal distal sensory latency is <3.5 ms) are consistent with carpal tunnel syndrome. There is Level 1 and 2 evidence supporting local steroid injection or splinting

    for the nonoperative treatment of carpal tunnel syndrome. Phonophoresis, Vitamin B6 (pyridoxine), heat therapy, bumetanide, and physical therapy are not considered the most appropriate options for carpal tunnel syndrome management.

     

    The AAOS clinical guidelines for carpal tunnel syndrome consist of 9 clinical recommendations supported with a grading of the recommendation and levels of evidence for the literature contributing to the recommendation.

     

    The use of neutral wrist splints for carpal tunnel syndrome is most useful for improving night-time symptoms. However wrist splinting is most functional at 30 degrees of extension, and the neutral splints can be functionally limiting when used during productive daytime hours.

     

     

     

  31. Which of the following options is the MOST reliable method to reduce the complication seen in Figure A

     

     

     

     

    1. Use of hard soled shoes

    2. Use of industrial toe protection footwear

    3. Compression stockings

    4. Thermal insulation footwear

    5. High protein diet CORRECT ANSWER: 4

    This patient presents with bilateral frostbite of the toes. The most reliable method to reduce the risk of cold exposure injury is to reduce thermal heat loss.

     

    Frostbite is a cold exposure injury to local tissues and can be classified as a first-, second-, third-, and fourth-degree injury, depending on the depth of tissue damage. Risk factors for injury include: prolonged exposure to sub-zero temperature, increased wind chill factor, direct skin contact with wind, water or metal, poor thermal insulation, alcohol abuse, smoking, peripheral vascular disease, diabetes, and prior cold exposure injury.

     

    Kuklane summarizes the research on cold protection of feet. The authors state that the toes have little local metabolic heat production because of their small muscle mass, and the capacity to generate heat decreases with tissue temperature. They concluded that footwear thermal insulation is the most important factor for protection against cold induced injury.

     

    Figure A shows a third degree frostbite injury to the toes, bilaterally. Illustration A shows the classification system of frostbite injury.

     

    Incorrect Answers:

    Answer 1: Comfortable shoes (not hard soled) have been shown to decrease the incidence of frostbite

    Answer 2: This is not a picture of an industrial toe crush injury.

    Answer 3: Compression stockings would not reduce the risk of thermal injury to the toes.

    Answer 5: Malnourished or chronically ill are also more susceptible to cold-induced injuries because of their decreased capacity for normal thermoregulation. However, this is not the most reliable measure to reduce the risk of thermal injury.

     

     

     

     

     

     

  32. A 23-year old male presents with a growing mass on the dorsum of his foot and reports pain with shoe wear. On exam, he has a firm nodular mass on the surface of his foot. Imaging is shown in Figures A, B, C, and D. A biopsy specimen is shown in Figure F. Which of the following is true regarding this condition?

     

     

     

     

     

     

     

     

     

     

     

     

     

     

     

     

    1. Histologically similar to giant cell tumor of bone

    2. Mass transilluminates on physical exam

    3. Metastasis may occur

    4. Radical excision is recommended

    5. Marginal resection is recommended

    CORRECT ANSWER: 5

    This patient has giant cell tumor of tendon sheath, (GCTTS), and recurrence is common after incomplete excision.

     

    GCTTS are associated with the many tendon sheaths in the foot and ankle. They also occur in the hand. The tumor presents as a well-defined nodule with or without pain. On ultrasound, a relationship of the lesion with adjacent tendon may be seen. The lesion also exhibits internal vascularity. Marginal resection is recommended and the tumor rarely recurs after complete excision. If not completely excised, the tumor has a high rate of recurrence. It is histologically similar to pigmented villonodular synovitis (PVNS).

     

    Bos et al. performed a review of foot tumors. They report that patients with GCTTS seek medical care as the lesion is compressed by shoewear, causing pain. Histologically, they exhibit a benign proliferation of giant cells, retinacular cells, and foam cells which pathologists report are related to PVNS. They report that the tumor may involute spontaneously and rarely recurs after marginal excision.

     

    Bouchard et al. performed a review of foot masses. They report that GCTTS and PVNS have similar appearances on MRI, which is the modality of choice for imaging. They tend to be hypointense on both T1- and T2-weighted images

    due to their hemosiderin content. GCTTS is almost always extra-articular, while PVNS is diffuse and intra-articular.

     

    Gibbons et al. perform a retrospective review of 17 patients with GCTTS in the foot and ankle. All patients were treated with complete excision. They report that MRI was the most useful preoperative investigation. They found no recurrences in patients available for follow up at a mean of 85 months after surgery.

     

    Figures A and B are radiographs demonstrating dorsal soft tissue swelling. Figure C is a sagittal T2 MRI, Figure D is a coronal T1 MRI, and Figure E is a coronal T2 MRI all demonstrating a heterogeneous soft tissue mass at the dorsum of the foot, lifting the EDL tendons with the involvement of the EDB. Figure F demonstrates histiocyte-like tumor cells, lymphocytes, hemosiderin-laden macrophages, and giant cells consistent with giant cell tumor of tendon sheath.

     

    Incorrect Answers:

    Answer 1: GCTTS is histologically similar to PVNS.

    Answer 2: A transilluminating mass is consistent with a ganglion cyst. Answer 3: Metastases have not been reported with GCTTS.

    Answer 4: Radical excision is not recommended for GCTTS.

     

     

     

  33. A 31-year-old mother of a 2-month-old infant complains of radial sided wrist pain. Corticosteroid injections should be directed into what anatomic area?

    1. First carpometacarpal joint

    2. Carpal tunnel

    3. First dorsal compartment near the radial styloid

    4. A1 pulley of thumb

    5. At the crossing of the first and second dorsal compartments

    CORRECT ANSWER: 3

    There is an association between the postpartum state and de Quervain’s tenosynovitis. De Quervain’s is a pathologic process of the 1st dorsal (extensor) compartment which contains the extensor pollicis brevis and abductor pollicis longus tendons. The best choice is #3 because of the very common and known association of postpartum state and de Quervain’s as well as the potential for resolution with appropriately placed steroid injection.

    Answer #1 refers to basal joint arthritis which is typically seen in older

    patients. Answer #2 refers to carpal tunnel syndrome, which would present with paresthesias in the median nerve distribution. Answer #4 refers to a trigger thumb. Answer #5 alludes to intersection syndrome which is generally more proximal to the wrist and results from inflammation at crossing point of 1st dorsal compartment (APL and EPB) and 2nd dorsal compartment (ECRL, ECRB). To review, the wrist extensor compartments (from radial to ulnar) are:

    1) APL & EPB; 2) ECRL & ECRB (common radial wrist extensors); 3) EPL; 4) EIP & EDC; 5) EDM; 6) ECU.

     

     

     

  34. A 2-year-old boy has the upper limb deformity seen in Figures A and B. All of the following are associated with this condition EXCEPT?

     

     

     

     

    1. Fanconi's Anemia

    2. Holt-Oram syndrome

    3. VATER syndrome

    4. VACTERL syndrome

    5. Osteogenesis Imperfecta CORRECT ANSWER: 5

    The clinical presentation is consistent with radial longitudinal deficiency, also known as "radial clubhand", which is associated with all of the listed conditions except for osteogenesis imperfecta.

     

    Maschke et al report "radial longitudinal deficiency encompasses a spectrum of upper limb dysplasias and hypoplasias. The clinical presentation of the involved upper limb is often more obvious than the potentially life-threatening associated systemic conditions. All children presenting with radial longitudinal deficiency, regardless of severity, require a renal ultrasound, echocardiogram, and complete blood count to evaluate the potential for associated systemic conditions; these include Fanconi’s anemia, the Holt-Oram syndrome, and the VATER (vertebral anomalies, anal atresia, tracheoesophageal fistula, esophageal atresia, renal agenesis) syndrome or VACTERL (vertebral anomalies, anal atresia, cardiac abnormalities, tracheoesophageal fistula, renal agenesis, and limb defects) association."

     

     

     

  35. A 35-year-old female office worker reports 6 months of deep aching pain that is worse at night on her lateral dominant elbow. The pain also worsens with repetitive movements. On physical exam, the patient has tenderness located 4cm distal to the lateral epicondyle over the mobile wad, and she has subtle weakness of the wrist extensors. Extending her long finger against resistance with a flexed wrist is very painful for her. She also complains of her pain worsening at night. What is the most likely diagnosis?

    1. Radiocapitellar Arthritis

    2. Radial tunnel syndrome

    3. Carpal tunnel syndrome

    4. Lateral epicondylitis

    5. Intersection Syndrome

    CORRECT ANSWER: 2

    The patient has radial tunnel syndrome, which often presents with insidious onset of pain and tenderness several centimeters distal to the lateral epicondyle, and pain elicited with active extension of the long finger against resistance can help differentiate the condition from lateral epicondylitis.

     

    Radial tunnel syndrome is a compressive neuropathy that can occur between the mobile wad laterally and the biceps aponeurosis and brachialis insertion medially as the nerve courses over the radiocapitellar joint into the forearm. Patients usually have diffuse pain over the site of the radial tunnel, sometimes

    have radiating pain in the distribution of the superficial radial nerve, and occasionally have subtle weakness or fatigue of the wrist and extrinsic finger extensors. Initial treatment should include conservative measures such as stretching, activity modification, and NSAIDS; Injections can be performed for both diagnostic and therapeutic reasons. Surgical intervention is indicated if pain persists despite exhausting conservative measures. The most common anatomic causes of radial tunnel syndrome are fibrous adhesions between the brachialis and brachioradialis, the Leash of Henry (radial recurrent vessels), the fibrous edge of the ECRB, the arcade of Fröhse (supinator arch), and fibrous bands of the leading edge of the supinator muscle.

     

    Dang et al. discuss compression neuropathies of the upper extremity in their 2009 review article. They highlight the importance of the clinical exam in diagnosing radial tunnel syndrome, especially the location of pain, which is distal to that of lateral epicondylitis. Additionally ruling out other less common diagnoses on the differential can be assisted by EMG (radiculopathy or plexopathies), MRI (tumor or other causes of mass effect), and diagnostic injections.

     

    Illustration A shows the anatomy of the five common sites of compressing in the radial tunnel.

     

    Incorrect Answers:

     

    1. Radiocapitellar arthritis would not be antagonized by stretch of the common extensors of the wrist

    1. Carpal tunnel syndrome is diagnosed by evidence of nerve compression of the median nerve at the wrist and should not be associated with pain near the origin of the wrist extensors

    2. Lateral epicondylitis can very much mimic radial tunnel syndrome; however, the location of the pain and tenderness on exam can be a very helpful

    3. Intersection syndrome is a chronic tenosynovitis of the ECRL and ECRB characterized by pain at the intersection of the 1st and 2nd dorsal compartments of the wrist

     

     

     

     

     

     

  36. A professional baseball player develops acute hand pain after fouling off a pitch. He is tender over the hypothenar eminence and has paresthesias in the ring and small fingers. Which radiographic view is most likely to reveal the pathology?

    1. PA wrist

    2. AP wrist in ulnar deviation

    3. Lateral wrist

    4. Carpal tunnel

    5. Scaphoid CORRECT ANSWER: 4

    Plain radiographs usually do not reveal the fracture; carpal tunnel and supinated oblique views should be obtained. Diagnosis is confirmed by CT scan and bone scan.

     

    Fractures of the body of the hamate may occur from trauma and may occur in combination with fractures of the base of the fourth and fifth metacarpals.

    Fractures of the hook of the hamate are more common in athletes. The incidence of hook of the hamate fractures is 2% to 4% of all carpal fractures.

    The mechanism of injury is thought to be caused by abutment of the hook on an object or by a shearing force applied by the flexor tendon of the small and ring fingers. The injury usually occurs in athletes who participate in baseball, golf, and racquet sports because of the position of the implement in the hand.

     

    Rettig reviewed hand injuries in athletes. He noted that hamate hook fractures occur in a watershed area that may explain the high incidence of nonunion post fractures. Hook of the hamate fracture must be suspected in athletes participating in racquet sports, golf, or baseball who are seen with ulnar wrist pain. Examination reveals tenderness over the hook of the hamate, which lies on a line between the pisiform and second metacarpal head. Treatment of hook of the hamate fractures in athletes varies from casting to open reduction and internal fixation to excision.

     

    Bishop and Beckenbaugh reported 21 cases of this fracture: 17 were treated by excision, 3 underwent ORIF, and 1 had casting. Although two of three fractures that were treated with ORIF healed, many authors recommend excision, which has an average return to sport of 6 to 10 weeks.

     

    Illustration A shows a representative carpal tunnel view, with the bony anatomy labeled.

     

     

     

     

     

     

  37. The sural artery provides the vascular supply to which musculocutaneous flap?

    1. soleus

    2. gastrocnemius

    3. latissimus dorsi

    4. tibialis anterior

    5. peroneus longus CORRECT ANSWER: 2

    The sural artery supplies the both heads of the gastrocnemius and is the pedicle for rotational flaps. Eighty-five percent of the time there is a single vascular source.

     

     

     

  38. Which of the following nerves has the most favorable regenerative potential in restoring motor function after a graft repair within half a year after being injured?

    1. Median

    2. Ulnar

    3. Radial

    4. Tibial

    5. Peroneal CORRECT ANSWER: 3

    Of the choices listed, the radial nerve has the best opportunity for recovery.

     

    Roganovic performed a prospective study of 393 graft repairs of the median, ulnar, radial, tibial, peroneal, femoral, and musculocutaneous nerves which showed that peripheral nerves differ significantly regarding the motor recovery potential, and the difference depends on the level of nerve repair. The following nerves had excellent recovery potential: the radial, musculocutaneous, and femoral nerves. The following nerves had moderate recovery potential: the median, ulnar, and tibial nerves. The following nerve had poor recovery potential: the peroneal nerve.

     

    Mohler et al, recommends testing nerve action potentials at the time of nerve exploration to guide surgical treatment.

     

     

     

  39. All of the following are contents of the carpal tunnel EXCEPT:

     

    1. Flexor pollicis longus (FPL)

    2. Flexor digitorum sublimis (FDS)

    3. Flexor digitorum profundus (FDP)

    4. Flexor carpi radialis (FCR)

    5. Median nerve CORRECT ANSWER: 4

      The only neurovascular structure that runs in the carpal tunnel is the median nerve. Flexor carpi radialis is (FCR) is not a tendon within the carpal tunnel. In summary, the carpal tunnel contains the median nerve, FPL and 4 tendons each of the FDP and FDS. Of note, with respect to the FDS tendons, the FDS tendons to the 3rd and 4th digit are volar to the FDS tendons to the 2nd and 5th digit.

       

       

       

  40. A 40-year-old male sheet metal worker sustained a crush injury to his hand. His hand was treated in a short arm splint after closed reduction and percutaneous pinning of multiple metacarpal fractures. The patient’s fractures healed uneventfully however, he presented six months later with the deformity shown in Figure A. What pathoanatomic process is responsible for his deformity?

     

     

     

     

    1. Volar plate laxity and tethering of the lateral bands at the proximal interphalangeal joints

    2. FDP laceration distal to the origin of the lumbricals

    3. Adhered FDP tendon of the middle finger

    4. Imbalance between spastic intrinsics and weak extrinsics

    5. Imbalance between strong extrinsics and deficient intrinsics

    CORRECT ANSWER: 5

    The clinical presenatation is consistent with a claw hand deformity characterized by MCP hyperextension and IP joint flexion.

     

    Imbalance between strong extrinsics and deficient intrinsics is the pathoanatomic process of a claw hand, also called intrinsic minus hand deformity. Intrinsic minus hand posture can result from a variety of causes including ulnar or median nerve palsy, Volkmann’s ischemic contracture, leprosy, hereditary motor-sensory neuropathy, failure to splint a crush-injured hand using intrinsic plus posture, or compartment syndrome of the hand, as was the case in this clinical vignette.

     

    Ouellette et al performed a retrospective review of nineteen patients managed with fasciotomy for compartment syndrome of the hand. They found that the most consistent clinical finding in making the diagnosis of compartment syndrome was a tense, swollen hand in an intrinsic minus position.

     

    Dellaero et al, in their review of compartment syndrome of the hand, discuss the etiology, diagnosis, and treatment of acute hand compartment syndrome. They emphasize that the main goal in the management of ischemic contracture is restoration of function; however the return of normal limb functionality is an unlikely result.

     

    Figure A is a clinical photograph showing a classic claw hand deformity. Notice the MCP hyperextension and IP joint flexion.

     

    Incorrect Answers:

    1. Answer choice 1 describes the pathoanatomy of swan neck deformity leading to hyperextension of the PIP joint and DIP flexion.

    2. Answer choice 2 describes the mechanism for lumbrical plus deformity characterized by paradoxical extension of the IP joints while attempting to flex the digits.

    3. Answer choice 3 describes the mechanism for a quadrigia effect characterized by an active flexion lag in multiple digits adjacent to an FDP injury.

    4. Answer choice 4 describes the pathoanatomy of an intrinsic plus hand characterized by MCP flexion and IP joint extension.

     

     

     

  41. A 42-year-old sustains a left finger injury while attempting to catch a baseball for his son. He presents with left, long finger pain and an inability to extend his middle finger at the distal interphalangeal

    joint. A radiograph after closed reduction and splinting is shown in Figure A. What is the best course of treatment?

     

     

     

     

    1. Reduction and pinning

    2. Repeat splinting of the distal interphalangeal joint in extension

    3. Splinting of the distal and proximal interphalangeal joints in extension

    4. Observation

    5. Fusion of the distal interphalangeal joint

    CORRECT ANSWER: 1

    The radiograph depicts a bony mallet injury with volar subluxation of the distal phalanx after splinting of the DIP joint in extension, which is an indication for reduction and pinning.

     

    A mallet deformity is caused by disruption of the terminal extensor tendon distal to DIP joint. Occasionally, a bony avulsion of the distal phalanx is noted on radiographs. "Bony" mallet fingers will rarely require surgical fixation. It is important to attempt to splint a bony mallet injury and get a new radiograph prior to making the decision for operative treatment. Indications for surgical management of this condition include volar subluxation of the distal phalanx even after DIP splinting.

     

    Stern et al. found a higher long-term complication rate with surgical treatment of mallet injuries. He also noted 15 degrees more DIP flexion at follow-up in the splinting group compared to the surgical group.

     

    Pegoli et al. describe an extension block technique for treatment of this injury with good results. Their indications for surgery included the presence of a large bone fragment, and palmar subluxation or the loss of joint congruity of the distal interphalangeal joint.

     

    Theivendran et al. review the surgical treatment of DIP joint fractures and state that 30% articular involvement is an indication for operative treatment.

    Figure A shows a lateral radiograph with a large intra-articular bony avulsion fragment and volar subluxation of the distal phalanx.

     

    Incorrect Answers:

    Answer 2,3,4: This patient meets the indications for ORIF and nonoperative modalities would not be appropriate.

    Answer 5: A DIP fusion in a young patient would not be appropriate.

     

     

     

  42. A 30-year-old healthy female sustains a traumatic digit amputation while working at a factory. Which of the following is a absolute indication for digit replantation in this patient?

    1. Isolated amputation of the index finger proximal to the FDS insertion

    2. Amputation due to crush injury

    3. Replantation of an index finger with a segmental injury

    4. Isolated amputation of the thumb proximal to the FPL insertion

    5. Presentation 13 hours after the amputation occurred

    CORRECT ANSWER: 4

    The unique functional role of the thumb in opposition and pinch dictates that it be replanted whenever possible in a healthy patient, regardless of the level of amputation. The remainder of the answer choices are relative contraindications for digit replantation.

     

    Boulas et al outline indications and contraindications for digit replantation after traumatic amputation. Contraindications to replantation include multilevel or segmental injury, a single digit proximal to the FDS insertion, a severe crush or mangling injury, extreme contamination, prior impaired function, concomitant life-threatening injury, severe medical problems, anesthetic risk, and major psychiatric disorder.

     

    Waikakul et al determined the influencing factors of the immediate and late outcome of replantation and revascularization of amputated digits. They found that the type of injury was the most important factor influencing immediate and late outcomes. They also determined that connecting the profundus tendon stump of the proximal part to the superficialis tendon of the amputated part gave a better result than two tendon repair and repairing only the profundus tendon.

     

    Illustration A shows a clinical photo of an isolated amputation of the thumb proximal to the FPL insertion.

    Incorrect Answers:

    Answer 1: Replantation of a single digit proximal to the FDS insertion is associatedwith poor results related to the loss of PIP joint motion due to flexor sheath adhesion formation.

    Answer 2: Crush or mangling injury is associated with serious damage to tissues, which are at risk for infection, problematic healing, and scarring, thereby contributing to a poor outcome.

    Answer 3: Segmental injury to the index finger is a contraindication to replantation due to poor function post-operatively.

    Answer 5: Prolonged warm ischemia time, defined as more than 12 hours, is associated with replantation failure.

     

     

     

     

     

     

  43. A 22-year-old college student presents with significant finger pain after coming into contact with liquid nitrogen in his chemistry lab. A clinical photo of the affected finger in shown in Figure A. What is the most appropriate next step in treatment?

     

     

     

    1. Blister debridement and hyperbaric oxygen therapy

    2. Drainage of the blister with the overlying skin left intact

    3. Full thickness blister and skin debridement with local flap coverage

    4. MRI scan of the digit to assess degree of soft tissue damage

    5. Wet to dry twice-daily dressing changes to the digit

    CORRECT ANSWER: 2

    The clinical presentation is consistent with a hemorrhagic blister due to acute frostbite injury. Of the options presented, the most appropriate treatment is drainage of the blister with the overlying skin left intact. Hemorrhagic blisters represent deeper injuries, and débriding them could lead to desiccation of the underlying dermis. Alternatively, intact blisters can be left in place and wrapped in dry gauze dressings until they resolve.

     

    Golant et al completed a review article discussing cold exposure injuries to the extremities. They state that frostbite, the most serious peripheral injury, results in tissue necrosis from direct cellular damage and indirect damage secondary to vasospasm and arterial thromboses. With regards to treatment, the authors conclude that débridement of necrotic tissues is generally delayed until there is a clear demarcation from viable tissues, a process that usually takes from 1 to 3 months from the time of initial exposure. They advocate drainage of hemorrhagic blisters, leaving the overlying skin intact.

     

    Bruen et al review the treatment of digital frostbite in their current concepts review. They state that physical examination results that are concerning for severe upper-limb injury include the absence of Doppler pulse signals, absent capillary refill, dark purple discoloration of the digits, and hemorrhagic blisters. They state that intact blisters should left in place and wrapped in dry gauze dressings until they resolve. These findings should lead to further perfusion evaluation with technetium-99m triple-phase bone scanning or angiography.

    Incorrect Answers:

    Answer 1: Hyperbaric oxygen therapy is not indicated as initial treatment of frostbite injury.

    Answer 3: Full thickness skin débridement should be delayed until full demarcation of the injury is determined. This can take as long as three months in some cases.

    Answer 4: MRI scan is not indicated initially.

    Answer 5: Wet to dry dressing changes are most beneficial in the presence of an open wound, as removal of the dressing when it dries removes drainage and debris. This treatment would not be beneficial in the presence of a blister with the overlying skin intact.

     

     

     

  44. The patient sustains the injury seen in Figure A from a gunshot injury. The physical exam is notable for lack of sensation in his fourth and fifth digits as well as a positive Froment's sign. Which of the following factors has not been shown to be a significant prognostic indicator of functional recovery following nerve repair?

     

     

     

     

    1. Duration to time of repair

    2. Repair level

    3. Length of repair

    4. Postoperative physical rehabilitation

    5. Type of autograft used

    CORRECT ANSWER: 5

    The clinical scenario describes an ulnar nerve laceration. Studies have shown that the ulnar nerve does not typically have good outcomes after nerve repair.

    (worse recovery than repairs of the tibial, radial, femoral, and musculocutaneous nerves)

     

    Nerve injuries from gunshot injuries (GSWs) can cause both a direct injury to the nerve as well as surrounding structures (zone of injury). Many factors including age of patient, time to repair, repair level, and length of repair have been shown to be important determinants in nerve recovery following repair. The type of nerve graft (sural, saphenous, etc) used has not shown to be statistically significant in terms of functional recovery after nerve repair.

     

    Secer et al.(J. Neurosurg) reviewed 2210 peripheral nerve lesions in 2106 patients which were injured by a GSW and who were treated surgically. Of the peripheral nerves repaired surgically, the tibial, median, and femoral nerve lesions showed the best recovery rate. The deep peroneal nerve, ulnar nerve, and brachial plexus lesions had the worst recovery.

     

    Secer et al.(Surg. Neur.) found that of 455 patients with 462 ulnar nerve lesions only a good outcome was noted in 15.06% of patients who underwent high-level repair, 29.60% of patients who underwent intermediate-level repair, and 49.68% of patients after low-level repair. The authors also noted that a better functional recovery was noted in patients who were treated earlier.

    Figure A shows a distal humerus fracture caused by a GSW. Incorrect Answers

    Answer 1: Earlier nerve repairs typically have better functional results.

    Answer 2: The lower level of nerve repair (more distal), the better functional results.

    Answer 3: Shorter length of the nerve repair typically leads to better functional results.

    Answer 4: Pre and post operative physical rehabilitation after nerve repairs has been shown to have better results.

     

     

     

  45. A 34-year-old male sustains the closed finger injury shown in Figure A one week ago. He undergoes closed reduction and pinning shown in Figure B to correct alignment. Which of the following is responsible for the apex palmar fracture deformity noted on the preoperative radiographs?

     

     

     

     

     

     

    1. Indirect pull of the central slip on the distal fragment and the interossei insertions at the base of the proximal phalanx

    2. Intrinsic muscle fibrosis and intrinsic minus contracture

    3. PIP joint volar plate attenuation and extensor tendon disruption

    4. Rupture of the central slip with attenuation of the triangular ligament and palmar migration of the lateral bands

    5. Flexor tendon disruption with associated overpull of the extensor mechanism

    CorreCt answer: 1

     

    The clinical presentation is consistent with a transverse proximal phalanx fracture. These fracture have an apex palmar angulated deformity under the indirect pull of the central slip on the distal fragment and the interossei insertions at the base of the proximal phalanx.

     

    If proximal phalanx fractures are allowed to heal with the apex palmar deformity, an extensor lag will result. Therefore CRPP or ORIF is indicated in transverse fractures with > 10° angulation. To correct this deformity prior to surgical fixation, the MCP joint should be flexed, which allows the extensor mechanism as a whole to function as a tension band to help reduce the fracture. This is referred to as intrinsic plus splinting. Collateral ligament, capsule, and intrinsic muscle attachments render transverse fractures in the proximal 6 to 9 mm of the P1 base more stable than fractures located distally.

     

    Henry provides a review of fractures of the proximal phalanx and metacarpals. He states that most transverse or short oblique P1 fractures without comminution are best stabilized by two 0.045-inch K-wires placed longitudinally through the fully flexed MCP joint. A single wire alone risks rotational malunion, but some fracture patterns may provide inherent rotational stability that would allow use of one wire for angular control.

     

    Figure A shows a transverse fracture of the proximal phalanx with apex volar angulation. Figure B shows two K-wires placed transarticular through the MCP joint in a flexed (intrinsic plus) posture to correct the deformity and stabilize the fracture.

     

    Incorrect Answers:

    Answer 2: Intrinsic muscle fibrosis and contracture is usually associated with chronic crush injuries and significant soft tissue damage.

    Answer 3: This is describing a swan neck deformity. Answer 4: This is describing a Boutonnierre deformity.

    Answer 5: Flexor tendon disruption is not likely in this closed injury pattern.

     

     

     

  46. A 54-year-old male presents with a slowly enlarging mass on the dorsum of his left wrist which has been present for 3 years. He denies any significant symptoms. Physical exams shows a 1 cm palpable mass. A MRI is shown in Figure A. A biopsy of this lesion would most likely show?

     

     

     

    1. Synovial cells with mucin accumulation

    2. Proliferating histiocytes of moderate cellularity and frequent multinucleated giant cells

    3. Polymorphonuclear neutrophils

    4. Spindle cells arranged in intersecting bundles

    5. Lipocytes, spindle cells, and scattered atypical giant cells

    CORRECT ANSWER: 1

    The clinical presentation is consistent with a ganglion cyst. Histology of a ganglion cyst would show a mucin filled synovial cyst.

     

    Ganglion cysts are the most common mass found on the hand or wrist. Dorsal ganglions originating from the scapholunate (SL) ligament are the most common (60%). They are caused by trauma, mucoid degeneration, or synovial herniation. On exam, they appear fixed to the underlying deep tissue, but not to the skin and are commonly translucent to light illumination. Radiographs of a ganglion will be normal, although a T2-weighted MRI axial image of the wrist will show increased signal where the cyst is located.

     

    Nahra et al. give a thorough review of ganglion cysts including known epidemiology, etiology, and treatment. He notes that dorsal ganglions are the most common (60% to 70%) and are found between the third and fourth dorsal compartments arising most commonly from the scapholunate ligament. Supportive splints and anti-inflammatories in conjunction with aspiration are an important part of nonoperative management. Aspiration of dorsal ganglions

    (not recommended for volar ganglions) yields a recurrence rate of around 50%. Surgical intervention is not common, but there is a low recurrence rate when performed.

     

    Peh et al. reviewed the MRI features found for benign soft tissue masses in the hand, especially ganglion cysts. They noted that ganglion cysts show an increased signal intensity on T2-weighted MRI images.

     

    Figure A is a T2 weighted MRI showing signal intensity in the fluid of the cyst consistent with a ganglion cyst. Illustration A is a clinical photo of a ganglion cyst.

    Illustration B shows a histologic specimen of a ganglion cyst Incorrect Answers:

    Answer 2: Proliferating histiocytes of moderate cellularity and frequent

    multinucleated giant cells would be consistent with Giant-cell tumor of tendon sheath.

    Answer 3: Polymorphonuclear neutrophils would be the primary cell type with infection and are the predominant cells in pus, accounting for its whitish/yellowish appearance.

    Answer 4: Spindle cells arranged in intersecting bundles would be consistent with a Schwannoma.

    Answer 5: Lipocytes, spindle cells, and scattered atypical giant cells are the histologic findings of a pleomorphic lipoma.

     

     

     

     

     

     

     

     

     

  47. A 27-year-old male presents with finger pain 2 days after suffering an injury while playing basketball. Physical exam shows swelling of the distal interphalangeal joint with no evidence of open injury. A radiograph is shown in Figure A. Which of the following is the most appropriate treatment at this time?

     

     

     

    1. Extension splinting of DIP joint for 6-8 weeks

    2. Closed reduction and percutaneous pinning

    3. Open reduction and internal fixation

    4. DIP arthrodesis

    5. Swan neck deformity correction

    CORRECT ANSWER: 1

    The clinical presentation is consistent with a non-displaced bony mallet finger without joint subluxation. Extension splinting of the DIP joint for 6-8 weeks is the most appropriate treatment.

     

    A mallet finger is a deformity caused by disruption of the terminal extensor tendon distal to DIP joint. Treatment is dictated by the degree of displacement and acuity of injury. Acute injuries with minimal displacement and no joint subluxation are treated with extension bracing for 6-8 weeks. ORIF or closed reduction and percutaneous fixation is indicated for chronic injuries or acute injuries with volar displacement of the distal phalanx, a >2mm articular step-off, or when a majority (>50%) of the articular surface is involved.

     

    Pegoli et al. report the results of extension block Kirschner wire fixation for the treatment of mallet fractures of the distal phalanx in 65 consecutive patients. Their results showed 46% excellent, 32% good, 20% fair and 2% poor results. The recommend the following indications for operative treatment: presence of a large bone fragment, palmar subluxation, or the loss of joint congruity of the distal interphalangeal joint.

     

    Theivendran et al. report operative fixation is indicated when more than 30% of the articular surface is involved with or without subluxation of the joint.

    They summarize the management options for intra-articular distal interphalangeal fractures, placing particular emphasis on surgical treatment.

     

    Figure A shows a non-displaced bony mallet Injury. Illustration A shows an example of an extension splint used for non-operative management of mallet injuries. Illustration V is a video showing the surgical technique for a Mallet finger.

    Incorrect Answers:

    Answer 2: Closed reduction and percutaneous pinning is indicated for a displaced mallet finger injury with joint subluxation.

    Answer 3: Open reduction and internal fixation is indicated for displaced, subluxed mallet finger injuries that can not be reduced closed.

    Answer 4: DIP arthrodesis is indicated in patients with a painful, stiff, arthritic DIP joint.

    Answer 5: Swan neck deformity correction is indicated for a chronic mallet finger that has led to a swan neck deformity.

     

     

     

     

     

     

  48. What is the optimal treatment for the proximal phalanx fracture shown in Figure A?

     

     

     

    1. Open reduction and placement of two 0.045-inch K-wires placed longitudinally through the metacarpal head

    2. Application of a 1.5-mm straight plate applied dorsally through and extensor tendon splitting approach

    3. Open reduction and lag screw fixation with 1.3mm screws through a radial approach

    4. Placement of a 1.5-mm condylar blade plate through a radial approach

    5. Open reduction and retrograde passage of two 0.045-inch K-wires retrograde trough the PIP joint

    CorreCt answer: 3

     

    Open reduction and lag screw fixation through a radial approach is the treatment of choice for long oblique proximal phalanx fractures.

     

    Lag screws (1.3 mm is preferred to 1.5 mm) can achieve stability through interfragmentary compression in a two-part spiral or long oblique fracture of the proximal phalanx that rivals the stability of an intact bone. This capability, combined with the need to achieve precise correction of rotation in spiral fractures, makes open reduction and lag screw fixation the treatment of choice for the spiral P1 shaft fracture.

    Kawamura et al. discuss the treatment options for closed phalangeal and metacarpal fractures. They state that percutanous fixation with K-wires can be successful in the setting of an adequate closed reduction. However, lag screw fixation may be the best choice for open fixation of long oblique phalangeal and metacarpal fractures.

     

    Henry et al. outline the methods of stablization for phalangeal and metacarpal fractures. With regards to long oblique proximal phalanx fractures, they state that open reduction and lag screw fixation the treatment of choice for the spiral P1 shaft fracture. This is due to the lack of inherent stability of the long fracture segment, and the tendency for the fracture to shorten even in the presence of smooth k-wire fixation.

     

    Freeland et al. provide a review which cites new developments in the treatment of extra-articular hand fractures in adults.

     

    Figure A shows a long obligue fracture of the index finger proximal phalanx. Illustration A demonstrates fixation of this injury with interfragmentary lag screws.

     

    Incorrect Answers:

    Answers 1,5: Most transverse P1 fractures without comminution are best stabilized by two 0.045-inch

    K-wires placed longitudinally through the metacarpal head or PIP joint, depending on the location of the fracture.

    Answers 2,4: Application of plates is usually reserved for comminuted bicondylar phalanx fractures, and is performed through a radial approach if possible to avoid postoperative adhesions.

     

     

     

     

     

     

  49. A 22-year-old man presents with pain and hyperesthesias in the dorsoradial wrist and thumb 6 months after undergoing wrist arthroscopy for an acute wrist injury. Placement of which of the following portals is likely responsible for his symptoms?

    1. 6U

    2. 6R

    3. 1-2

    4. MCU

    5. 4-5

     

    CorreCt answer: 3

     

    The 1-2 wrist arthroscopy portal is the only portal listed that places the superficial branch of the radial nerve (SBRN) at risk.

     

    Wrist arthroscopy plays an important diagnostic and therapeutic role in injuries about the wrist, but carries a 2% complication rate. The 1-2 portal is placed between the ECRB and APL. Care must be taken when accessing this portal,

    due to reported injuries to the superficial branch of the radial nerve, as well as radial artery.

     

    Kilic et al. dissected 6 cadavers to determine the course of the SBRN. They found that it was .9mm from the 1-2 portal at its closest, and that care should be taken when creating this portal given frequent variations in course.

     

    Auerbach et al. dissected 20 cadavers to determine the course of the SBRN. In all specimens, the nerve arose between the brachioradialis and ECRL ~8cm proximal to the radial styloid. On average, 5.8 branches of the SBRN crossed the wrist joint. They found some variability within the course, and urge surgeons to be respectful of the nerve during dissection.

     

    Tryfonidis et al performed a cadaveric study to determine the anatomic relation of dorsal wrist arthroscopy portals and superficial nerves. They found that the 1-2 portal was only 1.82 mm from the SBRN. Compared to the 3-4 portal, 4-5 portal, 6U portal and midcarpal portal, the 1-2 portal was the closest in proximity to any sensory nerve.

     

    Illustration A shows the various portals used in wrist arthroscopy. Illustration B shows the relation of the SBRN to the various arthroscopic portals.

     

    Incorrect Answers:

    Answer 1: The 6U portal places the dorsal sensory branch of the ulnar nerve at risk.

    Answer 2: The 6R portal places the dorsal sensory branch of the ulnar nerve at risk.

    Answer 4: The MCU portal places the EDC and EDM tendons at risk. Answer 5: The 4-5 portal places the EDC and EDM tendons at risk.

     

     

     

     

     

     

     

     

     

  50. What is a potential complication of an amputation at the level of the distal interphalangeal joint?

    1. Central slip rupture

    2. Swan neck deformity

    3. Boutonniere deformity

    4. Lumbrical plus finger

    5. Quadrigia effect

    CorreCt answer: 4

     

    A lumbrical plus finger is descibed as paradoxical extension of the IP joints while attempting to flex the fingers. In the case a lumbrical plus finger secondary to a DIP amputation, the PIP will extend upon attempted finger flexion.

     

    The review article by Parkes describes how the lumbricals originate from the FDP. When the FDP is lacerated or amputated, FDP contraction leads to pull on the lumbricals. This leads to shortening of the lateral bands and paraodoxical PIP and DIP extension. (Illustrations A-C). There are several causes of lumbrical plus finger including (1) FDP laceration or rupture distal to the lumbrical origin, (2) amputation of the DIP distal to central slip insertion, and

    (3) excessively long flexor tendon graft. Treatment consists of lumbrical release at the level of the flexor sheath in the palm, which then prevents paradoxical PIP extension.

     

    Quadrigia may occur when the profundus is advanced of greater than 1 cm in repair. The FDP tendons share a common muscle belly, and distal advancement of one tendon will effect the flexion strength of the adjacent digits.

     

     

     

     

     

     

     

     

     

     

     

     

  51. A 45-year-old male smoker presents with the clinical appearance shown in Figure A. Which of the following statements is true regarding his condition?

     

     

     

     

    1. Treatment with aspirin has been shown to decrease the incidence of amputation

    2. Arteriography is useful in the diagnosis of his condition

    3. Prophylactic amputation of unaffected digits leads to improved patient outcomes

    4. The condition is usually painless

    5. Involves proximal vessels first and distal vessels last

    CORRECT ANSWER: 2

    The clinical description and photograph are most consistent with a diagnosis of Buerger's disease, or thromboangiitis obliterans. Arteriography is the best method for the diagnosis of this condition.

     

    Buerger's disease is an inflammatory occlusive disorder of small and medium-sized vessels of the digits most frequently occurring in male smokers. The condition may mimic other autoimmune and vascular diseases. If the diagnosis is uncertain, an arteriogram is the study of choice. The only treatment that has been found to reduce the risk of amputation is the cessation of smoking.

    Phillips et al. review the vascular conditions of the upper extremity. They discuss that patients with Buerger's disease present with rest pain, claudication, and ulceration, and that cessation of smoking decreases disease progression and the incidence of amputation.

     

    Figure A shows the classic clinical appearance of a patient with Buerger's disease. Illustration A shows an arteriogram with "corkscrew" arteries that result from vascular destruction. In the appropriate clinical setting, this finding is diagnostic of Buerger's disease.

     

    Incorrect Answers:

    Answer 1: Treatment has not been shown to halt the progression of Buerger's disease.

    Answer 3: Prophylactic amputation of uninvolved digits has not been described for the treatment of Buerger's disease.

    Answer 4: Buerger's disease is frequently painful.

    Answer 5: Buerger's disease involves distal vessels first and proximal vessels last.

     

     

     

     

     

     

  52. A 42-year-old male sustains the injury seen in Figure A. Which of the following is true regarding management of this injury?

     

     

     

     

    1. Replantation at this level has a worse functional outcome when compared to above the elbow amputations

    2. The veins should be reconstructed prior to the arteries

    3. Bony stabilization should occur at the end of the procedure

    4. Vein grafting should not be used at this level of amputation

    5. A warm ischemia time of 8 hours is a relative contraindication to replantation

    CorreCt answer: 5

     

    Wrist-proximal amputations should be performed before 12 hours of cold ischemia time or 6 hours of warm ischemia time have elapsed.

     

    Wrist-proximal replantation should be strongly considered for patients in whom the mechanism allows adequate debridement, the cold ischemia time is less than 12 hours, and whose general health and comorbidities allow the patient to tolerate an extended surgical procedure. In general, amputation at the distal forearm and wrist have excellent functional results with replantation when compared to amputations at other levels. Similar to other major amputations, replantation should proceed in the following sequence: 1) bone,

    2) extensor tendons, 3) flexor tendons, 4) arteries, 5) nerves, 6) veins (can be done prior to nerve repair) , 7) skin.

     

    Sabapathy et al. review replantation surgery in the upper extremity. They discuss that a “functional extremity” could be reconstructed at the upper-arm level in 22% to 34%, at the proximal forearm level in 30% to 41%, and at the distal forearm level in 56% to 80% of cases.

     

    Hanel et al. review wrist level and proximal amputations in the upper extremity. Among other things, they state that wrist-proximal amputations should be performed before 12 hours of cold ischemia time or 6 hours of warm ischemia time have elapsed.

    Figure A shows a sharp transcarpal amputation. Incorrect Answers:

    Answer 1: Replantation at the distal forearm and wrist have better functional

    outcomes than above the elbow replantations.

    Answer 2: Arteries should be reconstructed prior to veins.

    Answer 3: Bony stabilization should be performed at the beginning of the procedure.

    Answer 4: Vein grafting should be used generously.

     

     

     

  53. A 45-year-old carpenter sustained a table saw injury to his right hand while at work earlier today. Evaluation in the Emergency Department reveals the defect depicted in Figure A. An island volar advancement flap was selected for wound closure. What is the largest defect that could be covered with this technique?

     

     

     

     

    1. less than 1 cm

    2. 1-1.5 cm

    3. 1.5-2 cm

    4. 2-2.5 cm

    5. 2.5-3.5 cm

     

    CorreCt answer: 5

     

    The clinical vignette is consistent with an oblique amputation of the distal phalanx of a thumb with a defect measuring >2.5 cm. Island volar advancement flaps are a safe and effective procedure for single-stage closure of considerably large thumb defects measuring up to 3.5 cm in length.

     

    The operative technique chosen for reconstruction of distal volar thumb defects depends largely on the size of the defect. Island volar advancement flaps used for defects up to 3.5 cm are pure island flaps in which all of the proximal attachments, with the exception of the neurovascular bundles, are divided to provide maximal advancement. Mobility up to 4 cm can be achieved with elevation of the entire volar skin of the thumb from the underlying tendon sheath providing a considerable advantage in thumb reconstruction. The island volar advancement flap is useful for coverage of the entire distal phalanx from the IP joint crease to the nail bed.

     

    Foucher et al. reviewed long-term clinical results of 13 neurovascular palmar advancement flaps for thumb tip coverage. Specifically, they reported on Moberg and O’Brien flaps. The Moberg flap is a pedicled advancement flap

    proximally-based on an intact skin pedicle of the thumb including both neurovascular bundles. The O’Brien flap is a modification of the Moberg technique which advances a volar flap based on a subcutaneous pedicle including both neurovascular bundles by incising the proximal skin and skin grafting the donor site. The study found that both flaps preserved near-normal pulp sensibility, MP and IP joint motion, and grasp and pinch strength. They suggested that Moberg and O’Brien flaps remain the first choice for coverage of 1-2 cm thumb pulp defects.

     

    Baumeister et al. reported on the functional outcomes of 25 patients that underwent thumb pulp reconstructions utilizing Moberg volar advancement flaps. They found that 72% of patients had no or only minor subjective complaints, 74% had normal sensitivity, DASH scores showed only minor impairments, no flaps resulted in decreased grip strength, and only minor restrictions were identified in active IP joint motion. All defects with a length less than or equal to 2 cm were successfully reconstructed, whereas, patients presenting with defects >2 cm developed complications.

     

    Mutaf et al. reviewed outcomes of 12 patients that underwent thumb reconstruction utilizing an island volar advancement flap for traumatic distal thumb injuries measuring 3 to 3.5 cm in length. Their results showed that none of the flaps failed, no patients had limited mobility or scar contractures, near-normal sensation was achieved, excellent recovery of pinch strength occurred, and maximal preservation of thumb length was possible in all patients.

     

    Figure A and Illustrations A through C represent a case example presented by Mutaf et al. Figure A depicts an oblique amputation of the distal phalanx of a right thumb. Illustration A reveals elevation of an island volar advancement flap on both sides of the digital neurovascular bundles in the same thumb.

    Illustration B reveals flap advancement and Illustration C reveals a postoperative image of the same thumb 4 months after surgery.

     

    Incorrect Answers:

    Answers 1 & 2: Small or superficial defects may be amenable to conservative treatment or local flaps depending on the location of the defect.

    Answers 3 & 4: The Moberg flap with modifications to lengthen distal advancement as necessary is considered a standard option for medium-sized defects of the thumb pulp less than or equal to 2 cm.

     

     

     

     

     

  54. A 58-year-old man presents with right middle finger swelling. Radiographs and an MRI of his hand are shown in Figure A and B. Tissue biopsy and staging investigations are performed. The tumor is then resected en bloc with the middle metacarpal, which is amputated

    1.5cm from the carpometacarpal joint. The attached deep transverse intermetacarpal ligaments are sacrificed. To prevent scissoring of the remaining digits and small objects falling through the gap between index and ring fingers, which of the following procedures should be performed?

     

     

     

     

     

     

    1. iliac crest bone grafting

    2. ring metacarpal transposition

    3. second toe transfer

    4. index metacarpal transposition

    5. suture of deep transverse intermetacarpal ligaments

    CORRECT ANSWER: 4

    Index metacarpal transposition is indicated to reduce the space left between the index and ring finger. In this case, the middle ray is amputated because of malignancy.

     

    With amputation of the middle or ring metacarpals, small objects fall through the gap and the adjacent fingers scissor. For single central ray defects, techniques to reduce the gap include transposition of the index finger (for middle ray amputation), small finger (for ring ray amputation), complete removal of the metacarpal (without leaving a proximal metacarpal base stump) to allow the bases of index and ring metacarpals to migrate together and reconstruction of the deep transverse metacarpal ligament. The technique of index transposition may vary depending on the osteotomy (straight vs step-cut) and fixation (K wires vs plate) as seen in the illustrations below.

     

    Muramatsu et al. describe bony transposition for reconstruction after ray amputation for malignancy. The advantage is immediate closure of the space. The disadvantages include prolonged postoperative immobilization until union, malrotation (leading to scissoring), mal-tension of tendon (because of different metacarpal heights), and delayed or nonunion.

     

    Lyall et al. advocate total middle ray amputation. They believe that leaving the metacarpal base behind leads to difficulty in aligning the adjacent rays as the index and ring must angulate over the bony obstruction to close the distal gap, leading to scissoring. They believe that index transposition leaves an abnormally wide 1st web space and a remnant 2nd metacarpal stump that can protrude dorsally.

     

    Figure A is an AP radiograph of the right hand showing a destructive lesion of the proximal phalanx of the middle finger abutting the metacarpophalageal joint. Figure B is a STIR coronal MRI image showing the tumor mass extending into surround soft tissue. Illustration A is a diagram showing index transposition for middle ray amputation using a straight osteotomy and crossed K-wires. Illustration B is a diagram showing index transposition using a step-cut osteotomy and multiple K-wire fixation to the adjacent metacarpals. Illustration C is a diagram showing index transposition using a straight osteotomy and plate fixation. Illustration D is a diagram showing an alternative technique of suturing deep transverse metacarpal ligaments together to close the gap.

    Incorrect Answers

    Answer 1: Iliac crest bone grafting is not indicated. Together with a skin flap, iliac crest bone grafting may be used for thumb reconstruction.

    Answer 2: Ring metacarpal transposition is not indicated. Transposing a metacarpal from ring-to-long position leads to a new gap where the ring digit used to be. For ring metacarpal defects, the adjacent small metacarpal is transposed instead.

    Answer 3: Second toe transfer may be indicated for thumb reconstruction, or digital reconstruction for multiple digit losses. It involves microsurgical repair. For solitary central ray loss, a lesser procedure is preferred.

    Answer 5: Gap closure by suturing deep transverse metacarpal ligaments together is only possible in the absence of malignancy, where the ligaments are not compromised. In this case, the ligaments were resected together with the tumor.

     

     

     

     

     

     

     

     

     

     

  55. A 65-year-old man fell and injured his right wrist. Radiographs taken in the emergency room are seen in Figure A. He was treated as a sprain and no further follow-up was planned. He sustained 2 minor falls over the next 6 years and his wrist pain recurred. Recent radiographs are seen in Figure B. Surgical treatment that will best address his symptoms and preserve wrist motion consists of

     

     

     

     

    1. Anterior and posterior interosseous neurectomy

    2. Scaphotrapezialtrapezoidal (STT) fusion

    3. Complete wrist arthrodesis

    4. Proximal row carpectomy

    5. Four-corner fusion with scaphoidectomy

    CORRECT ANSWER: 5

    Four-corner fusion with scaphoidectomy is indicated for Stage III SLAC wrist.

     

    Surgical treatment of SLAC wrist is stage dependent. Stage I disease (scaphoid-radial styloid arthritis) is treated with AIN/PIN neurectomy. This procedure can also be done in addition to other bony procedures for Stages II-III disease. Stage II (scaphoid-entire scaphoid facet) is treated with PRC or scaphoid excision with 4-corner fusion (4CF). Stage III (capitolunate arthritis with proximal migration of the capitate into the scapholunate interval) is treated with either scaphoidectomy with 4CF or total wrist fusion.

     

    Some other conditions exist: If capitolunate arthritis exists, PRC is contraindicated and 4CF is performed. If radiolunate arthritis exists, both PRC and 4CF are contraindicated and total wrist fusion is performed. If both radiolunate and capitolunate surfaces are preserved, then either PRC or a 4CF may be performed.

     

    Cohen et al. compare PRC with 4-corner fusion plus scaphoid excision. PRC is technically easier, but leads to shortening of the carpus with weakness and incongruity exists between the capitate and lunate fossa of the distal radius. Scaphoid excision and four-corner fusion maintains carpal height and preserves the radiolunate relationship, but is more technically demanding, there is risk of nonunion, and it requires longer postop immobilization. Pain relief is more reliable following 4-corner fusion.

     

    Figure A shows scapholunate ligament disruption. Figure B shows late stage SLAC wrist. There is capitolunate arthritis but no radiolunate arthritis.

    Illustration A shows an example of PRC. Illustration B shows an example of 4CF and scaphoidectomy.

     

    Incorrect Answers

    Answer 1. Neurectomy of AIN and PIN is performed for Stage I disease and can also be done in addition to other bony procedures for Stages II-III. Answer 2. STT fusion is indicated for chronic scapholunate instability, STT arthritis and Kienbock's disease. It is not appropriate for Stage III SLAC wrist as it does not address capitolunate arthritis.

    Answer 3. Complete wrist arthrodesis is indicated for pancarpal arthritis in a young patient. It is less appropriate for this 65-year-old patient. It sacrifices wrist motion. Wrist arthrodesis would be performed if BOTH capitolunate and radiolunate arthritis were present

    Answer 4. Proximal row carpectomy is indicated for Stage II disease. It is contraindicated where capitolunate arthritis is present (Stage III).

     

     

     

     

     

     

     

     

     

  56. Which of the following statements is true regarding zone II flexor tendon injuries?

    1. At this level, FDS and FDP are located within separate tendon sheaths

    2. FDS repair has not been shown to improve outcomes

    3. Improved gliding is seen with repair of 1 slip of FDS compared to repairing both slips

    4. Repairing FDS does not affect post-operative digit strength

    5. FDP repair has not been shown to improve outcomes

    CORRECT ANSWER: 3

    In zone II flexor tendon injuries, repairing only one slip of FDS has been shown to improve gliding when compared to repair of both slips.

     

    Zone II flexor tendon injuries have notoriously had poor outcomes secondary to high rates of adhesion formation at the pulleys. However, new advances in post-operative rehabilitation have significantly improved outcomes to the point where it is no longer considered "no man's land." Management of the FDS has been a source of controversy. In the past, the FDS was occasionally excised to theoretically make more room for the FDP. This has now been largely abandoned and the FDS is repaired whenever possible. Whether or not to repair both slips of FDS remains controversial, with in vitro data suggesting that gliding resistance is improved if only one slip is repaired.

     

    Zhao et al. review the effect of partial vs. complete FDS excision following repair of FDP for zone II flexor tendon injuries. Preserving the whole FDS resulted in a significantly larger increase in gliding resistance after FDP repair than did full or partial FDS removal, which were not significantly different from each other.

     

    Illustration A shows the zones of flexor tendon injury. Note that zone II injuries occur between the FDS insertion and the distal palmar crease. Illustration B shows the anatomy of the flexor tendons in detail. Video V shows a technique for repair of zone II injuries.

     

    Incorrect Answers:

    Answer 1: In zone II, the FDS and FDP are located within the same tendon sheath.

    Answer 2: While the FDS was excised in the past, clinical outcomes have recently been shown to be improved with repair of either one or both slips. Answer 4: Repairing FDS has been shown to increase digit strength.

    Answer 5: Repair of FDP has been shown to improve long-term clinical outcomes.

     

     

     

     

     

     

     

     

  57. A 6-year-old girl sustains transverse amputations through her long and ring fingertips after getting her hand caught in a lawn mower. She presents to the emergency room 30 minutes after the injury with the amputated tissue which was placed on ice in a waterproof bag. On physical exam the amputation levels are found to be 6 millimeters distal to the lunula. The wounds are noted to be fairly

    contaminated with no evidence of exposed bone. Skin defects are less than 1 centimeter. Which of the following is the most appropriate management at this time?

    1. Emergent replantation of the amputated parts

    2. Revision amputation through the distal interphalangeal joint

    3. Thorough irrigation and debridement followed by elective Moberg advancement flaps

    4. Thorough irrigation and debridement followed by elective Z-plasty reconstruction

    5. Thorough irrigation and debridement, soft dressing application, and followup within 1 week

    CorreCt answer: 5

     

    Distal fingertip amputations can be successfully managed with local wound care and healing by secondary intention if no bone is exposed and the soft tissue defects are minimal. This is especially true in the pediatric population.

     

    Distal fingertip amputations are common injuries seen in the emergency department. If bone is not exposed, the wounds can be successfully treated with local wound care and dressing changes, followed by soaks in a hydrogen-peroxide solution after 7-10 days. Some controversy exists in the pediatric population if the soft tissue loss is > 1 cm, with options for management including a V-Y advancement flap or conservative management with dressing changes.

     

    Quell et al. review the results of 82 patients with fingertip amputations treated conservatively; 31 of the digits were treated with primary closure with or without shortening of bone and 54 digits were treated with semiocclusive dressings. No complications were observed, and all healed fingertips were well padded and painless.

     

    Tupper et al. review sixteen patients with twenty fingertip injuries who underwent V-Y plasty for transverse fingertip amputations. Sensitivity was 73% of normal, with eight patients reporting hypersensitivity. Contrary to popular belief, they believe normal sensation following a V-Y plasty is not a reasonable expectation.

     

    Illustration A shows the three levels of fingertip amputations. Zone I is distal to the phalanx; Zone II is distal to the lunula; and Zone III is proximal to the lunula.

     

    Incorrect Answers:

    Answer 1: Emergent replantation is not indicated in distal fingertip amputations.

    Answer 2: Revision amputation through the DIP joint could be considered for Zone III injuries

    Answer 3: Moberg advancement flaps are considered for volar thumb soft tissue loss.

    Answer 4: Z-plasty is considered for soft tissue loss in the webspaces.

     

     

     

     

     

     

  58. Which of following malformations is most commonly associated with Poland's syndrome?

     

     

     

     

     

     

     

     

     

     

     

    1. Figure A

    2. Figure B

    3. Figure C

    4. Figure D

    5. Figure E CORRECT ANSWER: 4

    Figure D demonstrates symbrachydactyly which is most commonly associated with Poland's syndrome.

     

    Poland's syndrome is a rare birth defect characterized by underdevelopment or absence of the chest muscle in conjunction with ipsilateral symbrachydactyly. Poland syndrome most often affects the right side of the body, and occurs more often in males than in females.

    Ireland et al. reviewed 43 consecutive cases of Poland's syndrome, and reviewed the relevant literature up to that point. The authors state that the clinical features are variable but always include congenital aplasia and syndactyly, and the right side is affected more than the left. They also note that although the hand remains hypoplastic and functional capacity is limited by the inherent skeletal anomalies, surgical treatment improves functional capacity and cosmetic appearance in the majority of patients.

     

    Van Heest summarizes normal formation and growth of the upper limb as a basis for understanding malformation, with the goal of providing a basic understanding of the evaluation necessary for appropriate counseling and referrals for treatment of the child with hand and upper extremity congenital deformities.

     

    Incorrect Answers:

    Answer 1: Figure A demonstrates the classic deformity associated with Apert's syndrome, which commonly consists of complex syndactyly.

    Answer 2: Figure B demonstrates the lymphedema characteristic of amniotic band syndrome.

    Answer 3: Figure C demonstrates radial club hand which can be associated with VATER syndrome.

    Answer 5: Figure E shows and example of thumb polydactyly which is associated with a variety of syndromes, but not typically Poland's.

     

     

     

  59. A 55-year-old male laborer comes in with a chief complaint of clumsiness with his right hand for the past 3 months including difficulty using a hammer while at work. He has had no injury to the right upper extremity. On physical examination, he has persistent small finger abduction/extension with finger extension and active adduction. An EMG is performed and demonstrates ulnar nerve conduction velocities of 31 m/sec (normal >52m/sec). The patient symptoms are most accurately described as:

    1. Axonotmesis with ischemia origin

    2. Axonotmesis with myelin disruption

    3. Neurapraxia with ischemia origin

    4. Neurapraxia with endoneurium disruption

    5. Neurotmesis CORRECT ANSWER: 3

      The history and clinical presentation are consistent with ulnar entrapment neuropathy at the level of the cubital tunnel. This would be classified as a neuropraxia with ischemia origin.

       

      Compression injuries to the peripheral nerves are often the result of microvascular dysfunction as the nerves traverse a high to low pressure gradient. Peripheral nerve injury can be classified as neuropraxia, axonotmesis, and neurotmesis. Compressive neuropathies are typically neuropraxias, with local myelin damage but not compromise of the major components of the nerve. In axonotmesis, there is Wallerian degeneration and myelin loss distal to the site of injury. The most severe type is that of neurotmesis. Neurotmesis is composed of a spectrum of injury in which the endoneurium is always disrupted (perineurium or epineurium may be intact). The worst form of neurotmesis is that of nerve transection.

       

      Elhassan et al. review the pathophysiology of cubital tunnel syndrome. They report nerve dysfunction results from ischemic changes secondary to compression. Compressive effects on the nerves can last greater than 24 hours, even after the source of compression has been removed.

       

      Rempel et al. review the pathophysiology of peripheral nerve compression syndromes. The authors indicate that deforming pressures to nerves are often the result of stenotic soft tissue canal boundaries. This leads to interference with local microvasculature of the nerve itself.

       

      Illustration A demonstrates the Wartenberg sign, where the patient has persistent small finger abduction/extension resulting from weakness of the 3rd palmar interosseous/small finger lumbrical.

      Illustration B reveals clawing which results from overpowering of the intrinsic muscles by the extrinsic muscles; a tenodesis effect results in flexion of the PIP/DIP joints. This is more severe in ulnar nerve compression at Guyon’s canal. Illustration C shows the Froment sign, where the FPL attempts to compensate for a deficient pinch, because of weakness of the adductor pollicis. Illustration D demonstrates atrophy of the 1st dorsal webspace from chronic compressive changes. Illustration E demonstrates atrophy of the thenar compartment which is consistent with carpal tunnel syndrome.

       

      Incorrect Answers:

      Answer 1, 2: Axonotmesis is considered a second degree nerve injury, characterized by Wallerian degeneration of axons distal to site of injury. Compression neuropathies are more often neuropraxias (1st degree nerve injury)

      Answer 4: Compression neuropathies result from ischemic insult to the nerve

      Answer 5: Neurotmesis may be characterized by complete disruption of all components of nerve (as in transection) or with disruption of all components except for the perineurium or the endoneurium. This is not characteristic of a compression neuropathy such as cubital tunnel syndrome.

       

       

       

       

       

       

       

       

       

       

       

       

       

       

       

  60. Which of the following hand injuries seen in Figures A-E is most appropriately treated with a first dorsal metacarpal artery flap?

     

     

     

     

     

     

     

     

     

     

     

    1. Figure A

    2. Figure B

    3. Figure C

    4. Figure D

    5. Figure E CORRECT ANSWER: 3

    Figure C shows a dorsal thumb laceration with exposed tendon that would be most appropriately treated with a first dorsal metacarpal artery (FDMA) flap.

     

    The first dorsal metacarpal artery is a branch of the radial artery that supplies the dorsal hand skin from the thumb metacarpal to the long metacarpal, as well as the skin on the dorsal surfaces of the thumb and index to the proximal interphalangeal joint. The flap is raised distal to proximal as an island flap containing the FDMA, branches of the radial nerve, fascia of the underlying interosseous muscle of the first web space, and skin overlying the MP joint and proximal phalanx of the finger. It is an excellent option for large soft tissue defects on either side of the thumb. In this case, skin grafting is contraindicated because of exposed tendon without paratenon.

    Sherif et al. detail the anatomy of the first dorsal metacarpal artery. They found three consistent branches, including the radial, ulnar, and intermediate branch. In part II of their study, they review the results of 23 patients where the FDMA flap was used as a fasciocutaneous or fascial flap for the coverage of soft tissue hand defects.

    Illustration A shows a FDMA flap being raised for coverage of a thumb defect. Incorrect Answers:

    Answer 1: Fingertip amputations with minimal soft tissue loss and no exposed

    bone can be allowed to heal through secondary intention.

    Answer 2: The posterior interosseous fasciocutaneous flap is an excellent option for lacerations to the first web space.

    Answer 4: This large soft tissue defect on the dorsum of the hand may be treated with a groin flap.

    Answer 5: Fingertip amputations with exposed bone are best treated with local advancement flaps such as a VY advancement flap.

     

     

     

     

     

     

  61. Figure A is a radiograph of a 35-year-old women who sustained an isolated left wrist injury after a fall onto an outstretched hand. She has been complaining of left dorsal wrist pain since the fall. Examination reveals a positive Watson's scaphoid shift test. What ligamentous structure is an important secondary stabilizer to prevent dorsal intercalated segment instability (DISI) deformity in this patient?

     

     

     

    1. Transverse carpal ligament

    2. Dorsal intercarpal ligaments

    3. Triangular fibrocartilage complex

    4. Dorsal lunotriquetral ligament

    5. Volar lunotriquetral ligament

    CORRECT ANSWER: 2

    The integrity of the dorsal intercarpal ligaments is important in preventing dorsal intercalated segment instability (DISI) deformity and persistent scapholunate instability.

     

    Scapholunate instability is the most common carpal instability. The primary stabilizing structure of the scaphoid and lunate bones is the scapholunate ligament, which is commonly injured with a fall on an outstretched hand.

    Secondary stabilizers of the scaphoid and lunate include the dorsal intercarpal ligaments and the dorsal radiocarpal ligaments. Failure to recognize injury of these structures can cause persistent dorsal intercalated segment instability (DISI). This can predispose patients to a SLAC wrist and early wrist osteoarthritis.

     

    Mitsuyasu et al. examined the role of dorsal intercarpal ligaments (DIC) in scapholunate instability. They showed that the DIC had an important role in stabilizing the scaphoid and lunate bones with static and dynamic movements. The authors of this study suggest that the DIC ligament should be assessed intraoperatively and consideration should be given to repair and/or reconstruction with surgical management of scapholunate ligament tears.

     

    Viegas et al. showed that the dorsal intercarpal and the dorsal radiocarpal ligaments form a lateral V configuration over the dorsal wrist. This configuration acts as an indirect dorsal stabilizing effect on the scaphoid

    throughout the range of motion of the wrist. Their integrity acts to ensure normal wrist kinematics.

     

    Figure A shows an AP and lateral radiograph of the left hand. There is significant gapping between the scaphoid and lunate articulation. This is indicative of a complete scapholunate dissociation, however both wrists should be imaged as this deformity may exist without injury. Illustration A shows the anatomy of the dorsal intercarpal and the dorsal radiocarpal ligaments.

     

    Incorrect Answers:

    Answers 1, 3, 4, 5: Carpal ligamentous injuries can occur with falls onto an outstretched hand. However, the clinical and radiographic features of this patient do not show injury to these structures.

     

     

     

     

     

     

  62. A 50-year-old patient presents with stiffness in her hand. A clinical photo is shown in Figure A. During surgical exposure, the neurovascular bundle is identified and dissected. What is the clinically most important pathologic structure to identify and what is its location relative to the neurovascular bundle in the digit?

     

     

     

    1. Spiral cord which is central and superficial to the neurovascular bundle

    2. Central cord which is midline and superficial to the neurovascular bundle

    3. Retrovascular cord which is central and superficial to the neurovascular bundle

    4. Spiral cord which is lateral and deep to the neurovascular bundle

    5. Central cord which is lateral and deep to the neurovascular bundle

    CORRECT ANSWER: 4

    Based on clinical findings, the patient has evidence of Dupuytren’s contracture affecting her ring finger. Relative to the neurovascular bundle, the spiral cord will lie lateral and deep.

     

    Dupuytren’s disease is a benign hand condition characterized by pathologic nodules and cords of existing fascial bands. The most clinically relevant structure in Dupuytren's disease, is the spiral cord. The spiral cord is the result of pathology of 4 structures: the middle layer of the pretendinous band, the spiral band, the lateral digital sheet, and Grayson's ligament. The spiral cord is found predominantly at the palmodigital transition. The spiral cord displaces the neurovascular bundle centrally and superficially.

     

    Benson et al. review the etiology, pathophysiology and treatment options for Dupuytren’s contracture. They highlight that while the pretendinous band is located volar and central to the neurovascular bundle in the palm, the spiral band and lateral digital sheath cause the neurovascular bundle to be displaced superficially and volarly as they become pathologically affected.

     

    Black et al. review the pathoanatomy, diagnosis and management of Dupuytren's disease. They note that the spiral cord lies superficial to the neurovascular bundle proximal to the MCP joint. Distal to the MCP joint it passes deep to the bundle. At that location, the spiral cord lies lateral to the

    neurovascular bundle as the lateral digital sheet becomes involved

     

    Figure A demonstrates the cord formation that is characteristic of the pathologic Dupuytren’s condition. It is the central cord that causes contracture of the MCP, whereas the retrovascular and spiral cords cause contractures of the DIP and PIP respectively. Illustration A shows the relationship of spiral cord formation in Dupuytren's disease relative to the normal anatomy of the palmar fascia. The structures implicated in the formation of the spiral cord are the pretendinous band, the spiral band, the lateral digital sheet, and Grayson's ligament. Cleland's ligament, more dorsally located, is spared in Dupuytren's disease. The neurovascular bundle is displaced superficially and towards the midline, as the pathological cord spirals around. Illustration B shows the presence of other affected structures, including the natatory ligament and the central band. The central band is an extension of the pretendinous cord and attaches to the base of the middle phalanx. It may insert onto the tendon sheath of the flexor tendon at this level. Formation of natatory cords cause webspace contractures. Formation of central cords lead to flexion contractures of the PIP. Illustration V is a video that provides an educational overview of Dupuytren's.

     

    Incorrect Answers:

    Answer 1: The spiral cord is the most important clinically, but displaces the neurovascular bundle centrally/superficially.

    Answers 2, 5: The central cord is a structure found in the midline. It is an extension of the pretendinous cord and attaches to the base of the middle phalanx. It leads to a PIP flexion contracture, but does not affect the neurovascular bundle.

    Answer 3: The retrovascular cord forms from digital fascia found dorsal to the neurovascular bundle. This may may lead to displacement of the neurovascular bundle centrally and superficially. In conjunction with the lateral cord, it may lead to the formation of hyperextension contractures of the DIP

     

     

     

     

     

     

     

     

     

  63. An infant is brought to your office for evaluation of his hands. Clinical photos are shown in Figures A and B. The clinical features are most consistent with a genetic mutation in which of the following:

     

     

     

    1. Sonic Hedgehog (SHH)

    2. FGFR2

    3. FGFR3

    4. PMP22

    5. COL1A1 CORRECT ANSWER: 2

    Based on the clinical features seen in the figures provided, the most likely syndrome is that of Apert syndrome, which is consistent with a mutation in FGFR2.

     

    Apert syndrome is an autosomal dominant condition that gives rise to facial dysmorphism and complex syndactyly of the hands. The craniosynostosis that develops causes flattening of the skull and facial features.

     

    Goldberg et al review congenital hand conditions and the malformations associated with them. They indicate that not only does identification allow for natural history to be better elucidated, but also timing of surgical intervention can be better gauged.

     

    Figures A and B demonstrate clinical features consistent with Apert Syndrome. The “rosebud” hand is a complex syndactyly that affects the index, middle and ring fingers most commonly. Hypertelorism is exemplified with increased distance between the eyes; additionally, acrocephaly is noted with forehead broadening and skull flattening.

     

    Incorrect Answers

     

    1: Mutation in sonic hedgehog gene (SHH) is associated with a longitudinal deficiency of the radius. This is seen in conditions like TAR, Holt-Oram and VACTERL syndromes.

    3: Mutation in FGFR3 leads to achondroplasia

    4: Mutation in PMP22 gives rise to Charcot Marie Tooth syndrome 5: Mutation in COL1A leads to osteogenesis imperfecta

     

     

     

  64. A 45-year-old patient presents with recurrence of radial sided wrist pain after undergoing a first dorsal compartment release about 3 months ago. The surgery was completed by one of your partners; operative reports indicate that the sheath was incised on the dorsal edge. On physical exam she is found to have normal appearing skin, a negative Tinel’s sign, and a positive Finklestein test. What is the most likely cause of the recurrence of her symptoms?

    1. Development of neuroma

    2. Complex regional pain syndrome

    3. Failure to decompress the EPB sub-sheath

    4. Failure to decompress the EPL sub-sheath

    5. Failure to decompress the APB sub-sheath

    CORRECT ANSWER: 3

    Based on the history and clinical findings this patient has de Quervain’s tenosynovitis. The recurrence of her symptoms can be attributed to a failure to recognize and decompress the EPB sub-sheath.

     

    De Quervain’s tenosynovitis is a stenosing inflammatory condition of the first dorsal compartment of the wrist (APL/EPB). Surgical release of the compartment is indicated after conservative measures have failed. At the time of the operation, the incision is made on the dorsal side of the sheath to prevent volar subluxation of the tendons. Failure to identify and release a distinct EPB sub-sheath or a separate fibro-osseous compartment of the APL can lead to a recurrence of symptoms.

     

    Alegado et al. report a case of a patient with dysesthesias in the superficial radial nerve distribution 3 months after undergoing first dorsal compartment release for de Quervain’s tenosynovitis. They found a persistent fibrous remnant of the dorsal aspect of the sheath causing elevation of the superficial radial nerve. They recommend sheath excision or incision of the sheath at its dorsal attachment to avoid this complication.

     

    Ashurst et al. report a case of a patient presenting with bilateral de Quervain’s tenosynovitis secondary to excessive text messaging. Conservative measures

    afforded the patient complete symptomatic recovery. They recommend limitation of texting, in conjunction with other standard treatments, to treat text messaging- associated de Quervain’s tenosynovitis

     

    Ilyas et al. review the etiology, diagnosis and management of De Quervain’s tenosynovitis. Non-surgical management is largely successful and includes splinting and cortisone injections. In refractory cases, surgical release of the first dorsal compartment is completed. They recommend meticulous care of the radial sensory nerve and identification of all separate sub-sheaths.

     

    Illustration A shows an operative photo in a patient with multiple APL slips and an EPB that is hidden within a sub-sheath. Video V gives a brief overview of de Quervain’s tenosynovitis.

     

    Incorrect Answers

    Answer 1: Given the negative Tinel’s sign on physical exam, the patient is less likely to have a neuroma.

    Answer 2: Her history, symptoms and lack of skin changes are not consistent with complex regional pain syndrome.

    Answers 4, 5: The first dorsal compartment is composed of the APL/EPB. The EPL is in the third dorsal compartment and the APB is in the thenar compartment.

     

     

     

     

     

  65. A 28-year-old NFL running back complains of continued hand pain three days following an injury sustained while being tackled. He was splinted on the field. He has tenderness over the long finger metacarpal head, with subluxation of the extensor tendon into the intermetacarpal area during active metacarpophalangeal joint flexion. A representative MRI is shown in Figure A. What is the next best step in management of this patient?

     

     

     

     

    1. Observation alone

    2. Continued splinting in flexion

    3. Continued splinting in extension

    4. Open repair of the disrupted junctura tendinae

    5. Open repair of the disrupted sagittal band

    CORRECT ANSWER: 5

    Based on the history and physical exam findings this patient has sustained a traumatic rupture of the sagittal band. In this professional athlete, the next best step would be to perform an open repair of the sagittal band. This will allow for earlier aggressive rehabilitation and a quicker return to sport.

     

    Sagittal band ruptures may be traumatic (as in this case) or attritional in nature (as in rheumatoid arthritis). A direct blow to the MCP leads to forced flexion of the digit and subsequent stretching/rupture of the affected structure. On physical exam the tendons are most unstable with the wrist flexed; MCP flexion will lead to dislocation of the tendon into the intermetacarpal gutter.

    Acute injuries may be treated with extension bracing for 4-6 weeks, but in professional athletes, direct open repair of the sagittal band is indicated.

     

    Catalano et al. review sagittal band injuries treated with a thermally molded

    plastic splint that held the MCP in ~25-35 degrees of hyperextension. Patients were evaluated over 14 months; out of 11 sagittal band injuries, splinting was successful in eight of them. They recommend initial nonsurgical management with custom splinting.

     

    Hame et al. review the results of the management of sagittal band injuries in the professional athlete. The lesion commonly found was the disruption of the extensor mechanism with predictable sagittal band tears. In their series, all patients regained full range of motion and returned to their respective sports. They recommend surgical intervention in elite athletes in the form of extensor tendon centralization and sagittal band repair.

     

    Figure A shows a T1 weighted axial cut of the affected hand; subluxation of the tendon (arrow) can be identified with disruption of the sagittal band (arrowhead).

    The video provided briefly reviews injury to the sagittal band. Incorrect Answers

    Answer 1: Observation is not indicated in this patient

    Answer 2, 3: Splinting in extension would be an acceptable option in the non-athlete, but direct repair is indicated in a professional athlete

    Answer 4: The junctura tendinae are not injured in this patient

     

     

     

     

  66. Compressive injury to the posterior interosseous nerve will lead to EMG fibrillations in which of the following muscles?

    1. Extensor Carpi Radialis Longus/Extensor Carpi Radialis Brevis/Brachoradialis

    2. Extensor Carpi Radialis Longus/Supinator/Abductor Pollicis Longus

    3. Extensor Pollicis Longus/Supinator/Abductor Pollicis Longus

    4. Brachoradialis/Supinator/Extensor Pollicis Longus

    5. Extensor Pollicis Longus/Supinator/Abductor Pollicis Brevis

    CORRECT ANSWER: 3

    Based on the choices above, fibrillations will be seen in the extensor pollicis longus, supinator and abductor pollicis longus muscles.

     

    The radial nerve splits into the superficial radial branch and the posterior interosseous nerve (PIN) at the anterior aspect of the radiocapitellar joint, just proximal to the supinator muscle. The PIN innervates the EDC, EDM, ECU,

    EPB, EPL, EIP, APL and sometimes the ECRB. Compressive neuropathy of the PIN leads to motor dysfunction, namely weakness with wrist and finger extension.

     

    Lubhan et al. review uncommon compression neuropathies affecting the upper extremity. They indicate that PIN syndrome may be caused by rheumatoid arthritis and compressive ganglion cysts. Depending on which nerve branch is affected, partial lesions may develop. They recommend use of conservative measures (rest, activity modification and splinting) first. Decompressive procedures may be indicated in symptoms lasting greater than 3 months.

     

    Illustration A shows the course of posterior interosseous nerve from proximal to distal along the course of the supinator. This proximal edge of the supinator (Arcade of Froshe), the fibrous edge of the ECRB and the leash of Henry are three main points of compression of the PIN.

     

    Incorrect Answers

    Answer 1, 2, 4: The radial nerve proper innervates the ECRL, ECRB and Brachoradialis

    Answer 5: The recurrent motor branch of the median nerve innervates the APB

     

     

     

     

     

     

  67. Figure A shows a traumatic laceration of the distal forearm with a 5cm segmental median nerve defect. Which of the following repair or reconstruction techniques would allow for the best recovery of motor function?

     

     

     

    1. Autogenous venous nerve conduit

    2. Collegen synthetic nerve conduit

    3. Biodegradable polyglycolic acid

    4. Processed nerve allograft

    5. Nerve autograft CORRECT ANSWER: 5

    Figure A shows a traumatic laceration with 5cm of median nerve defect. The use of nerve autograft for this size defect has been shown to have the best recovery of motor function.

     

    The optimal surgical treatment of nerve laceration is direct tension-free repair. In segmental nerve defects this approach cannot be achieved. The use of interposed autologous nerve grafting remains the gold standard of repair in this setting. The use of alternative techniques, such as processed allografts and synthetic conduits, have not shown to have equivalent recovery of motor function as compared to nerve autograft.

     

    Giusti et al. used a rat model to examine techniques of peripheral nerve repair. They showed that nerve autograft resulted in better motor recovery than did the use of processed allograft or a collagen conduit.

     

    Deal et al. discussed tubular interposition substitutes, or nerve conduits, as an alternative to nerve autograft in segmental nerve defect. Nerve conduits can include autogenous nerve conduits (venous or arterial) and synthetic nerve conduits (collagen, PGA, or caprolactone). In general, there is an upper limit of 3-cm when using nerve conduit.

     

    Figure A is an image of the volar forearm. There is a traumatic laceration to

    the anterior compartment tendons as well as the median nerve.

     

    Incorrect Answers:

    Answer 1: Autogenous venous nerve conduit has been shown to effective in digital nerve defects. However, the results are not superior to nerve auto grafting and have not been shown to be effective in gaps >5-cm.

    Answers 2,3: Synthetic nerve conduit (eg. collagen, biodegradable polyglycolic acid) have not been shown to be superior to nerve auto grafting. Synthetic nerve conduit may be useful in digital nerves with gaps <3-cm.

    Answer 4.: Processed allograft has been shown to be inferior to nerve autograft in terms of motor recovery.

     

     

     

  68. A 45-year-old man presents with a three-month history of unilateral symptoms in his right wrist and hand. He first noticed a palpable nodule over the volar aspect of his wrist about three months ago. The nodule would become painful after weekends of heavy drinking at which time he noticed tingling sensation in his index and middle fingers. He notes that ibuprofen has helped improve the pain in the past. On clinical examination, he has a palpable, nontender, solid nodule over the volar aspect of his wrist. He has no motor or sensory deficits and negative carpal tunnel provocative tests. An axial CT and MRI image are provided in figures A and B. What would be the most appropriate next step in the management of his symptoms?

     

     

     

     

     

     

    1. Fine needle aspiration

    2. Chemotherapy

    3. Night splints

    4. Establish a tissue diagnosis and referral to a rheumatologist

    5. Surgical excision CORRECT ANSWER: 4

    The clinical presentation is consistent with carpal tunnel syndrome caused by an atypical space occupying lesion - in his case, gout. The most appropriate next step in the management of his symptoms would be establishing a tissue diagnosis and referral to a rheumatologist where medical therapy, such as prophylaxis with colchicine, could be initiated.

     

    Carpal tunnel syndrome is the most common compressive neuropathy, affecting up to 10% of the general population. Risk factors include female sex,

    advanced age, obesity, and repetitive motion activities. Typically, patients will develop symptoms of median nerve compression including thenar muscle atrophy, numbness in the radial 3.5 digits, night pain, and positive Tinel's and Phalen tests. First line management is non-operative, including NSAIDs, night splints, and activitiy modification. Carpal tunnel release surgery is indicated for those who have failed conservative management.

     

    Chen et al. described 23 unusual cases of CTS in which space-occupying lesions were responsible for the symptoms and signs of median nerve compression. In patients with an atypical presentation, such as male gender, non-middle-aged, or unilateral involvement, space-occupying lesions such as gout, synovial sarcoma, lipoma, and ganglions should be investigated as a cause.

     

    Fitzgerald et al. discussed gout affecting the hand and wrist. The medical treatment of gout includes NSAIDs such as indomethacin or ibuprofen for acute flares, and colchicine and allopurinol for chronic prophylaxis.

     

    Figures A and B represent axial CT and MRI images showing calcification and gouty tophi deposition in the carpal tunnel floor.

     

    Incorrect Answers:

    Answer 1: Aspiration is not a first line treatment for tophaceous gout. Answer 2: There is no role for chemotherapy in the treatment of tophaceous gout.

    Answer 3: Night splints would not help diminish the space occupying lesion, in this case, tophaceous gout.

    Answer 5: Chronic tophaceous gout that has failed medical therapy may require surgical excision.

     

     

     

  69. Which statement most accurately describes the physiology of peripheral nerve regeneration following an axonotmesic lesion?

    1. The proximal nerve segment undergoes Wallerian degeneration

    2. Axon growth occurs from the distal segment to proximal segment

    3. Neurotrophic factors direct phagocytic activity

    4. Proximal axon budding allows for antegrade (or distal) axon migration

    5. Axoplasm and myelin are degraded distally predominantly by Schwann cells for the first 12 months following injury

    CorreCt answer: 4

    Axonomesis is a disruption of the nerve axon following injury. Repair/regeneration of the nerve occurs via proximal budding, followed by antegrade (or distal) axon migration.

     

    The peripheral nerve regeneration process begins with the distal segment undergoing Wallerian degeneration (axoplasm and myelin are degraded distally by phagocytes). Existing Schwann cells proliferate and line-up along the basement membrane. Proximal budding occurs after a one-month delay. This is followed by sprouting axons that migrate in an antegrade fashion to connect to the distal tube. Repair of the nerve can take months, and often have poor outcomes.

     

    Lee et al. reviewed peripheral never injury and repair. They commented that Wallerian degeneration (i.e., breakdown of the axon distal to the site of injury) is initiated 48 to 96 hours after transection. The Schwann cells then align themselves longitudinally, creating columns of cells called Büngner bands. At the tip of the regenerating axon is the growth cone.

     

    Illustration A shows a chart of peripheral nerve injury. The two main classification systems are Seddon and Sunderland. Video V is a lecture discussing peripheral nerve injury and management.

     

    Incorrect Answers:

    Answer 1: The distal nerve segment undergoes Wallerian degeneration. Answer 2: Axon growth occurs from the proximal to distal segment.

    Answer 3: Neurotrophic factors do not direct phagocytic activity. Answer 5: Schwann cells do not degrade axoplasm and myelin.

     

     

     

     

     

     

  70. A 28-year-old male injures his hand while playing basketball and presents to the emergency room. Closed reduction is performed and is stable. Post-reduction rehabilitation is discussed with the patient.

    Which of the following radiographs demonstrates an injury that would be treated best by dorsal extension block splinting?

     

     

     

     

     

     

     

     

     

     

     

     

    1. Figure A

    2. Figure B

    3. Figure C

    4. Figure D

    5. Figure E CORRECT ANSWER: 2

    Dorsal extension-block splinting is the treatment of choice for dorsal proximal interphalangeal joint (PIPJ) fracture dislocations that are stable following reduction and have less than 40% articular surface fracture involvement.

     

    Dorsal PIPJ dislocations are a common injury, often resulting from jamming or hyperextending the finger. In the absence of an associated fracture or presence of a small volar plate avulsion, dorsal PIPJ dislocations are often

    treated with closed reduction and buddy-taping to the adjacent digit. Injuries that are unstable following reduction or those associated with an intra-articular fracture of the middle phalanx are stabilized with a dorsal extension-block splint to maintain reduction. It is important to initiate early range of motion exercises within the constraints of the splint to minimize scar formation and subsequent PIPJ contracture.

     

    Elfar et al. reviewed fracture-dislocations of the PIPJ. Dorsal PIPJ fracture-dislocations can be categorized as avulsion or impaction shear injuries.

    Avulsion fractures result from hyperextension of the PIPJ, tensioning the volar plate (VP) with eventual VP rupture or avulsion of the volar lip of the middle phalanx. Axial load applied to the digit in PIPJ flexion drives the head of the proximal phalanx across the middle phalangeal base, resulting in a shear fracture or comminuted impaction fracture of the middle phalanx, depending on the amount of energy imparted and the bone quality.

     

    Morgan et al. reviewed hand injuries in athletes. Dorsal PIPJ dislocations without associated fracture that are stable following successful reduction are treated by buddy taping the injured digit to the non-injured digit adjacent to the compromised collateral ligament. Buddy taping with active motion should be continued for 6 weeks. Unstable injuries and those with an intra-articular fracture of the middle phalanx should be treated with dorsal extension-block splinting with incremental extension of the splint on a weekly basis for 4 weeks, followed by buddy-taping for 3 months during sports activities.

     

    Figure A shows a simple dorsal PIPJ dislocation. Figure B shows a dorsal PIPJ fracture dislocation. Figure C shows a simple volar PIPJ dislocation. Figure D shows a volar PIPJ fracture dislocation. Figure E shows a dorsal avulsion fracture at the base of the distal phalanx (bony mallet injury). Illustration A depicts an dorsal extension-block splint that blocks extension of the digit past a set point while allowing full active flexion of the digit. Illustration B is a lateral radiograph of a digit showing a small minimally displaced volar plate avulsion fracture at the PIPJ with minimal intra-articular involvement (as compared to Figure B). This injury may be managed with buddy taping and active range of motion as tolerated.

     

    Incorrect Responses:

    Answer 1: A simple dorsal PIPJ dislocation that is stable following reduction should be treated with buddy-taping.

    Answers 3 + 4: A simple volar PIPJ dislocation and a volar PIPJ fracture dislocation that is stable following reduction should be treated with splinting in extension for 4-6 weeks.

    Answer 5: A bony mallet injury should be treated with splinting in full

    extension for 6-8 weeks to limit flexion of the digit and therefore fracture displacement.

     

     

     

     

     

     

     

     

     

  71. A 35-year-old mixed martial arts fighter and recreational cocaine user presents with symptoms concerning for hypothenar hammer syndrome (HHS). Significant ischemia is found on physical exam. Arteriography is shown in Figure A. What is the most appropriate next step in treatment?

     

     

     

    1. Conservative treatment with cocaine abstinence

    2. Conservative treatment with activity modifications and medical management with calcium channel blockers

    3. Therapeutic endovascular fibrinolysis

    4. Excision of involved segment and reconstruction with or without a vein graft

    5. Medical management with coumadin for 6 months

    CORRECT ANSWER: 4

    Figure A shows a bilobed aneurysm overlying the ulnar artery with normal appearing distal vasculature. Hypothenar hammer syndrome (HHS) can be associated with an aneurysm and is most appropriately treated with resection of the involved segment and either reconstruction with a primary anastomosis or vein graft.

     

    HHS syndrome consists of two separate entities, thrombosis and aneurysm. In the setting of thrombosis without aneurysm, conservative management is preferred. If the thrombosis is acute (<2 weeks), endovascular fibrinolysis has shown good results. In patients with an HHS and an aneurysm, surgery is required for resection to prevent distal embolization and remove the often painful aneurysmal mass.

     

    Yuen et al. review HHS. In patients with HHS and aneurysms, resection of the involved segment of the ulnar artery prevents distal embolic events, eliminates the painful mass, relieves ulnar nerve compression, and removes the thrombus which initiated the reflex vasospasm and closed off the collateral

    vessels in the region.

     

    Lifchez et al. review the long-term outcomes of 11 patients with HHS treated with ulnar artery reconstruction. 2 of the patients underwent excision and direct ulnar artery repair, and the rest underwent reconstruction with a vein graft. All patients had a mean improvement in digital brachial index, decrease in pain and dysesthesia symptoms, and decrease in cold intolerance compared with preoperatively.

     

    Nitecki et al. review a case series of 6 patients with HHS. They state that the treatment of thrombosis should be largely conservative, but thrombolytic treatment could be considered if the event happened <2 weeks prior to presentation.

     

    Illustration A shows an excised ulnar artery aneurysm in a patient with HHS. Note the typical "corkscrew" appearance of the distal segment.

     

    Incorrect Answers:

    Answer 1: HHS with aneurysm is best treated surgically. Answer 2: HHS with aneurysm is best treated surgically.

    Answer 3: Therapeutic endovascular fibrinolysis is useful for acute thrombosis without aneurysm.

    Answer 5: HHS with associated aneurysm is best treated surgically.

     

     

     

     

     

     

  72. A 36-year-male was using a high-pressure paint gun when he suffered the injury shown in Figure A. Which of the following variables would have the worst impact on his prognosis?

     

     

     

    1. Delay in surgical treatment

    2. Injected solvent was grease

    3. Injected solvent was water-based paint

    4. An entry wound of greater than 3 cm

    5. Injected solvent was at room temperature

    CORRECT ANSWER: 1

    The clinical presentation is consistent for a high-pressure injection injury. Delays in surgical treatment are associated with serious sequelae.

     

    High-pressure injection injuries are characterized by extensive soft tissue damage associated with a benign high-pressure entry wound. They should be treated with irrigation & debridement, foreign body removal and broad-spectrum antibiotics. There is a higher rates of amputation when surgery is delayed.

     

    Bekler et al. looked at the results of 14 surgically treated high-pressure injection injuries of the hand with a minimum of two years follow-up. Ten of the injuries required formal operative debridement and foreign body removal. Six required reconstructive microsurgical procedures and one underwent digital tip amputation. They concluded that high-pressure injection injury to the hand is a significant problem, which can easily lead to serious sequelae and, even, amputation.

     

    Rosenwaser et al. report wide débridement of all involved tissues, decompression of tissue compartments, exploration and incision of tendon sheaths, removal of injected material, and saline irrigation are critical in the management of high-pressure injection injuries to the hand. They emphasize

    delayed surgery has been associated with increased incidence of morbidity and amputation.

     

    Figure A shows a typical high-pressure injection injury. Notice the benign looking entry wound.

     

    Incorrect Answers:

    Answer 2: Grease as an injected solvent has a more favorable prognosis when compared to industrial solvents & oil based paints.

    Answer 3: Water-based paint as an injected solvent has a more favorable prognosis when compared to industrial solvents & oil based paints.

    Answer 4: The size of the entry wound does not have a strong correlation with the severity of injury. Often times these injuries have a benign looking entry wound.

    Answer 5: Injected solvent at high temperatures are associated with a worse prognosis.

     

     

     

  73. A healthy 50-year-old secretary is about to undergo an open carpal tunnel release. Which of the following peri-operative steps will have the greatest influence on minimizing the risk of a surgical site infection in this patient?

    1. Administration of cefazolin within 1 hour before incision

    2. Administration of cefazolin within 1 hour before incision followed by 5 days of cephalexin post-op

    3. Cleanse with bacitracin solution immediately before skin incision

    4. Standard sterilization and prepping

    5. Administration of one dose of cephalexin within 1 hour before incision

    CORRECT ANSWER: 4

    The patient is undergoing a clean, elective hand surgery. Prophylactic antibiotics, systemic or local, are not indicated for these procedures.

     

    Carpal tunnel syndrome is the most common compressive neuropathy. Individuals who fail medical management (night splints, NSAIDs, activity modification) are candidates for carpal tunnel release surgery (CTS). The surgery may be performed open or endoscopically. The reported incidence of post-operative infections following CTS varies between studies from 0% to 8%.

     

    Whittaker et al. performed a prospective, randomized, double-blinded, placebo

    controlled trial investigating the use of antibiotic prophylaxis in clean, incised hand injuries. They found no significant difference in infection rates between patients who received IV flucloxacillin, IV followed by oral flucloxacillin, and an oral placebo (13% vs. 4% vs. 15%, p=0.19). They did not support the use of routine antibiotic prophylaxis prior to clean hand surgery.

     

    Bykowski et al. retrospectively reviewed 8,850 outpatient elective hand surgeries and found no significant difference in the rate of surgical site infection, including patients with diabetes or history of smoking. They concluded that antibiotics should not be routinely administered prior to clean, elective hand surgeries.

     

    Harness et al. found no statistical difference in the incidence of surgical site infection following CTS without prophylactic antibiotic compared with patients who received prophylactic antibiotics (0.7% vs. 0.4%, p=0.354). They did not recommend routine antibiotic prophylaxis.

    Illustration A reviews the anatomic components of the carpal tunnel. Incorrect Answers:

    Answers 1, 2, 3, 5: Antibiotics do not decrease the risk of post-operative

    infection in clean, elective hand surgery. Surgeons should consider the potential risks of antibiotics prior to administration, including Clostridium difficile colitis, antibiotic allergies, bacterial resistance, and so on.

     

     

     

     

     

     

  74. A 30-year-old male laborer sustained a right wrist injury 9 months ago. He continues to have symptoms of recurrent ulnar-sided wrist pain that impairs his ability to work. An MRI is performed and

    shows a triangular fibrocartilage complex (TFCC) injury. Which of the following is an indication to combine a Wafer procedure with arthroscopic TFCC debridement?

    1. Ulnar styloid fracture

    2. Radial styloid fracture

    3. 2 mm of positive ulnar variance and ulnocarpal impingment

    4. 2 mm of negative ulnar variance and radiocarpal joint arthritis

    5. Scapholunate ligament injury

    CORRECT ANSWER: 3

    A Wafer procedure is indicated for positive ulnar variance and symptomatic ulnocarpal impingement associated with degenerative TFCC tears.

     

    Ulnar impaction syndrome and triangular fibrocartilage complex (TFCC) injuries are relatively common causes of ulnar-sided wrist pain. Positive ulnar variance causes increased contact pressures between the lunate and the ulnar head. The Wafer procedure removes 2-4 mm of distal ulnar head to reduce ulnar variance to neutral or negative. This is thought to reduce ulnar impaction and decrease pain.

     

    Faber et al. examined the role of MRI in wrist injuries. They showed that the sensitivity and specificity to detect TFCC tears using MRI is approximately 80%. They conclude that there is no supporting evidence for routine MRI's for patients with non-specific ulnar-sided wrist pain.

     

    Illustration A is a coronal view MRI (without arthrogram) of the right wrist that shows a TFCC tear (blue arrow) with positive ulnar variance. Illustration B shows a series of images showing a TFCC tear on MRI and intra-operatively.

     

    Incorrect Answers:

    Answers 1,2,4,5: Arthroscopic Wafer procedure is indicated for positive ulnar variance and ulnocarpal impingement associated with TFCC tears. This procedure uses the central TFCC tear to access the distal ulna with the arthroscope. The pathological findings supplied in the other answer options are not indications for this surgical procedure.

     

     

     

     

     

     

     

     

  75. A 27-year-old male sustains the injury shown in Figure A. He is taken to the operating room and the lesion is repaired primarily. Two months later, he feels a "pop" while using his hand and is no longer able to flex the distal phalanx of the involved digit. He is taken to the operating room for surgical exploration where 1.8 cm of scar tissue between the tendon ends is identified. The tendon sheath is found to be intact and allows smooth passage of a pediatric urethral catheter. What is the next step in management?

     

     

     

    1. Resection of scar and primary repair of tendon ends.

    2. Resection of scar and adjacent 1cm of tendon, placement of Hunter rod for staged reconstruction.

    3. Debulking of scar, partial excision of 25% of the A2 and A4 pulleys.

    4. Resection of scar, harvest of ipsilateral palmaris longus tendon for tendon reconstruction.

    5. Resection of scar and proximal tendon, tendon transfer from adjacent digit.

     

    CorreCt answer: 4

     

    This patient sustained an FDP laceration that was treated initially with primary repair. He subsequently re-ruptured the tendon 2 months later. With scar >1 cm, tendon grafting is indicated and primary tendon grafting with palmaris longus is commonly performed as it is the most accessible tendon in the operative field.

     

    Flexor tendon lacerations commonly result from volar lacerations. Concomitant neurovascular injury is common. Partial lacerations <60% of tendon width are treated with debridement and early range of motion. With partial lacerations, the least amount of gliding resistance can be obtained with debridement alone. Lacerations >60% of tendon width are treated with flexor tendon repair and controlled mobilization. Failed primary repair and chronic untreated injuries are indications for flexor tendon reconstruction and intensive postoperative rehabilitation.

     

    Lilly et al. reviewed complications after flexor tendon injuries. Common complications include adhesions, joint contracture, tendon rupture, triggering, pulley failure and bowstringing, quadrigia, swan-neck deformity and lumbrical plus deformity.

    Figure A shows a zone II laceration of the left index finger FDP.

     

    Incorrect Answers:

    Answer 1: Primary repair is not indicated. Advancement of the FDP >1cm leads to contractures and quadrigia effect.

    Answer 2: As the sheath is intact and not collapsed (as evidence by easy passage of a catheter), primary tendon grafting is preferred as it is a single-staged procedure. If the sheath is collapsed/scarred, two-stage reconstruction is performed.

    Answer 3: The scar is structurally weak and should be excised. Pulley excision or venting is used to decrease triggering and gliding resistance. It is not indicated in this instance.

    Answer 5: Tendon transfer from a nearby digit is not indicated if there is sufficient autograft tendon (e.g. palmaris longus) for reconstruction.

     

     

     

  76. A 55-year-old female patient presents with pain along the thumb ray and increasing deformity of her right hand. Key pinch causes her pain. The appearance of her hand is seen in Figure A. Range of motion of her thumb is seen in Figure B. What is the most likely cause of her deformity?

     

     

     

     

     

     

     

    1. Type II hypoplastic thumb

    2. Median nerve neuropathy

    3. Lupus thumb deformity

    4. Extensor tendon rupture

    5. Osteoarthritis of the trapeziometacarpal joint

    CORRECT ANSWER: 5

    The patient has 1st carpometacarpal (CMC) arthritis.

     

    With 1st CMC arthritis, the patient avoids painful thumb abduction and an adduction deformity gradually develops, with 1st webspace contracture. With progressive 1st CMC stiffness, the thumb metacarpophalangeal joint (MCP) develops hyperextension deformity to compensate for the loss of motion, leading to a secondary "Z" deformity.

     

    Rozental et al. reviewed hand and wrist reconstruction. They believe that arthrosis arises from loss of the anterior oblique ("beak") ligament.

    Compensatory MCP hyperextension should be treated with MCP capsulodesis or arthrodesis.

     

    Van Heest et al. reviewed thumb CMC arthritis. Treatment for Eaton stage I/II arthritis is open/arthroscopic debridement, volar ligament reconstruction (with APL or FCR tendons), or metacarpal extension osteotomy. For stage III/IV arthritis, treatment options include implant arthroplasty or resection arthroplasty +/- LRTI (with APL, FCR or palmaris longus), and fusion (young patients).

     

    Figure A shows adduction contracture of the 1st webspace, with hyperextension deformity of the 1st MCP joint. Figure B illustrates decreased thumb abduction because of adduction contracture with decreased palmar abduction (normal, 45deg) and decreased radial abduction (normal, 60deg). Illustration A is a radiograph showing thumb CMC arthritis with Z deformity. Illustration B shows lupus thumb deformity ("hitchhiker thumb"). Illustration C shows hand changes in inflammatory arthritis.

     

    Incorrect Answers:

    Answer 1: Type II thumb hypoplasia does not present with "Z" deformity. Answer 2: Chronic median nerve neuropathy leads to flattening of the thenar eminence and an ape hand deformity from loss of opponens pollicis. There is no loss of thenar bulk or thumb opposition in Figure A.

    Answer 3: There is no lupus thumb deformity (Illustration A). Lupus thumb deformity is characterized by flexion at the MCP and hyperextension at the interphalangeal joint.

    Answer 4: Extensor tendon rupture will not lead to hyperextension deformity of the MCP joint.

     

     

     

     

     

     

     

     

     

     

  77. A 26-year-old man presents with chronic hand weakness. The clinical appearance of his hand, and radiographs are shown in Figures A through C. Surgical exploration and decompression is performed. Besides addressing thumb interphalangeal and index distal interphalangeal joint flexion, which is the most appropriate treatment to restore thumb opposition?

     

     

     

     

     

     

     

     

     

    1. Ring flexor digitorum superficialis transfer to the abductor pollicis brevis

    2. Extensor indicis proprius transfer to the abductor pollicis brevis

    3. Neurotization of thenar muscles

    4. Camitz palmaris longus transfer to the abductor pollicis brevis

    5. Thumb carpometacarpal joint arthrodesis

    CORRECT ANSWER: 2

    This patient has a high median nerve neuropathy because of a supracondylar spur and ligament of Struthers. Reconstruction is best performed with extensor indicis proprius (EIP) transfer to the abductor pollicis brevis (APB).

     

    In low median nerve palsy, the primary concern is restoration of thumb opposition. In high median nerve palsy, thumb opposition and IP flexion, and index and middle finger flexion have to be addressed. The four common opposition transfers include (1) ring or long FDS, (2) EIP, (3) Camitz palmaris longus (PL), or the Huber abductor digiti minimi (ADM).

     

    Anderson et al. reviewed EIP transfer vs FDS transfer. They found a higher percentage of excellent results in the EIP group. In their series, complications included index finger extensor lag (EIP transfer if the extensor expansion was not repaired) and limited donor finger extension because of lateral band damage or adhesions between the remaining FDS tendon and flexor sheath (FDS transfer).

     

    Cawrse et al. modified the Huber ADM opponens transfer by releasing the proximal end to prevent compression of the ulnar nerve in Guyon's canal by the rotated ADM belly. They found that this technique successfully restored

    opposition and thenar bulk.

     

    Figure A shows thenar wasting. Figures B and C show a supracondylar spur. The ligament of Struthers attaches from this spur to the medial epicondyle, under which median nerve and brachial artery pass. Illustration A shows EIP transfer. Illustration B shows FDS transfer. Illustration C shows Camitz PL transfer. Illustration D shows Huber ADM transfer.

     

    Incorrect Answers:

    Answer 1. The FDS is denervated in a high median nerve palsy. Answer 3: Neurotization is performed for pre-ganglionic brachial plexus injuries.

    Answer 4: The palmaris longus is denervated in a high median nerve palsy. A palmaris longus transfer does not restore thumb pronation and metacarpal flexion, thus not simulating opposition.

    Answer 5: Thumb CMC arthrodesis will not restore opposition.

     

     

     

     

     

     

     

     

     

     

     

     

     

     

     

  78. A 2-year-old child is referred by her pediatrician for fixed flexion deformity of the left thumb. She has been wearing a splint for the last 6 months. She has ventricular septal defect and left renal agenesis. The interphalangeal joint does not extend past 40 degrees of flexion as seen in Figures A and B. There is no triggering. There is a firm, nontender nodule overlying the metacarpophalangeal joint as outlined in blue in Figure C. What is the diagnosis and most appropriate treatment?

     

     

     

     

     

     

     

     

     

    1. Thumb camptodactyly. Therapy including passive stretching exercises.

    2. Congenital clapsed thumb. Percutaneous release of the A1 pulley.

    3. Pediatric trigger thumb. Open release of the A1 pulley.

    4. Pediatric trigger thumb. Open release of the A1 pulley and resection of the tendon nodule.

    5. Blauth Type I hypoplastic thumb. Open release of the A1 pulley and volar plate, and resection of the tendon nodule.

    CorreCt answer: 3

     

    This child has pediatric trigger thumb (PTT). The potential for spontaneous resolution beyond the age of 2 years is limited. Surgical release of the A1

    pulley is indicated.

     

    Pediatric trigger thumb presents as fixed flexion at the interphalangeal joint (IPJ) rather than triggering. It is likely to be acquired (rather than congenital). It is associated with the presence of Notta's nodule, a thickening of the FPL tendon and overlying tendon sheath. Treatment involves A1 pulley release.

    The role of non-surgical management (splinting/stretching) remains unclear. The duration of non-surgical treatment is long (up to 30 months) and compliance can be difficult.

     

    Shah et al. reviewed pediatric trigger thumb. The condition is associated with MCP hyperextension. The authors note no advantage to percutaneous release as general anesthetic is required anyway.

     

    Marek et al. performed a retrospective review and survey response review of surgery for pediatric trigger thumb. They found that age at the time of surgery influences residual flexion contracture and rate of recovery. They found surgery to be safe and effective, and recommend: (1) surgery for a 2-year-old child with a locked thumb for 6 months, (2) observation for a child <1 year if the thumb is triggering (not locked), and (3) a 6-month observation period if observation is advocated.

     

    Figures A and B show a fixed flexion deformity of the thumb and an attempt at thumb extension. Figure C shows the outlined Notta nodule.

     

    Incorrect Answers:

    Answer 1: Camptodactyly is a congenital flexion deformity that usually involves the little finger. Surgery is indicated in congenital trigger thumb for patients >3 yrs, and those that have failed a course of splinting/stretching. The efficacy of passive exercises has not been established.

    Answer 2: Percutaneous release of the A1 pulley in PTT carries a risk of incomplete release, radial digital nerve injury, and FPL laceration. The gold standard is open release. This is not congenital clasped thumb.

    Answer 4: The tendon nodule, or Notta's nodule, does not have to be resected. Answer 5: The volar plate does not have to be released. This is not thumb hypoplasia. Type I thumb hypoplasia is treated nonoperatively.

     

     

     

  79. A 48-year-old hairdresser presents with pain and swelling of his ring finger for 4 days. On examination, there is generalized tenderness along the entire digit. Passive extension of the digit triggers

    excruciating pain. The clinical appearance of the digit is shown in Figure A. What is the most appropriate next step in management?

     

     

     

     

    1. Acyclovir

    2. Intravenous antibiotics, splinting and elevation

    3. Closed tendon sheath irrigation from the level of the A1 pulley (proximal) to the distal interphalangeal joint (distal)

    4. Continuous closed tendon sheath irrigation from the wrist (proximal) to the distal interphalangeal joint (distal)

    5. Open irrigation and debridement

    CORRECT ANSWER: 5

    This patient has advanced pyogenic flexor tenosynovitis (PFT) with visible ischemia/necrosis. Open irrigation and debridement is necessary.

     

    Pyogenic flexor tenosynovitis is usually caused by a puncture wound (although it may infrequently arise from hematogenous spread). The most common organism is Staphylococcus aureus. Kanavel signs help differentiate this disease from herpetic whitlow, septic arthritis, gout/pseudogout, and other hand infections such as paronychia, felons, cellulitis, and deep space infections.

     

    Draeger et al. reviewed the treatment of pyogenic flexor tenosynovitis (PFT). They recommend open irrigation and debridement for advanced PFT and atypical or chronic tenosynovial infections where tenosynovectomy may be

    indicated. Both midaxial and volar zigzag incisions can be used.

     

    Pang et al. reviewed factors affecting the prognosis of PFT. Of the 4 Kanavel signs, they found that fusiform swelling was most often present (97% of patients), followed by pain on passive extension (72%), semiflexed digit posture (69%), and tenderness along the flexor tendon sheath (64%).

     

    Figure A shows advanced PFT demonstrating subcutaneous purulence and local ischemia in addition to fusiform digital swelling. Illustration A shows the Nevasier technique of closed tendon sheath irrigation. Illustration B shows the setup for continuous tendon sheath irrigation using nested catheters.

    Illustration C shows the incision for open irrigation and debridement.

     

    Incorrect Answers:

    Answer 1: Herpetic whitlow is treated with acyclovir. Pain is at the distal tip of the digit. Painful, clear, fluid-filled coalescent vesicles and bullae are present. Answer 2: Nonsurgical treatment may be appropriate for patients with PFT who present early (<48h after penetrating trauma). Recommended antibiotics include vancomycin and piperacillin/tazobactam.

    Answers 3, 4: In advanced PFT, open drainage is necessary. Closed drainage is possible for early presenting disease. The catheter is passed from proximal to the A1 pulley to the level of the distal interphalangeal joint. In the thumb, the catheter exits just distal to the carpal tunnel. In the small finger, if the ulnar bursa is involved, a second catheter is placed from the A1 pulley to the wrist.

     

     

     

     

     

     

     

  80. Madelung's deformity of the distal radius is caused by which of the following?

    1. Premature fusion of the distal radial ulnar joint

    2. Physeal growth mismatch between the distal radius and ulna

    3. Nutritional deficiency affecting the physeal zone of provisional calcification

    4. Impaired growth of the volar and ulnar aspect of the distal radial physis

    5. Unrecognized trauma CORRECT ANSWER: 4

    Madelung's deformity is that of excessive ulnar/palmar angulation of the distal radius caused by impaired growth of the volar and ulnar aspect of the distal radial physis. It may be caused by either a bony lesion in the palmar/ulnar corner of the distal radial physis or an abnormal radial-carpal ligament (Vicker's ligament). The other answers do not cause Madelung's deformity.

    Leri-Weill dyschondrosteosis is a rare genetic disorder caused by mutation in the SHOX gene that causes mesomelic dwarfism with associated Madelung's defomity of the forearm.

     

    Illustration A is a radiographic example of Madelung's deformity.

     

     

     

     

     

  81. A 17-year-old boy presents with pain in his right elbow for 2 years and limitation in elbow motion bilaterally. He denies any pain or discomfort in his left elbow. He reports no history of trauma to either elbow. He has had two courses of physical therapy, but has noted no noticeable improvement in pain or motion. Examination demonstrates no elbow tenderness on palpation, and there are no neurological deficits. Manual reduction is unsuccessful. The range of motion of both elbows is shown in Figure A. Radiographs of left and right elbow are shown in Figure B and C respectively. What is the most appropriate treatment plan for the right and left elbow?

     

     

     

     

     

     

     

     

     

    1. Bilateral open reduction and application of a hinged external fixator to both elbows

    2. Radial head resection of the right elbow and non-operative management of the left elbow.

    3. Bilateral radial head arthroplasty

    4. Physical therapy and splinting to both elbows

    5. Radial head resection and interposition arthroplasty for the right elbow and radial head resection alone for the left elbow

    CorreCt answer: 2

     

    This patient has bilateral congenital radial head dislocation (CRHD). The right side is symptomatic with significant loss of motion. The left is asymptomatic with minimal loss of active motion. Therefore the most appropriate treatment is radial head resection of the right elbow and non-operative management of the left elbow.

     

    It is important to differentiate CRHD from traumatic dislocation. Clinical features of CRHD include bilateral involvement, presence at birth, other congenital anomalies, familial occurrence, irreducible by closed methods, and

    lack of a history of trauma. Radiological features include dome-shaped radial head and hypoplastic capitellum, relatively short ulna or long radius, deficient trochlea, prominent medial epicondyle, grooving of the distal radius, and anterior curvature of the posterior outline of the ulna.

     

    Bengard et al. reviewed 10 surgically treated and 6 nonsurgically treated CRHD patients. They found no change in flexion-extension and carrying angle postoperatively, but forearm rotation was improved. Surgically treated patients had significant improvement in elbow pain. Ultimately, >25% of patients had wrist pain postop and this must be weighed in the decision process of treatment. They recommend radial head excision as an effective intervention in selected patients with significant elbow pain.

     

    Figure A is a table showing moderately diminished ROM of the right elbow, and minimally reduced ROM of the left elbow. Figures B and C both show posterior dislocation of the radial head (a line along the long axis of the radius should intersect the capitellum in all views).

     

    Incorrect Answers:

    Answer 1: Open reduction is not reliable because of a high likelihood of recurrence of dislocation (even if combined with ulnar/radial osteotomy or annular ligament reconstruction) especially in the setting of a hypoplastic/absent capitellum, dome shaped radial head, and a relatively longer radius.

    Answer 3: Bilateral radial head arthroplasty is not indicated. Prosthetic radial head replacement may lead to recurrent dislocation because of an underdeveloped capitellum and angulation of the radial head away from the capitellum (non-collinear).

    Answer 4: Physical therapy is unlikely to improve pain and ROM of the right elbow, and unlikely to improve ROM in the left elbow.

    Answer 5: Ulnohumeral distraction and interposition arthroplasty (DIA, distraction interposition arthroplasty) may be an option for ulnohumeral arthritis. Interposition arthroplasty is not indicated for radial head disease.

     

     

     

  82. Which of the following muscles provide the primary deforming forces to Bennett and Rolando fractures (base of the 1st metacarpal fractures)?

    1. Pronator quadratus

    2. Flexor pollicis longus

    3. Extensor pollicis longus

    4. Adductor pollicis longus and abductor pollicis

    5. Abductor pollicis longus and adductor pollicis

    CORRECT ANSWER: 5

    The primary deforming forces in Bennett and Rolando fractures are the Abductor pollicis longus and adductor pollicis.

     

    In a Bennet's or Rolando fracture-dislocation the volar-ulnar fracture fragment is held reduced by the anterior oblique ligament while strong deforming forces pull the remaining metacarpal shaft proximally and dorsally, angulate the shaft ulnarly and supinate the shaft. Most important in these deforming forces are the abductor pollicis longus (APL) inserting on the base of the metacarpal which pulls the metacarpal shaft proximally and dorsally and the adductor pollicis (AP) which inserts on the ulnar base of the proximal phalanx and angulates the metacarpal shaft ulnarly and supinates the shaft. Less important is the extensor pollicis longus (EPL) which inserts on the base of the distal phalanx and also adds to the ulnar angulation of the distal fragment.

     

    Soyer reviews the diagnosis, pathoanatomy, and treatment for fractures at the base of the 1st metacarpal. Understanding the biomechanics, anatomical deforming forces, and the exact fracture pattern aids the treating surgeon in determining the most appropriate method of fixation. The most essential factor for obtaining a good functional result is anatomic restoration of the articular surface.

     

    Elgafy et al. examined the terminal anatomy of the posterior interosseous nerve in their cadaver study - identifing six terminal branches and describing methods to avoid injury. They describe how treating surgeons can maximize function and recovery after base of the 1st metacarpal fractures by understanding these nervous branches and specific fracture pattern treatment to avoid iatrogenic injury to the PIN.

     

     

     

     

     

     

  83. A 28-year-old man sustained a complete laceration of the flexor digitorum profundus of his index finger while cutting a watermelon 3 days ago. A clinical photograph is shown in Figure A. The surgeon plans to repair the tendon using a 4-strand core suture technique. Which method of tendon repair will give him the best results in terms of load to failure and gliding resistance?

     

     

     

     

    1. Repair with core suture purchase 5mm from the cut edge only. No epitendinous suture

    2. Repair with core suture purchase 10mm from the cut edge only. No epitendinous suture

    3. Repair with core suture purchase 5mm from the cut edge. Circumferential

      simple running epitendinous suture.

    4. Repair with core suture purchase 10mm from the cut edge. Circumferential Silfverskiold epitendinous suture.

    5. Repair with core suture purchase 10mm from the cut edge. Circumferential simple running epitendinous suture.

    CorreCt answer: 5

     

    Repair with core suture purchase 10mm from the cut edge, coupled with circumferential simple running epitendinous suture will give him the best load to failure and gliding resistance.

     

    The strength of tendon repairs depend on the number of strands crossing the repair site. Ideally, repairs should have 4-6 strands to allow for early active motion. A running epitendinous suture is recommended to improve tendon gliding and repair strength.

     

    Gulihar et al. compared 3 different epitendinous suture techniques. They found that compared with an intact tendon, gliding resistance increased 100% with the Halsted repair, 80% with the Silfverskiold repair and 60% with a running suture. They thus recommend a simple running suture when an epitendinous suture is needed.

     

    Lee et al. compared core suture purchase at 3, 5, 7 and 10mm from the cut edge. The 10mm-repair group had the highest 2-mm gap force and ultimate failure load. They recommend 10-mm suture purchase for optimal performance and to allow early active motion.

     

    Figure A shows a laceration to the volar aspect of the index finger in flexor zone II. Illustration A shows a core suture purchase distance from the cut edge (represented by "X", where 10mm is the ideal distance). Illustration B shows 3 different epitendinous suture techniques (A, simple running; B, Silfverskiold; C, Halsted).

     

    Incorrect Answers:

    Answers 1, 3: Suture purchase 5mm from the cut edge is inferior to purchase at 10mm from the cut edge.

    Answer 2: An epitendinous suture adds to repair strength and improves gliding compared with no epitendinous suture.

    Answer 4: The Silfverskiold technique has increased gliding resistance compared with a simple running suture.

     

     

     

     

     

     

     

     

     

  84. A 28-year-old professional baseball player injures his middle finger sliding into the catchers shin guard at home plate. He complains of pain and deformity of the middle finger. A radiograph is provided in figure A. All of the following are true EXCEPT:

     

     

     

    1. Anatomic reconstruction of the articular surface is prognostic of clinical function

    2. Proximal interphalangeal joint subluxation precludes a normal gliding flexion arc

    3. Hinging at the fracture site must be avoided

    4. Early motion should be initiated in postoperative therapy

    5. Early degenerative arthritis can be expected if the joint is not adequately reduced.

    CorreCt answer: 1

     

    The radiograph demonstrates a dorsal fracture dislocation of the proximal interphalangeal joint of the middle finger. Kiefhaber and Stern review the presentation, evaluation, and treatment of PIP fractures. Congruent reduction of the joint to allow the middle phalanx to glide around the proximal phalangeal head is paramount to prevent joint subluxation and instability.

    Anatomic reconstruction of the articular surface is desirable but not necessary for successful clinical outcome.

     

     

     

  85. A 30-year-old male sustains a 3.5 cm long thumb pulp injury seen in Figure A. He undergoes a procedure to restore the soft tissue envelope. Which treatment option is contraindicated because of increased risk of interphalangeal joint stiffness?

     

     

     

     

    1. Moberg volar advancement flap

    2. Foucher first dorsal metacarpal artery flap

    3. Littler neurovascular island flap

    4. Free great toe pulp transfer

    5. Holevich first dorsal metacarpal artery flap

    CorreCt answer: 1

     

    This patient has a large thumb pulp defect measuring 3.5 cm in length, extending proximal to the interphalangeal joint (IPJ) crease. Inset of a Moberg flap large enough to cover the defect would necessitate IPJ flexion >45 degrees, increasing the risk of IPJ stiffness.

     

    Thumb pulp defects may be resurfaced by different means, depending on size. The Moberg flap is suited for medium (1.8-3 cm) defects. For defects >1.5 cm, there is increased risk of wound dehiscence, parrot beak nail deformity, and decreased soft tissue padding. Modifications such as V-Y flaps, bilateral Z-plasties, Burrow triangles, 2 lateral triangular flaps at the proximal edge of the flap, or advancement of an island flap with skin grafting of the secondary defect (O’Brien modification), are recommended.

     

    Baumeister et al. reviewed the functional outcome of Moberg flaps. These flaps do not cause marked impairment of active ROM and any reduction in the AROM of the IP joint is because of a loss of hyperextension.

     

    Horta et al. reviewed the use of multiple flaps (Moberg, radial innervated cross-finger, Venkataswami-Subramanian, Foucher, Tezcan, and Littler). They recommended the Foucher flap because of good sensibility, single-stage surgery, and no need for cortical reintegration (unlike the Littler flap)

     

    Figure A shows a large thumb pulp defect. Illustration A shows the options for resurfacing thumb pulp defects of different sizes. Illustration B is a diagram of these options. Illustrations C and D depict the Holevich dorsal metacarpal artery flap (with overlying skin strip). Illustrations E and F depict the Foucher dorsal metacarpal artery flap (islanded).

     

    Incorrect Answers:

    Answers 2, 3, 4, 5: These flaps are all possible options for large thumb pulp defects.

     

     

     

     

     

     

     

     

     

     

     

     

     

     

     

     

     

     

  86. Percutaneous screw fixation for non-displaced scaphoid waist fractures has been shown to have which of the following differences compared to closed treatment?

    1. Increased direct and indirect cost

    2. Slower return to work

    3. Higher union rates

    4. Reduced time to fracture union

    5. Improved motion and grip strength after 2 years

    CORRECT ANSWER: 4

    Fixation of non-displaced scaphoid fractures with a percutaneous screw has resulted in a shorter time to union (6-7 weeks versus 10-12 weeks) and faster return to work or sports.

     

    Arora et al found the indirect cost reduction by a quicker return to work was shown to offset the direct costs of surgical intervention.The operatively treated group had a better mean DASH-score than the conservative group. Fracture

    union was seen in the screw fixation group at a mean of 43 days and in the cast immobilization group at a mean of 74 days.

     

    Bond et al found in active military personnel there was faster healing but no difference in ultimate union rates or final grip strength or range of motion between percutanous screw fixation and non-operative groups. The average time to fracture union in the screw fixation group was seven weeks compared with twelve weeks in the cast immobilization group. There was no significant difference in the range of motion of the wrist or in grip strength at the two-year follow-up evaluation.

     

     

     

  87. Constriction ring syndrome, also known as amniotic band syndrome, is a congenital disorder associated with which paediatric foot condition?

    1. Equinovalgus foot

    2. Clubfoot (Congenital talipes equinovarus)

    3. Tarsal coalition

    4. Congenital vertical talus

    5. Polydactyly CORRECT ANSWER: 2

    Constriction ring syndrome is a congenital disorder that is most commonly associated with clubfeet (congenital talipes equinovarus). The reported incidence of clubfeet with concomitant constriction bands ranges from 12-56%.

     

    Constriction ring syndrome is a collection of congenital malformations that occur as a result of intrauterine rings or bands that constrict fetal tissue. The etiology of constrictive ring syndrome remains elusive, though Streeter postulated in 1930 that a germline developmental abnormality is responsible for the development of amniotic constriction bands, hence one of the synomonous terms used to describe the disorder, Streeter’s dysplasia. Normal anatomy is found proximal to the band. Distally, a constrictive band can cause compression of lymphatic and neurovascular structures and result in lymphedema, altered circulation and neuropathy. In severe cases congenital amputation can occur. In terms of other orthopaedic conditions, constrictive ring syndrome is associated with clubfeet, acrosyndactyly and pseudoarthrosis. With respect to clubfeet, surgical treatment is commonly required, which consists of z-plasty releases of the constricted bands, in addition to surgical correction of the clubfoot deformity.

    Gomez reviewed 35 children with clubfeet associated with constriction ring syndrome. In this cohort there was a poor response to casting, as 77% of the children required surgical corrections. Z-plasty releases of the deep bands were performed before the clubfoot correction.

     

    Allington et al. examined the outcome of treatment of clubfeet distal to a lower extremity band in 18 patients (21 feet). Sixteen children (88.9%) underwent surgical treatment after manipulation and serial casting were unsuccessful.

    Mild initial foot deformities and constriction bands located in the distal aspect of the lower leg were associated with the best outcomes.

     

    Incorrect Answers:

    Answer 1: Equinovalgus foot can be seen with cerebral palsy or fibular hemimelia.

    Answer 3: Multiple tarsal coalitions are not typically associated with constriction ring syndrome but can be seen with Pfeiffer syndrome Answer 4: Congenital vertical talus is associated with neuromuscular and genetic disorders.

    Answer 5: Constriction ring syndrome is associated with acrosyndactyly not polydactyly.

     

     

     

  88. You are consulted on a newborn male inpatient who presents with the clinical sign shown in Figure A. All of the following are commonly associated with this syndrome EXCEPT?

     

     

     

     

    1. Bronchopulmonary dysplasia

    2. Cardiac defects

    3. Cleft palate

    4. Encephalocele

    5. Rigid talipes equinovarus

    CORRECT ANSWER:

    1

    All of the above are associated with congenital amniotic band syndrome (ABS) except bronchopulmonary dysplasia, which is most often linked to prematurity.

     

    ABS, or Streeter dysplasia, is a complex congenital disorder characterized by constricting amniotic bands formed in utero. While the etiology remains a topic of debate, the common final pathway results in encircling strands of fibrous tissue causing extrinsic compression of developing fetus, resulting in deformity, vascular occlusion, and possible amputation. Although variable in presentation, appendages are more often involved, and common orthopaedic manifestations include the characteristic extremity bands (Figure A), club foot, anterolateral bowing of the tibia, tibial pseudarthrosis, leg-length discrepancy, and hemihypertrophy. Other associated clinical findings include craniofacial abnormalities, cardiac defects, renal abnormalities, and neural tube defects.

    ABS is a clinical diagnosis, though ultrasonography may aid in characterizing the associated defects in utero. Treatment involves surgical release of the constricting bands, sometimes urgently in the presence of neurovascular compromise. Long-term monitoring is necessary until skeletal maturity to assess for the development of contractures and growth disturbance.

     

    Moran et al. review the diagnosis and management of ABS. The authors discuss the controversial etiology, though the most support exists for the extrinsic theory in which amniotic tissue entangles the developing fetus causing compression and deformation. As a result, the physical appearance and associated features relate to the stage of development at the time of the insult. Appendages are more vulnerable, with a predilection for the more distal aspects of the upper extremities. They note that management is individualized and ranges from cosmetic repair to emergent limb-sparing band release.

     

    Allington et al. evaluated the outcomes following treatment for clubfoot distal to lower extremity constriction bands in the setting of ABS. The authors defined 3 types of bands (I-III) and 3 grades of feet (A-C) based on the depth of the bands and severity of the deformity, respectively. They found that 50% of the patients had a poor long-term result following club foot casting, while the remainder were evenly divided between good and fair. The results were correlated with the type of band and the grade of foot, favoring patients with the least foot deformity.

     

    Gomez et al. also investigated the outcomes of treatment of clubfoot associated with ABS. The author found that the club feet were in almost all

    cases rigid even in the absence of constricting bands of the ipsilateral leg. He reported an abysmal success rate of club foot casting (6%), with 77% of patients requiring surgical Z-plasty and constriction band release prior to correction.

     

    Figure A shows the characteristic appearance of the extremity constriction bands and digital amputations associated with ABS.

     

    Incorrect Answers:

    Answer 2: Cardiac defects are associated with ABS and may result from constricting bands across the developing chest.

    Answer 3: Craniofacial defects such as cleft lip and palate are associated with ABS.

    Answer 4: Encephalocele is associated with ABS. As opposed to a typical encephalocele which is midline, those associated with ABS are off the midline. Answer 5: Talipes equinovarus, or club foot, is present in up to 25% of cases of ABS, attributable to a mix of paralytic or idiopathic deformity and most often rigid in nature.

     

     

     

  89. The parents of a 2-year-old girl are concerned that their daughter has difficulty feeding herself from a bottle. They have noticed that she rotates her elbow in front of her body when trying to bring the bottle to her mouth. Physical exam demonstrates 10 degrees of elbow hyperextension and 160 degrees of flexion. The forearm does not actively or passively rotate. A radiograph is provided in figure A. Which of the following would be an indication for a future surgical intervention?

     

     

     

    1. Forearm fixed in 45 degrees of pronation

    2. Forearm fixed in 30 degrees of supination

    3. Patient younger than 3 years of age

    4. An affected older sibling

    5. Bilateral involvement CORRECT ANSWER: 5

    Radiograph demonstrates proximal radioulnar synostosis. This rare congenital deformity is most frequently a pronation deformity, but is not frequently a functionally limiting deformity. Observation is the treatment of choice in most cases. Surgical osteotomy and fusion is beneficial for bilateral involvement with the objective of achieving one arm fixed in modest pronation and the other fixed in modest supination to facilitate competence in activities of daily living and hygeine. Surgical excision and reconstruction has not demonstrated successful outcomes.

     

     

     

  90. A 7-year-old nonverbal boy with severe Autism is brought to the emergency department by his caretaker after noticing a bump over the left elbow. She states that the patient falls often but is not sure when the bump first appeared. The patient moves his bilateral upper extremities spontaneously and without apparent discomfort. Examination of his left elbow is notable for a prominence over the

    posterolateral elbow that is nontender. Plain radiographs are pictured in Figures A and B. What is the next best step in management?

     

     

     

     

     

     

     

    1. Plain radiographs of the contralateral elbow

    2. Closed reduction under sedation

    3. Open reduction with annular ligament reconstruction

    4. Open reduction with ulna osteotomy

    5. Radial head resection CORRECT ANSWER: 1

    The clinical picture is most consistent with a congenital radial head dislocation (CRHD), which is often bilateral. Radiographs of the contralateral elbow can be useful in confirming diagnosis. The management of asymptomatic CRHD is observation.

     

    CRHD can occur in isolation or in association with a variety of syndromes. It is most commonly bilateral, posteriorly dislocated, atraumatic, difficult to reduce with distinct radiographic findings that include a concave radial head and hypoplastic capitellum. Management of asymptomatic CRHD is observation. If patients develop pain and/or symptomatic loss of motion, radial head resection can be considered.

     

    Manske et al reviewed the classification of congenital anomalies of the hand and upper extremity. CRHD can occur with a number of other anomalies including longitudinal deficiencies of the radius and proximal radioulnar synostosis. The underlying abnormality of CRHD is thought to be failure of capitellum development, resulting in a loss of contact pressure required for normal radial head development. Malformation of the radiocapitellar joint leads to altered biomechanics at the proximal radioulnar joint and abnormal ulna development.

     

    Agnew et al performed a retrospective review of six patients with unilateral radial head dislocations. Radiographic findings associated with congenital etiology include: relative shortening of the ulna or lengthening of the radius, absence or hypoplasia of the capitellum and a dome-shaped radial head with a long narrow neck. Though most often bilateral, the authors concluded that unilateral congenital radial head dislocation can occur.

     

    Figures A and B are the AP and lateral radiographs, respectively, of the left elbow showing a posterior radial head dislocation with concave radial head. Illustrations A and B are the AP and lateral radiographs, respectively, of the right elbow in this patient showing posterior dislocation of the radial head as well. Illustration C demonstrates clinical and radiographic criteria for diagnosis of CRHD.

     

    Incorrect Responses:

    Answers 2, 3, 4, 5: The best management for this patient with asymptomatic CRHD is observation and imaging of the contralateral elbow to evaluate for

    bilateral CRHD.

     

     

     

     

     

     

     

     

     

     

     

     

     

  91. The cross section of a peripheral nerve is shown in Figure A. What are the parts labeled A through F respectively?

     

     

     

     

    1. axon, myelin, perineurium, epineurium, endoneurium, node of Ranvier

    2. axon, myelin, epineurium, perineurium, endoneurium, node of Ranvier

    3. axon, endoneurium, epineurium, perineurium, myelin, node of Ranvier

    4. axon, epineurium, perineurium, endoneurium, myelin, node of Ranvier

    5. axon, perineurium, epineurium, endoneurium, myelin, node of Ranvier

    CORRECT ANSWER: 2

    The labelled parts are axon, myelin, epineurium, perineurium, endoneurium, node of Ranvier respectively.

     

    Peripheral nerves are highly organized structures. The axon is the main conducting structure. The myelin sheath surrounds the axon and increases conduction velocity. The endoneurium is fibrous tissue around the axon that aids formation of the Schwann cell tube.Fascicles are a group of axons and surrounding endoneurium. The perineurium is connective tissue sheath around each fascicle is is the primary source of tensile strength and elasticity. The epineurium is a layer around a group of fascicles and cushions against external pressure.

     

    Figure A and Illustration A show the hierarchical structure of a peripheral nerve.

     

    Incorrect Answers:

    Answers 1, 3, 4, 5: The myelin sheath surrounds the axon.

     

     

     

     

     

  92. A 25-year-old man sustains a shearing injury to the dorsal aspect of his ring finger. The nail plate was avulsed, leaving the nail bed exposed. The eponychial fold was also avulsed, leaving terminal extensor tendon exposed, devoid of paratenon. A clinical photograph is seen in Figure A. What surgical procedure will provide the best soft tissue coverage?

     

     

     

     

    1. Full-thickness skin graft

    2. Toe nail bed graft

    3. Volar cross-finger flap

    4. Heterodigital, reversed neurovascular island flap

    5. Reversed cross finger flap

    CORRECT ANSWER: 5

    This patient has a dorsal digital defect with exposed tendon. A reversed cross finger flap (RXFF) from the adjacent finger (middle finger) will provide the best soft tissue coverage.

     

    A RXFF is prepared similar to a standard cross finger flap (XFF) except that the hinge and donor tissue differ. The hinge of the flap is opposite the primary defect, unlike a standard XFF, where the hinge is adjacent to the defect.

    Dissection is kept to the level of the dermis, below hair follicles but above the subcutaneous veins. The subcutaneous tissue is raised and inset into the defect, leaving the donor paratenon untouched. A full thickness skin graft is then set on the flap. Flap division occurs at 2-3weeks.

     

    Kappel et al. describe the XFF in 205 patients. The index finger was the most

    common recipient, and the middle finger was the most common donor finger. Half had cold intolerance and mean 2 point discrimination was 8.25mm.

    Possible complications include hyperpigmentation and finger stiffness (especially in older patients). Contraindications include vasospastic conditions (e.g. Buergers) and preexisting deformity (e.g. Dupuytrens).

     

    Atasoy described the technique of a RXFF. He states that the best donor site is the dorsum of the middle and proximal phalanges, and recommends avoiding the dorsum of DIP and PIP joints because of thinness of subcutaneous tissues in these regions.

     

    Figure A shows a dorsal digital defect with exposed extensor tendon. Illustration A shows the steps in harvesting and insetting a reversed cross finger flap. Illustration B shows the difference between standard (above) and volar (below) cross finger flaps. Illustration C shows a volar cross finger flap from the middle finger. Illustration D shows a neurovascular island flap.

     

    Incorrect Answers

    Answer 1: Direct application of a full thickness skin graft will lead to adhesions to exposed tendon that is devoid of paratenon, and interfere with tendon gliding. An intermediary layer of subcutaneous tissue (from a RXFF) is necessary.

    Answer 2: A toe nail bed graft is used for nail bed defects and will not address eponychial deficiency.

    Answer 3: A volar cross finger flap (uncommon) is used to close a distal thumb amputations. The blood supply of the volar flap is vertical, limiting the size of flap that can be elevated without necrosis. The pedicle of the volar XFF must not be dissected free of the digital neurovascular pedicle near its base. When dividing the pedicle, care must be taken not to injure the neurovascular bundles.

    Answer 4: Heterodigital, reversed neurovascular island flaps are useful for coverage of extensive defects of the fingers that cannot be reconstructed using other flaps, and as an alternative to microsurgical reconstruction.

     

     

     

     

     

     

     

     

     

     

     

     

     

     

  93. A 13-month-old male presents to clinic with a with a fixed flexion deformity as seen in Figure A. When discussing treatment options with his parents it should be noted that delaying surgical intervention until age 2 years will likely cause:

     

     

     

    1. Permanent flexion contracture of the thumb

    2. The patient to become left hand dominant

    3. A slight delay in motor milestones that will catch up after surgical release

    4. A slight delay in motor milestones that will not catch up after surgical release

    5. No sequelae CORRECT ANSWER: 5

    Pediatric trigger thumb can be treated non-operatively until age 2 years without significant risk of sequelae. After age 2 years surgical intervention is recommended because full spontaneous resolution of the contracture after age 2 years is unlikely.

     

    Pediatric (or "congenital") trigger thumb typically progresses from notable triggering of the thumb interphalangeal (IP) joint that progresses to a fixed contraction deformity. Non-operative treatment may be attempted including passive extension exercises or extension splinting, and full spontaneous resolution during the first two years of life has been reported. After age 2 years, spontaneous resolution of the contracture is rare, and surgical intervention is recommended. Delaying surgical release, even in the face of a fixed contracture, until age 2 years is not associated with known sequelae.

    Surgical intervention involves release of the A1 pulley of the thumb, and leads to 95% resolution in all age groups.

     

    Shah et al. present a review of pediatric trigger thumb and trigger fingers. They note that non-operative treatments including passive exercises and extension splinting are of questionable efficacy, but may be attempted and resolution of the triggering may occur up until age 2 years. After age 2 they recommend surgical treatment since the contracture is unlikely to fully spontaneously resolve after that point.

     

    Baek et al. present a prospective cohort of patients with pediatric trigger thumb treated non-operatively (and with no stretching or splinting) and followed for 48 months. They found that 60% of patients resolved spontaneously, and that those who did not fully resolve did have improvement in their thumb IP range of motion. Most of the improvements seen were within the the first 2 years of follow-up after initial diagnosis (in some patients up to age 4 years).

     

    Figure A is a child's hand with a fixed flexion deformity of the interphalangeal joint of the thumb consistent with pediatric trigger thumb.

     

    Incorrect answers:

    Answer 1: Delaying surgical intervention of pediatric trigger thumb until age 2 years is not known to result in any permanent flexion contracture.

    Answer 2: Having a right sided trigger thumb until age 2 years is not known to increase the liklihood of becoming left hand dominant. Handedness is not typically developed until closer to age 4 years.

    Answers 3 and 4: No delay in motor milestones is expected in relation to a pediatric trigger thumb treated non-operatively.

     

     

     

  94. A 2-year-old child presents with flexion contracture of the digit as shown in Figure A. The triggering persists after A1 pulley release. At the revision surgery, the following steps are necessary:

     

     

     

     

    1. Horizontal incision and complete A2 pulley release. If triggering persists, perform circumferential debulking of the flexor digitorum profundus tendon.

    2. Extensile incision and release of A3 pulley. If triggering persists, release 1 slip of the flexor digitorum superficialis tendon.

    3. Extensile incision and release of 1 slip of the flexor digitorum superficialis tendon. If triggering persists, perform circumferential debulking of the flexor digitorum profundus tendon.

    4. Horizontal incision and complete A3 pulley release. If triggering persists, perform circumferential debulking of the flexor digitorum profundus tendon.

    5. Horizontal incision and release of 1 slip of flexor digitorum superficialis. If triggering persists, release the A3 pulley.

      CorreCt answer: 2

       

      An extensile incision should be chosen to approach the pediatric trigger finger as the pathology could lie at many sites. The ulnar FDS slip should be resected. If triggering persists, release of the A3 pulley may be performed, or release of the remaining FDS slip.

       

      In the pediatric population, trigger thumbs are 10 times more common than trigger fingers. Trigger thumb presents with flexion contracture, and is caused by constriction at the A1 pulley. Trigger fingers present with a palpable Notta's nodule proximal to the A1 pulley, flexion contracture, and triggering. Trigger fingers are caused by an abnormal relationship of the FDP and FDS tendons, a more proximal decussation of the FDS, nodules in either the FDS or FDP, a thickened A-2 pulley, or a tight A-3 pulley.

       

      Moon et al. reviewed 7700 children prospectively and found no incidence of trigger finger or thumb. They then reviewed 35 trigger thumbs and 8 trigger fingers. Surgical release was performed in 23 thumbs. Spontaneous resolution was noted in 12 thumb cases and all 8 fingers. They concluded that trigger finger develops earlier in life and has a higher rate of spontaneous resolution.

       

      Cardon et al. retrospectively reviewed 239 trigger digits in 176 children. Trigger fingers accounted for 14% of digits and 10% of patients. Average age at presentation was 31 months, and age at onset was 11 months. There were 9 long, 4 ring and 5 small trigger fingers.

       

      Figure A shows flexion contracture of the long finger of the left hand secondary to pediatric trigger finger.

       

      Incorrect Answers:

      Answer 1: Complete A2 pulley release would lead to bowstringing. A horizontal incision does not allow proximal and distal exploration.

      Answers 3: Debulking of the FDP tendon would lead to an irregular tendon surface, which would in turn promote tendon adhesions and scarring.

      Answer 4: The first step should be release of the ulnar slip of FDS. If triggering persists, the A3 pulley can be released if triggering is seen at this location.

      Debulking of the tendon should not be performed to reduce the risk of adhesions.

      Answer 5: While A3 pulley release is a correct second step if triggering persists in spite of A1 pulley and FDS slip release, a horizontal incision will not allow distal exploration.

       

       

  95. A 45-year-old patient undergoes multiple debridements for necrotizing fasciitis of the forearm, leaving the defect shown in Figure

    A. The surgeon plans free fasciocutaneous flap transfer using an antegrade flap based on septocutaneous perforators running in the intermuscular septum, as seen in the cadaveric dissection in Figure B. At 4 months, the donor site incision has the appearance shown in Figure C. Which vessel do these perforators originate from?

     

     

     

     

     

     

     

     

     

    1. Brachial artery

    2. Posterior radial collateral artery

    3. Recurrent interosseous artery

    4. Posterior ulnar recurrent artery

    5. Middle collateral artery

    CORRECT ANSWER: 2

    The surgeon performs a free antegrade lateral arm flap (LAF) for soft tissue coverage of the forearm. The flap is based on 3-5 septocutaneous perforators from the posterior radial collateral artery (PRCA), a branch of the profunda brachii.

     

    The lateral arm flap has a maximum width of 8cm, maximum pedicle length of 8cm, can be raised in the supine position, and the donor site can be closed primarily. During flap elevation, posterior antebrachial cutaneous nerve is often sacrificed, leading to hypoesthesia of the forearm. During flap elevation, care must be taken to protect the radial nerve, which runs nearby, to prevent transient radial nerve palsy. The flap can also be raised as an osteofasciocutaneous flap (with a vascularized humerus block), as a sensate flap (with posterior brachial cutaneous nerve), musculofasciocutaneous flap (with triceps), or just a fascial flap.

     

    Sauerbier et al. described a series of 21 patients undergoing LAF for hand and forearm defects. Maximum flap width was 8cm, and maximum length was 20cm. All flaps survived. One flap required revision of the anastomosis. There were no complications. They advocate this flap for good aesthetic appearance and reliable coverage.

     

    Katsaros et al. described the LAF in 32 cadaver dissections and 21 clinical

    cases. They found that the profunda brachii and PRCA were consistently present, and had 4-5 branches running in the lateral intermuscular septum. The most proximal branch was the largest (1mm diameter), and the maximum pedicle length was 8cm.

     

    Figure A shows a forearm defect with a long incision running into the hand, typical of that for radical debridement for necrotizing fasciitis. Figure B shows cadaveric dissection of the LAF. The incision is larger than would be in a clinical case. Figure C shows a healed but visible incision that was closed primarily. Illustration A is a cross-sectional diagram showing the proximity of radial nerve.

     

    Incorrect Answers:

    Answers 1, 3, 4, 5: The LAF is based on the PRCA, which is a branch of the profunda brachii.

     

     

     

     

     

     

  96. Figures A and B are the radiographs of a 15-year-old boy with a congenital condition of the left elbow. His forearm is fixed in 75 degrees of pronation. If operative treatment is undertaken with the goal of restoring motion, what step should be included in the procedure to prevent recurrence of the condition?

     

     

     

     

     

    1. Soft tissue reconstruction

    2. Interposition of material between radius and ulna

    3. Derotational osteotomy to fix the forearm in neutral position

    4. Postoperative radiation

    5. Postoperative casting CORRECT ANSWER: 2

    This patient has a congenital radioulnar synostosis. Recurrence of synostosis is common without interposition of something between the radius and ulna

     

    Radioulnar synostosis is a rare congenital anomoly may cause complete loss of forearm rotation with anklyosis in a position anywhere from neutral to hyperpronation. Physical exam is characteristic and xray confirms the diagnosis. Most cases are treated non-operatively. If excision of the synostosis is undertaken, something must be done to reduce the risk of recurrence, such as interposition of a tissue that does not form bone or scar (e.g. free fat, pedicled fat, muscle, Gelfoam, Micropore). The other surgical option is derotational osteotomy, which does not restore motion but may be used to fix the forearm in a more functional position.

     

    Cleary et al. evaluated 23 patients with forearms fixed in an average of 30 degrees of pronation. They found that most patients had few or no functional limitations and were employed in jobs that demanded heavy forearm use.

    They concluded that operative treatment is rarely indicated and that functional tests rather than forearm position should be emphasized.

     

    Kanaya et al. excised the synostosis of 7 patients (average age 8 years) using fat graft interposition. Average post op supination was 26 degrees. There were no cases of recurrent ankylosis. Their article includes a nice write up of the operative technique. Their review of the literature found that mobilization procedures without interposition failed (recurrent synostosis) in all but one of 22 patients (23 forearms).

     

    Figures A and B show anterior-posterior (AP) and lateral views of the elbow demonstrating proximal radio-ulnar synostosis. Illustrations A and B show AP and lateral views of the elbow demonstrating proximal radio-ulnar synostosis and posterior radial head dislocation. Illustrations C and D show postoperative radiography of the same patient.

     

    Incorrect answers:

    Answer 1: Reconstruction of the capsule, annular ligament or biceps tendon may be indicated depending on the deformity but will not affect recurrence of the synostosis.

    Answer 3: Although this is an accepted treatment for this procedure, the stem asks about treatment to restore motion.

    Answer 4: This is not described for this condition.

    Answer 5: Immobilization may be utilized in postoperative period but will not prevent recurrent synostosis.

     

     

     

     

     

     

     

     

  97. A hand surgeon is about to repair a zone II flexor tendon rupture in the middle digit. He decides to perform wide-awake repair. Which of the following is true about wide-awake tendon repair?

    1. It is safe to perform an injection of local lidocaine with epinephrine into the hand and digit

    2. Surgery is performed under brachial plexus block with sedation

    3. A forearm tourniquet is used instead to prevent arm tourniquet discomfort

    4. Gaps are revealed by the surgeon passively flexing the digit through the total arc of motion

    5. Wide-awake tendon repair has increased the rate of identification of gaps, but has not reduced the need for late post-operative tenolysis.

    CorreCt answer: 1

     

    Wide-awake flexor tendon repair uses local lidocaine and epinephrine injections.

     

    Epinephrine is now known to be safe when used in the digits and hand and does not cause necrosis even at high doses (up to 1:1000). Finger necrosis previously blamed on epinephrine is known to be caused by procaine. The antidote phentolamine reliably reverses epinephrine vasoconstriction.

     

    Lalonde et al. recommend tumescent local anesthesia (large volume, low concentration). For < 50 mL injections, they use 1% lidocaine with 1:100,000 epinephrine. If 50-100 mL is needed, they use 0.5% lidocaine with 1:200,000 epinephrine. If 100-200 mL is needed (large forearms), they use 0.25% lidocaine with 1:400,000 epinephrine. They perform the repair with 3-0/4-0 Ethibond Kessler sutures (4-strand) and running 6-0 nylon epitendinous sutures.

     

    Lalonde describe postoperative therapy after wide-awake tendon repairs. They immobilize and splint the hand for 3 days. After the 3rd day, they allow midrange active motion, allowing patients to make a half a fist: 45 degrees of active flexion at MP, PIP, and DIP joints, but not full flexion of PIP and DIP joints because friction across the repair and the risk the repair catching on a pulley edge might lead to rupture.

     

    Illustration A shows the sequence of injections (left to right) of lidocaine-epinephrine for wide-awake tendon surgery.

    Incorrect Answers:

    Answer 2: Wide-awake surgery is done only under local anesthesia without sedation.

    Answer 3: No tourniquet is used. Bleeding is reduced by using the admixture of epinephrine and lidocaine.

    Answer 4: Gaps are revealed by the awake patient ACTIVELY flexing and extending through the full range of motion intraoperatively. This allows on-the-spot identification of gaps and tightening of the repair. Without a tourniquet, the surgeon is able to performed a repair without rush as there is no tourniquet pain.

    Answer 5: Wide-awake repairs allow identification of repairs that will not fit through the pulleys because if they do not fit through pulleys during surgery, they will not fit through after surgery. This allows for debulking of bulky repairs, division of the A4 and venting (partial division) of the A2 pulleys.

    These actions have reduced the need for late tenolysis.

     

     

     

     

     

     

  98. A 49-year-old construction worker presents with pain in his small finger for 2 months. Figure A shows the Allen's test on the involved hand. An arteriogram is obtained and is shown in Figure B. Following a period of rest and activity modification, he returns 1 month later with ulceration at the tip of his small finger. At this point, a color flow doppler ultrasound image of the ulnar artery is shown in Figure C. The digital brachial index is 0.4. What is the next best step?

     

     

     

     

     

     

     

     

    1. Catheter-directed thrombolysis

    2. Systemic thrombolysis

    3. Ulnar artery stenting

    4. Ulnar artery ligation

    5. Ulnar artery reconstruction

    CORRECT ANSWER: 5

    This patient has hypothenar hammer syndrome complicated by ulnar artery aneurysm which eventually thrombosed, leading to digital ischemia and ulceration. Treatment involves resection of the involved segment and reconstruction with a vein (or artery) graft as the gap is usually too great for direct repair.

     

    Hypothenar hammer syndrome is post-traumatic digital ischemia from thrombosis of ulnar artery at Guyon's canal and is associated with the use of vibrating tools (e.g. mechanics, construction workers). The hamate hook functions as an anvil, leading to ulnar artery aneurysms, which may cause thrombosis. The digital brachial index (DBI) is a good predictor of outcome and treatment. DBI of 0.85 to 1.25 is normal. If the DBI > 0.7, there are minimal symptoms, the digit is viable, surgery is not warranted. If the DBI < 0.7, arterial reconstruction is suggested. DBI < 0.5 is associated with severe

    symptoms and digital gangrene.

     

    Chloros et al. reviewed 13 patients who underwent artery excision and reversed interpositional vein grafting. They found that 10 (77%) were patent at 2 years. Symptoms were improved from initial presentation, and isolated cold stress test responses were not different from normal controls. Two patients with obstructed grafts still had pain, numbness and cold sensitivity.They concluded that successful reconstruction improves function and quality of life.

     

    McClinton et al. reviewed hypothenar hammer syndrome. They describe the ulnar artery as the most common site of true and false aneurysms, where aneurysm is defined as permanent localized vessel wall dilatation of >= 50% of normal. The cause of ulnar aneurysms is usually hypothenar hammer syndrome. They reconstructed with ipsilateral veins from the forearm or foot dorsum, or artery segments from the thoracodorsal artery, the superficial inferior epigastric artery, or the descending branch of the lateral femoral circumflex artery.

     

    Figure A shows the results of the Allen's test. He has an incomplete palmar arch, with ulnar border digits supplied exclusively by the ulnar artery, and radial border digits by the radial artery. Figure B is an arteriogram showing aneurysmal dilatation of the ulnar artery. Figure C is a longitudinal color flow doppler image showing an area of flow (red, right) adjacent to a thrombus (no color, left). Illustration A shows reversed saphenous vein reconstruction of the ulnar artery and extending into the superficial arch.

     

    Incorrect Answers:

    Answer 1: Endovascular thrombolysis is an option for acute thrombosis without aneurysmal dilatation. In the presence of an aneurysm, the involved segment has to be excised and reconstructed to prevent recurrence and progression of ulceration/ischemia.

    Answer 2: Systemic thrombolysis is not used for this condition.

    Answer 3: Stenting is an option for arterial stenosis but not for thrombosis. Answer 4: Ulnar artery ligation is contraindicated, especially in the presence of an incomplete palmar arch.

     

     

     

     

     

     

     

     

     

  99. What clinical finding is characteristic of involvement of the natatory ligament in Dupuytren's disease?

    1. Palmar pits

    2. Metacarpophalangeal joint contracture

    3. Web space contracture

    4. Distal interphalangeal joint contracture

    5. Tender pads over the dorsal aspect of the proximal interphalangeal joints

    CORRECT ANSWER: 3

    The natatory ligament contributes to web space contracture.

     

    Specific pathological entities in Dupuytren's disease causes the following contractures: The pretendinous cord cases MCP contracture. The natatory ligament causes web space contracture. The spiral cord causes MCP and PIP contracture. The lateral cord causes PIP or DIP flexion contracture. The

    retrovascular cord causes DIP hyperextension contracture.

     

    Strickland et al. discuss the pathogenesis of Dupuytren's disease. They found that the natatory ligaments course between the web spaces of all digits and tighten with abduction. As they cross the flexor tendons, they send fibers to attach to the tendon sheath over the MP joints. They also contribute to the web space coalescence.

     

    McFarlane et al. studied the anatomy of Dupuytren's disease. They state that natatory ligament involvement causes web space and finger joint contracture. Fibers from the natatory ligament contribute to the lateral digital fascia, and thus contribute to PIPJ contracture.

    Illustration A shows the natatory ligament. Incorrect Answers:

    Answer 1: Palmar pits are caused by contracture of the most superficial fibers

    of the pretendinous bands.

    Answer 2: MCP joint contracture is caused by involvement of the pretendinous and spiral cords.

    Answer 4: DIPJ contracture is caused by lateral cord involvement.

    Answer 5: Garrod's pads over the dorsal aspect of the PIPJ are a fibroblastic proliferation and are not caused by palmar fascia involvement.

     

     

     

     

     

  100. All of following are absolute indication to replantation in the hand EXCEPT?

    1. Amputation of multiple digits

    2. Thumb amputation

    3. Amputation at the level of the palm

    4. Single digit amputation distal to the flexor digitorum superficialis insertion

    5. Isolated amputation of little finger proximal to flexor digitorum superficialis in a child

    CorreCt answer: 4

     

    A single digit amputation distal to the flexor digitorum superficialis (FDS) insertion is a RELATIVE indication for replantation, NOT an absolute indication.

     

    Absolute indications for replantation include thumb amputations, multiple digit amputations, amputations at the level of the wrist or palm and any amputation in a child. The aforementioned injuries result in significant loss of hand function and are therefore a priority for surgical repair. Although replantation of single digit amputations is rarely indicated, amputations distal to the FDS insertion have been found to have good functional results after replantation.

    As such, these injuries are a relative indication for replantation.

     

    Soucacos reviewed the indications and selection criteria for digital amputation and replantation. He found the most significant guideline underlining the philosophy of digital replantation to be the aim of not only ensuring the survival of a digit, but its functional use.

     

    Friedrich et al. analyzed the geographic distribution of upper extremity replant procedures, patient factors, and characterized the facilities performing these procedures. They found that regionalization of replantation care and, more broadly, emergency hand care may allow the hand surgery community to provide more efficient and facile care for these patients.

     

    Illustration A shows an amputation of the index finger distal to the FDS insertion. Illustration B and C are clinical and radiographic images, respectively, that demonstrate the successful replantation of the amputated digit.

     

    Incorrect Answers:

    Answers 1, 2, 3 and 5: These are all absolute indications for replantation.

     

     

     

     

     

     

     

     

  101. The median nerve lies immediately ulnar to which of the following structures at the level of the distal radioulnar joint?

    1. Flexor carpi radialis

    2. Flexor carpi ulnaris

    3. Radial artery

    4. Flexor digitorum profundus

    5. Pronator teres CORRECT ANSWER: 1

    The median nerve sits immediately ulnar to the flexor carpi radialis (FCR). This anatomic relationship is demonstrated by the fact that a median nerve injury is most likely to be associated with a deep laceration of flexor carpi radialis (FCR) at the level of the wrist. Additionally, the risk associated in dissecting between the flexor carpi radialis and palmaris longus is injury to palmar cutaneous branch of the median nerve.

     

    Illustration A shows the relative position of the median nerve to FCR at the level of the pronator quadratus on cross section anatomy.

    Illustration B shows a cross-sectional MRI at the level of the wrist.

     

     

     

     

     

     

     

     

     

  102. A 36-year-old man suffered a zone II complete flexor pollicis longus laceration 3 months ago, which was treated with primary repair. He was subsequently lost to follow-up. Now he has no active thumb flexion and imaging studies show a failure of the repair. The decision is made to proceed with a two-stage procedure. Which of the following statements is TRUE regarding this treatment approach?

    1. Palmaris brevis is the most common autograft used

    2. Postoperative tenolysis is required in less than 30% of cases

    3. Supple skin, a sensate digit, and adequate vascularity are required

    4. Preserved tendon sheaths are required for two-stage repair

    5. Full range of motion of adjacent joints is not necessary

    CORRECT ANSWER: 3

    Supple skin, a sensate digit, and adequate vascularity are required for a two stage procedure. Given the failure of primary repair and chronicity of the injury, flexor pollicis longus two stage reconstruction is a reasonable method of treatment.

     

    Flexor pollicis longus injuries can be classified into 3 zones (zone I: distal to IP joint, zone II: between IP and MCP joints, and zone III: deep to thenar muscles). Early end to end tendon repair is the treatment of choice but when primary repair fails and/or injury is chronic, reconstruction is advocated.

    Majority of reconstructions involve a first stage with insertion of a temporary silicone rod and subsequent return at least 3 months later for removal of the rod and insertion of autograft. In few cases, one stage reconstruction may be indicated. Alternatively, arthrodesis of the thumb IP joint may also be considered in this scenario to stabilize the thumb in slight flexion, as this procedure has quicker return to function and less complications than reconstruction.

     

    Kutsumi et al. evaluated gliding resistance at the edge of the oblique pulley in 10 cadaver thumbs. They found a significant increase in gliding resistance at the pulley edge in thumbs undergoing tendon repair. Squaring off the oblique pulley in thumbs undergoing repair did not decrease resistance.

     

    Unglaub et al. reported the results of the two-stage flexor pollicis longus reconstruction (13 patients with palmaris longus, 2 patients with plantaris tendon, one patient with half of FCR tendon). Adequate function was regained in 75% of cases and median DASH score was 11.

     

    Incorrect Answers:

    Answer 1: Palmaris longus is the most common autograft used

    Answer 2: Postoperative tenolysis is required in more than 50% of cases Answer 4: Preserved tendon sheaths are not required for two-stage repair as implantation of silastic implants during the first stage will help create room for the tendon. Preserved tendon sheaths are required in one stage reconstructions

    Answer 5: Full range of motion at adjacent joints is needed for reconstruction

     

     

  103. A 27-year-old motorcyclist is thrown from his vehicle and sustains an open radiocarpal dislocation. The injury is reduced in the emergency room and a post-reduction radiograph is shown in Figure

    A. The main volar extrinsic ligament that must be disrupted to cause this appearance is the

     

     

     

     

    1. Short radiolunate ligament

    2. Scapholunate interosseous ligament

    3. Radiotriquetral ligament

    4. Radioscaphocapitate ligament

    5. Radioscapholunate ligament

    CORRECT ANSWER: 4

    The stout radioscaphocapitate ligament is the main restraint against ulnar translation of the carpus.

     

    On a posteroanterior radiograph, carpal alignment is evaluated using the lunate position relative to the radius. The lunate should align with the ulnar

    column of the distal radius with a minimum of 2/3 of the lunate articulating with the radius. Radiocarpal fracture dislocations can be classified based on the presence of associated intercarpal dissociation (Moneim classification), or the extent of radial styloid involvement (Dumontier classification).

     

    Ilyas et al. reviewed radiocarpal fracture dislocations. They advise suspecting a scapholunate injury when the radial styloid fracture exits at the junction between scaphoid and lunate fossae. They enumerate the steps for surgical treatment as: (1) provisional reduction, (2) neurovascular decompression, (3) joint de´bridement, (4) treatment of intercarpal injuries, and (5) fracture fixation and/or soft tissue repair.

     

    Mudgal et al. retrospectively reviewed 12 patients with radiocarpal fracture dislocations. Open injuries and Moneim Type II injuries result in inferior outcomes compared with closed and Type I injuries. They opine that anatomical repair and stable fixation will lead to a satisfactory outcome in these injuries.

     

    Figure A shows ulnar translation of the carpus. The lunate is now centered over the distal ulna instead of over the lunate fossa of the radius. Illustrations A and B are the Moneim and Dumontier classifications of radiocarpal fracture-dislocations respectively. Illustration C shows the volar extrinsic ligaments.

    The main ligaments seen are

    the radioscaphocapitate (RSC), the radioscapholunate (RSL), long radiolunate (or radiolunotriquetral or volar radiolunate), and short radiolunate.

     

    Incorrect Answers:

    Answer 1: The short radiolunate ligament is likely disrupted. However, while it functions to add mechanical support to the lunate (and in turn, the rest of the carpus), it is not the main mechanical restraint to ulnar translation of the carpus.

    Answer 2: The SLIL is an intrinsic ligament. Disruption leads to a widened SL interval and "Terry Thomas sign" on PA radiograph.

    Answer 3: The radiotriquetral ligament is a dorsal extrinsic ligament. Disruption of this structure (together with lunotriquetral ligament rupture) leads to VISI deformity.

    Answer 5: The radioscapholunate ligament (Ligament of Testut and Kuentz) is a neurovascular conduit and confers no mechanical strength.

     

     

     

     

     

     

     

     

     

     

     

     

  104. A patient with Dupuytren's disease enquires about percutaneous needle fasciotomy and collagenase injections. Which of the following is true regarding these two treatment modalities?

    1. Collagenase injections are more expensive but require fewer treatments than percutaneous needle fasciotomy to achieve the same results

    2. Softening of Dupuytren cords and nodules is observed in cadaveric experiments after collagenase injections but not in a clinical setting

    3. Skin tears are seen following percutaneous needle fasciotomy but not after

      collagenase injections

    4. Pruritic rash and axillary lymphadenopathy may be observed after collagenase injections but are not typically seen after percutaneous needle fasciotomy

    5. Percutaneous needle fasciotomy has a higher incidence of iatrogenic flexor tendon rupture

    CorreCt answer: 4

     

    Pruritic rash and axillary lymphadenopathy is particular only to collagenase injections (CI). These reactions do not occur after percutaneous needle fasciotomy (PNF).

     

    Regional subtotal fasciectomy is the gold standard of care for treatment of Duyputren's contracture. However, because of high complication rates (up to 40%), nonoperative treatments are attractive. In PNF, a 22G or 25G needle is used to release palpable/visible cords in the office. This is followed by manipulation, and night orthosis wear. In CI, 0.25ml of collagenase is used for MCP contracture and 0.20ml for PIP contracture. Manipulation is performed the following day under local anesthesia. This is repeated at 4 weeks if the desired result is not achieved.

     

    Black et al. reviewed Dupuytren's disease. They note that recurrence rates were higher for PNF (85%) compared with fasciectomy (24%) and conclude that PNF is acceptable for early disease with less severe contracture and elderly, sick patients. Regarding CI, they note that the most common complications are edema, contusion, pain, lymphadenopathy, and skin laceration. Major complications, such as complex regional pain syndrome and flexor tendon rupture, are rare.

     

    Nydick et al. retrospectively compared PNF (30 patients) with CI (29 patients). Both groups achieved similar satisfaction, complication rates, and success rates defined as contracture reduction to 0-5 degrees. They concluded that both PNF and CI provide good alternatives to surgery.

     

    Incorrect Answers:

    Answer 1: Collagenase injections are more expensive and more injections are needed to achieve the same results as PNF.

    Answer 2: Softening of Dupuytren cords and nodules is seen in the clinical setting and is one of the reported short term advantages of CI.

    Answer 3: Skin tears are a result of forceful manipulation following either CI or PNF, and not a result of the injection or fasciotomy per se. Skin tears are therefore a complication of both treatments.

    Answer 5: No major complications such as tendon rupture, nerve, or vessel injury were observed in either group in the Nydick et al study.

     

     

     

  105. A 12-year-old boy crushes his middle finger in a car door. He presents with pain and swelling of the finger. Examination reveals an apparent mallet deformity. The nail plate is grossly intact, but there is a subungual hematoma and considerable swelling about the fingertip. Radiographs are shown in Figures A and B. What is the next best step?

     

     

     

     

     

     

    1. Mallet splint in situ

    2. Buddy taping to the ring finger

    3. Nail plate removal, nail bed repair, removal of interposed soft tissue, open reduction

    4. Nail plate removal and replacement, open reduction, tendon repair, distal

      interphalangeal joint-sparing pin fixation

    5. Closed reduction and pin fixation across the distal interphalangeal joint

    CORRECT ANSWER: 3

    This patient has a Seymour fracture with interposed soft tissue (germinal matrix), which must be removed from the fracture site.

     

    Seymour fractures include Salter-Harris I, II, and metaphyseal fractures 2mm distal to the physis. When treating the fracture, the nail should be preserved if possible. CLOSED fractures without nail injury are treated with closed reduction (by hyperflexion) and splinting, or pinning across the DIPJ (if the fracture is unstable). If the reduction is difficult, one can assume tissue interposition and nail plate elevation and removal of soft tissue is indicated (similar to treatment for OPEN fractures). OPEN fractures (with the elevation of the nail plate superficial to the eponychial fold) are treated with nail plate elevation, removal of interposed soft tissue (usually germinal matrix), reduction, and nailbed repair. Replacement of the nail plate beneath the nail fold is sufficient bony stabilization. Extension splinting is maintained for 4-6 weeks. Closed treatment of an open Seymour fracture puts patients at high risk of both osteomyelitis and physeal arrest.

     

    Krusche-Mandl et al. retrospectively reviewed the treatment of 24 Seymour fractures. Nine were treated with closed reduction and splinting; 9 with debridement, open reduction, nail fixation and splinting; 5 with debridement, open reduction, nail fixation and pinning; and 1 with debridement, open reduction and nail removal. They found that 23/24 regained full motion but 11 had nail growth disturbances. They recommend reduction and splinting for closed fractures, and reduction, debridement, and K-wire fixation for instability in open injuries.

     

    Abzug et al. reviewed Seymour fractures. They advocate a more aggressive approach - surgical management for both open and closed injuries to ensure that there is no interposed tissue in the fracture site to reduce the risk of physeal arrest, nail deformity, and chronic osteomyelitis.

     

    Figures A and B are lateral and AP radiographs showing a typical Seymour fracture (Salter-Harris II). Illustration A shows the Krusche-Mandl algorithm for the treatment of closed and open injuries.

     

    Incorrect Answers:

    Answer 1: Splinting alone without reduction will likely lead to physeal growth deformity and nail growth disturbances.

    Answer 2: Immediate buddy taping would immobilize a mal-reduced fracture.

    Buddy taping is not used for the distal phalanx.

    Answer 4: DIPJ sparing fixation with a K wire is not feasible because of limited purchase in the epiphyseal plate. There is also no indication of a tendon injury in this patient. The mallet posture occurs because the terminal extensor slip and the volar plate only insert into the epiphysis (extension force), whereas the FDP spans the epiphysis and inserts on the metaphysis (flexion force).

    Answer 5: Closed reduction and pinning would lock the interposed germinal matrix in the fracture site.

     

     

     

     

     

     

  106. A 50-year-old construction laborer has pain and numbness in his dominant right hand for the past 5 years that often wakes him up at night. He brings wrist splints his occupational health specialist provided him, which fit him well, but no longer alleviate his symptoms. He has a strong grip and pinch, but objective numbness over the thumb, index, and long fingers. He has positive flexion-compression and scratch collapse tests at the carpal tunnel only, and he has neither pain nor a Tinnel's sign over the flexor-pronator mass. You recommend:

    1. Guided hand therapy with an occupational therapist

    2. Modified work responsibilities until symptoms completely resolve

    3. Carpal tunnel release on an elective basis

    4. MRI of the cervical spine and referral to a spine surgeon

    5. Serial carpal tunnel injections

    CORRECT ANSWER: 3

    This patient has a strong history and physical examination findings of carpal tunnel syndrome and should be offered an elective carpal tunnel release.

     

    Carpal tunnel classically presents with numbness and pain in the median nerve distribution, which worsens at night. Physical exam findings such as a Tinnel's sign, Flexion-compression testing, Phalen's test, and Scratch collapse testing over the carpal tunnel allow practitioners to make a clinical diagnosis.

    Injections and electrodiagnositic studies can assist in diagnosing carpal tunnel syndrome, but are most helpful in cases where the diagnosis is less clear clinically. Additionally, AAOS guidelines recommend electrodiagnostic studies if surgical intervention is planned (Grade B evidence).

     

    The study by Graham concluded that in patients with a strong history and clinical examination consistent with carpal tunnel syndrome, electrodiagnostic tests were very unlikely to change the clinical diagnosis. Furthermore, in patients with poor pre-test probability, electrodiagnostic studies were found to be most useful in ruling out carpal tunnel syndrome.

     

    The chapter in OKU 10 by Rozental and LaPorte discusses the clinical presentation and management of carpal tunnel syndrome.

     

    The video below shows a scratch collapse test being performed on a patient with recurrent carpal tunnel syndrome after bilateral releases.

     

    Incorrect Answers:

    Answer 1: Alternative conservative strategies are less likely to be effective in relieving his symptoms than a carpal tunnel release

    Answer 2: Long-lasting carpal tunnel syndrome is unlikely to resolve by activity modification alone

    Answer 4: MRI of the cervical spine is unlikely to change management in this case.

    Answer 5: Serial carpal tunnel injections would not be appropriate in a patient that can undergo an elective carpal tunnel release

     

     

  107. A 32-year-old male sustains a stable dorsal proximal interphalangeal joint (PIPJ) dislocation. Following reduction, there is no hinging at the fracture site on PIPJ flexion and no instability on PIP flexion after reduction. A post-reduction radiograph is shown in Figure

    A. What is the next best step?

     

     

     

     

    1. Volar plate arthroplasty

    2. Cast immobilization for 6 weeks

    3. Dynamic distraction and external fixation

    4. Extension block splinting and active range of motion

    5. Extension block pinning

    CORRECT ANSWER:

    4

    The patient suffered a PIP fracture dislocation and therefore should have guarded ROM initially due to the potential unstable nature of this injury. These fracture-dislocations (FD) can be managed with extension block splinting.

    Extension block splinting prevents extension of the PIPJ into the range where it is unstable, while permitting motion within the stable range.

     

    Dorsal PIPJ FD happen because of 2 mechanisms: (1) hyperextension avulsion

    of the middle phalangeal volar lip (non-comminuted), and (2) axial loading on a flexed PIPJ, leading to a shear fracture (comminuted, impacted). These fractures are classified as stable, tenuous and unstable (Illustration A).

     

    Gaines et al. retrospectively reviewed 190 volar plate avulsion fractures. They achieved excellent/good outcome in 98%. Late presenting patients (>3 weeks) had poorer outcome. For a stable joint, the size and displacement of the avulsed volar plate fragment did not affect outcome. They concluded that early active mobilization with minimal or no splintage provides a good result.

     

    Elfar et al. reviewed fracture dislocations of the PIPJ. They concluded that stable dorsal FD can be managed with buddy taping and early ROM, tenuous FD (with no hinging) can be managed with extension block splinting. If >30° of flexion is necessary, surgery should be considered instead.

     

    Figure A shows a volar plate avulsion fracture with a concentric reduction and no comminution. The fracture fragment measures about 25% of the articular surface. Illustration A is a table showing the Hastings classification of dorsal PIPJ FD. Illustration B is a diagram illustrating the Hastings classification.

    Illustration C shows the concept of "hinging" at the PIPJ for an intraarticular fracture.

     

    Incorrect Answers:

    Answer 1: Volar plate arthroplasty is appropriate for dorsal FD in which reduction cannot be maintained (eg, dorsal subluxation, hinging at fracture site).

    Answer 2: Casting for 6 weeks in the digits will lead to loss of motion. Instead, early active range of motion is encouraged. However, the joint should be protected at least with buddy taping to the adjacent digit (stable injuries) or with an extension block splint (tenuous injuries).

    Answer 3: DDEF uses traction and ligamentotaxis to secure fracture alignment and maintain reduction. It is appropriate for unstable dorsal FD, especially those with comminution.

    Answer 5: Extension block pinning is used for unstable dorsal FD.

     

     

     

     

     

     

     

     

     

     

     

     

  108. Figure A is a clinical image of a 55-year-old female who presents with 5 days of left-sided long finger pain. On exam, the area of maximal tenderness is the midvolar pulp. She denies a history of injury or treatment to the finger. She was seen at urgent care and started on warm soaks, elevation, and oral 1st generation cephalosporin, but the pain and swelling have persisted. What is the best next step in treatment?

     

     

     

    1. Inpatient observation and IV antibiotics

    2. Outpatient observation and oral Colchicine

    3. Incision and drainage with a medial to lateral hemispherical incision and delayed primary closure

    4. Incision and drainage with a medial to lateral hemispherical incision and secondary closure

    5. Incision and drainage with a midline volar incision and secondary closure

    CORRECT ANSWER: 5

    The patient has a felon that needs to be incised over the point of maximal tenderness and allowed to close by secondary intention.

     

    A felon is an infection of the digital pulp at the tip of the finger. Patients will present with swelling and erythema over the affected finger. They may report a history of penetrating trauma however many patients will present without a history of injury to the finger. The most common organism is Staphylococcus aureus. Treatment should consist of incision and drainage with the incision centered over the area of maximal tenderness followed by antibiotic coverage against S. aureus. The wound should be left open to promote further drainage with soaks starting at 24-48 hours.

     

    Abrams et al. reviewed the treatment of various hand infections. For felons, they report once abscess formation has occurred, incision and drainage is indicated. The state that fishmouth incisions have been condemned because of the resultant vascular compromise of the digital pad. They recommend a midvolar or high lateral incisions, with the point of maximal tenderness guiding placement. They warn against proximal probing because of the risk of inoculation of the flexor sheath. They recommend empiric antibiotics and wound healing by secondary intention.

    Figure A shows a clinical picture of a patient with a felon. Illustration A shows an example of tophaceous gout deposits. Illustration B shows an example of a volar midline incision location on a normal finger. Illustration C is an example of a fishmouth incision.

     

    Incorrect answers:

    Answer 1: A felon should be decompressed to allow for adequate drainage. Answer 2: While gout with tophus deposits can look similar to this patient (see illustration A), the clinical presentation is more consistent with a felon.

    Answer 3: Fishmouth incisions can compromise vascular flow to the digital pad and the wound should be allowed to heal by secondary intention.

    Answer 4: Avoiding primary closure is important to allow for further drainage of the felon. However, fishmouth incisions can compromise vascular flow to the digital pad.

     

     

     

     

     

     

     

     

  109. A 50-year-old woman complains of severe pain and clicking in her right thumb after opening a jar 1 year ago. She recalls a history of steroid injection for a chronic tenosynovitis of the flexor pollicis longus tendon. Examination reveals diminished range of thumb motion. A clinical photograph is shown in Figure A. An MRI scan is

    shown in Figure B. An intraoperative photograph is shown in Figure C. What is the diagnosis?

     

     

     

     

     

     

     

     

     

    1. Pulley rupture

    2. Radial collateral ligament rupture

    3. Flexor pollicis longus musculotendinous avulsion

    4. Flexor pollicis longus bony avulsion

    5. Gout

     

    CorreCt answer: 1

     

    This patient has a rupture of the oblique pulley.

     

    The fingers have 5 annular and 3 cruciate pulleys. Closed flexor pulley rupture in the fingers is uncommon and happens in rock climbers. The thumb has 2 annular (A1 and A2) and 1 oblique pulley, and 1 variable annular pulley (Av). The arrangement is A1- Av-oblique-A2. Pulley ruptures in the thumb are rare. Loss of both oblique and proximal pulleys leads to FPL bowstringing.

    Treatment involves pulley reconstruction with a tendon graft. Reconstructing the oblique pulley alone is enough to prevent bowstringing.

     

    Bayat et al. explored the pulleys of the thumb using cadavers. The A1 and Av pulleys are 6mm long. The oblique pulley originated from the ulnar side of the proximal phalanx base and inserted to the radial side of the distal phalanx base and was 4mm wide. The oblique pulley experiences greater strain in extension than in flexion. The A2 pulley was 8mm wide. When both A1 and Av pulleys have been sectioned, the oblique pulley can no longer prevent bowstringing.

     

    Kosiyaktrakul et al described a case report of A1, oblique, and A2 pulley rupture, leading to bowstringing. This was successfully treated with pulley reconstruction of the oblique pulley alone.

    Figure A shows bowstringing of the FPL tendon. Figure B is an MRI image showing volar subluxation of the FPL tendon off the proximal phalanx (they should be closely apposed). Figure C is an intraoperative photograph showing separation of the FPL tendon (volar) from bone (dorsal). Illustration A shows the 3 different types of Av pulley (left, Type I, with gap between A1 and Av; center, Type II, with no gap; right, Type III, oblique orientation).

     

    Incorrect Answers:

    Answer 2: There is no rupture of the radial collateral ligament.

    Answers 3 and 4: The FPL is grossly intact. Bowstringing (tension) can only occur in the presence of an intact tendon that is fixed at both ends (proximal and distal).

    Answer 5: There are no deposits to suggest gout. Gouty attrition can occasionally lead to tenosynovitis and tendon rupture. This is not seen in this case.

     

     

     

     

     

     

  110. A 27-year old female sustains the closed injury shown in Figures A and B. She is treated non-operatively with splint immobilization. At follow-up, she is found to have an extension lag at the metacarpophalangeal (MCP) joint along with reduced grip strength. Which finding is the most significant contributor to this outcome?

     

     

     

     

     

    1. Rotational deformity

    2. 15-degree apex-dorsal angulation

    3. Nonunion

    4. 5mm of shortening

    5. Extensor tendonitis CORRECT ANSWER: 4

    Shortening of metacarpal fractures >2-5mm may result in extension lag at the MCP joint as well as reduced grip strength due to loss of tension on the extensor mechanism.

     

    Indications for nonoperative treatment of metacarpal shaft fractures with splint or cast immobilization include stable fracture pattern, no rotational deformity, and acceptable angulation. The amount of acceptable angulation differs for each metacarpal: 10-20 degrees for index and long fingers, 30 degrees for ring fingers, and 40 degrees for small fingers.

     

    Strauch et al. performed a cadaveric study quantifying the magnitude of MCP extensor lag produced by metacarpal shortening. The authors found that every 2mm of metacarpal shortening resulted in an average of 7 degrees of extensor lag at the MCP joint.

     

    Al-Qattan et al. prospectively reviewed conservative management of spiral/long oblique fractures of the metacarpal shaft treated with palmar wrist splinting. Extension lag was initially seen in all digits, but recovered by 1 year. Grip strength at 1 year was also found to be 94% of the contralateral hand.

    The authors' indications for surgery included severe angulation with failure of closed reduction, rotational malalignment, and significant displacement with loss of bone-to-bone contact.

     

    Figures A and B demonstrate AP and lateral views of a shortened, oblique fracture of the fourth metacarpal shaft.

     

    Incorrect Answers:

    Answer 1: Any amount of rotational deformity is an indication for surgical intervention, but rotation would not result in an extension lag.

    Answer 2: Angulation may result in hyperextension of the MCP and an extension lag of the PIP.

    Answer 3 and 5: Nonunion and tendonitis are rare and would not result in an extension lag.

     

     

  111. A 14-month-old child is brought into your office because the mother has noticed reduced motion in the left upper extremity. The child appears at ease, playing quietly by herself. She abducts her shoulder to pick up building blocks on the ground. Examination reveals normal elbow flexion and extension, but diminished supination compared with the contralateral side. Radiographs are shown in figures A through C. What is the next best step?

     

     

     

     

     

    1. CT

    2. MRI

    3. Arthrocentesis

    4. Reduction

    5. Observation CORRECT ANSWER: 5

    This child has congenital proximal radioulnar synostosis (CPRUS). There are no apparent fractures. Observation is all that is necessary.

     

    CPRUS is a rare condition arising as a failure of longitudinal segmentation. The cartilaginous anlage between the radius-ulna during the 7th week in utero leaves a persistent bridge of tissue. The average age of diagnosis is 6 years and there is a positive family history in 20% of cases. The average position of the forearm is 30 degrees of pronation. Surgery is rarely indicated. Surgical recommendations include positioning the dominant forearm in 10-20° pronation and the nondominant forearm in neutral rotation (bilateral) or 30° of pronation (unilateral cases).

     

    Cleary and Omer reviewed the natural history of CPRUS in 23 patients (10 unilateral, 13 bilateral). All forearms had fixed PRONATION, but the position did not affect functional limitations or employment. They found 4 radiographic patterns: (1) no bony synostosis, radial head reduced, (2) bony synostosis, radial head reduced, (3) long bony synostosis, radial head posteriorly dislocated, (4) radial head anteriorly dislocated.

     

    Kozin reviewed congenital elbow abnormalities. He states that CPRUJS is bilateral in 50%. The shoulder/wrist is able to compensate. The shoulder abducts to compensate for deficient pronation, and adducts to compensate for deficient supination. Children may develop excessive wrist motion. Difficulties from deficient pronation involve keyboard/tabletop activities. Difficulties from deficient supination include eating, washing face, catching a ball and using a soap dispenser.

     

    Figures A through C show a CPRUS of the left upper extremity. Illustrations A through D show the Cleary and Omer classification of Types I-IV CPRUS.

     

    Incorrect Answers:

    Answers 1 and 2: Further imaging will not yield any more useful information. Answer 3: Joint aspiration is unnecessary

    Answer 4: There is no dislocation that would necessitate joint reduction.

     

     

     

     

     

     

     

     

     

     

  112. A 32-year-old male sustains a 75% laceration of the flexor digitorum profundus tendon of the index finger of his dominant hand and presents with triggering. Which of the following techniques uses the least amount of suture necessary to prevent gap formation

    >2mm?

     

    1. No repair

    2. Core suture repair

    3. Epitendinous suture repair of the cut edges only

    4. Circumferential epitendinous suture repair

    5. Core and epitendinous suture repair

    CORRECT ANSWER: 3

    Epitendinous suture repair of the cut edges is all that is necessary to prevent gap formation.

     

    Most surgeons repair lacerations of = 50% if there is triggering and nonrepair for 50-60% lacerations without triggering. Although lacerations of up to 75% are capable of handling early, non-resisted active mobilization, most hand surgeons will repair 75% partial lacerations.

     

    Haddad et al. compared repair vs nonrepair of 75% lacerated sheep flexor tendons. They found that peripheral or peripheral + core repairs reduced gap formation (= 1mm at 500 cycles) but there was no difference in gap formation between these 2 groups. Nonrepaired tendons had gap formation early (100 cycles). They concluded that gapping is reduced with a peripheral repair (with or without core suture).

    Illustration A demonstrates epitendinous suture techniques, whereas Illustration B demonstrates core suture techniques.

     

    Incorrect Answers:

    Answer 1: Gap formation occurs rapidly with no repair.

    Answer 2: Core suture repairs do not provide adequate apposition at the cut edge to prevent gapping.

    Answer 4: For partial lacerations, there is no advantage to circumferential epitendinous sutures compared with epitendinous sutures at the lacerated edge alone.

    Answer 5: The addition of core sutures increases bulk of the repair and is unnecessary to prevent gapping. In the study above by Haddad et al., there was no difference between core+epitendinous and epitendinous alone for partial lacerations.

     

     

     

     

     

     

     

     

  113. A 32-month-old girl presents with bilateral flexion contractures of the small finger proximal interphalangeal joints (PIPJ). Passive extension of the right hand small finger PIPJ is measured at 25 degrees. Passive extension of the left hand small finger PIPJ is measured at 60 degrees. The parents believe these deformities have been present for some time, have been slowly progressing, and the left small finger now bothers the child. The child has no other health concerns and has met normal milestones. You inform the family that:

    1. Passive stretching exercises will correct both small fingers in 1-2 months; no follow up needed

    2. Serial casting has a 90% success rate for contractures > 50 degrees

    3. The patient will likely develop progressive contractures of all small joints of all four limbs regardless of treatment

    4. Passive stretching exercises should resolve the right small finger's contracture, but the left small finger may take significantly longer or require FDS tenotomy if conservative management fails

    5. Bilateral small finger FDS tenotomies will be required as observational management will fail

      CorreCt answer: 4

       

      This patient has infantile camptodactyly (Type I). The right small finger PIPJ (25-degree contracture) can be managed conservatively (stretching and/or splinting). The left small finger PIPJ (60-degree contracture) may improve from a stretching program but ultimately may benefit from surgical management if the severely deformed finger continues to bother the patient.

       

      Idiopathic camptodactyly can occur at birth (infantile, Type I) or in adolescence during growth spurts (Type II). It is sometimes seen in families with autosomal dominant inheritance and variable penetrance, and it is most often caused by abnormal lumbrical or FDS insertion sites. Type III is associated with other musculoskeletal syndromes, and frequently involves multiple digits and severe deformities. Mild deformities (<30 degree contractures, minimally bothersome to the patient) can usually be treated with passive stretching or splinting programs. Progressive contractures should first be management with a trial of stretching; however, if they become significantly bothersome, the patient/family may wish to undergo surgical releases of the FDS tendons. If the PIPJ can be extended fully when the metacarpophalangeal joint is flexed, an FDS tendon transfer to the radial lateral band can be performed as an alternative to a tenotomy.

       

      Rhee et al. published their results of a stretching protocol for congenital

      deformities in children under 3 years old. Mild contractures improved from an average of 20 degrees to 1 degree after 5 months of a rigorous protocol involving passive stretching for greater than 5 minutes, twenty or more sessions per day. In the severe deformity group (>60 degree contracture at onset), the average improvement was from 75 degrees to 28 degrees -however the stretching protocol lasted up to 2 years in this group until a plateau of progress was achieved.

       

      Goldfarb et al. reviews updates in the congenital hand deformity literature from 2009-2013. Little has changed in management of camptydactyly; interestingly, the study by Rhee et al. is the only new clinical study looking at management.

       

      Incorrect Answers:

      Answer 1: These are slowly-corrected deformities with passive stretching protocols, and it is unlikely that a complete resolution of deformity will occur Answer 2: Serial casting is not shown to be effective for significant deformities Answer 3: This condition is usually idiopathic and does not warrant further genetic workup

      Answer 5: Conservative management should always be tried first; only bothersome, severe, and progressive contractures should be considered for operative management

       

       

       

  114. A 26-year-old male sustains the index finger injury shown in Figure A. There is loss of the sterile nail matrix with exposed bare bone. Reconstruction is best achieved with a reverse cross finger flap from

     

     

     

     

    1. the dorsum of the distal phalanx of the thumb

    2. the dorsum of the distal interphalangeal joint of the thumb

    3. the dorsum of the distal phalanx of the middle finger

    4. the dorsum of the distal interphalangeal joint of the middle finger

    5. the dorsum of the middle phalanx of the middle finger

    CORRECT ANSWER: 5

    The injury is best resurfaced with a reverse cross finger (RXFF) flap from the dorsum of the middle phalanx of the middle finger.

     

    The RXFF is indicated for reconstruction of: (1) eponychial skinfold and coverage of an exposed extensor tendon near the IP joint, (2) sterile matrix nailbed defects with exposed distal phalanx, (3) contused, repaired, or grafted extensor tendon denuded of paratenon, (4) boutonniere deformity with poor-quality skin over the PIP joint after burn/avulsion injury, (5) complete avulsion of the nailbed, germinal matrix, and surrounding skin of digits. In such an injury of the index finger, the alternative procedure is a cross thumb to index flap, (6) elective correction of digital deformity. Ideal donor areas include the dorsal aspect of the middle and proximal phalanges of the adjacent fingers.

     

    Atasoy (2016) revisits the RXFF. He describes cases of RXFF for eponychial loss, thumb reconstruction (resection of duplication), nailbed defect, exposed bare tendon laceration.

     

    Atasoy (1982) described the original technique for RXFF. He states that the best donor site is the dorsum of the middle and proximal phalanges, and recommends avoiding the dorsum of DIP and PIP joints because of thinness of subcutaneous tissues in these regions.

     

    Figure A shows full-thickness tissue loss from the nailbed including the sterile matrix, with exposed bone. Illustrations A and B shows resurfacing of the defect in Fig A with a RXFF from the dorsum of the middle phalanx of the middle finger.

     

    Incorrect Answers

    Answers 1 and 3: There is insufficient tissue over the distal phalanges of adjacent digits to allow creation of a RXFF large enough to cover the defects. In addition, using tissue from this region would lead to mobilization of the eponychium and leave the germinal matrix of the donor digit prone to injury. Answers 2 and 4: The dorsum of the interphalangeal joints should be avoided because of thin skin/subcutaneous tissue.

     

     

     

     

     

     

     

     

     

  115. A 40-year-old homeless man comes to the emergency room after his tent got blown away by the wind. He has pain and discoloration in his hands, shown in Figures A and B. The weather is -15°C (5°F) with wind speed of 25km/h (15.5mph). When is the most appropriate time to perform surgical debridement/amputation?

     

     

     

     

     

     

    1. After immersion in a 50°C water bath for 30min, followed by re-evaluation after 6 hours

    2. After immersion in a 42°C water bath for 30min, followed by re-evaluation after 24 hours

    3. After demarcation, which may take 1 to 3 days

    4. After demarcation, which may take 1 to 3 weeks

    5. After demarcation, which may take 1 to 3 months

    CORRECT ANSWER: 5

    Demarcation takes 1-3 months. Surgical intervention should occur after that time period.

     

    Frostbite occurs more commonly in males (10:1) in the 30-50 year old age group. Homeless people and outdoor workers are most at risk. About 20% of injuries involve the upper limb, 47% involve the lower limb, 30% involve both upper and lower limbs and 3% involve the head and face. The most important host risk is alcohol abuse. The risk of frostbite is low when air temperatures are above 14°F (–10°C), regardless of wind velocity, but is high at temperatures below –13°F (–25°C) even when there is little or no wind.

     

    Golant et al. reviewed cold exposure injuries. They state that the spectrum of injury includes nonfreezing injuries (chilblain and trench foot), as well as freezing injuries (frostnip or frostbite). Rewarming should not begin until definitive medical care can be provided to avoid freeze-thaw cycles.

    Rewarming should be rapid and for an affected limb should be performed by submersion in warm water at 104° to 107.6°F (40° to 42°C) for 15 to 30 minutes. Debridement is delayed until there is a clear demarcation, a process that takes from 1 to 3 months.

     

    Figures A and B show the initial appearance of frostbite after 12 hours of exposure. Illustrations A, B and C show the appearance at 1 week, 6 weeks and after amputation, respectively. Illustration D shows the different levels of tissue damage after cold exposure. Illustration D shows the treatment for frostbite.

     

    Incorrect Answers

    Answers 1 and 2: Rapid rewarming should be in warm water (40° to 42°C). Rewarming at a lower temperature reduces the likelihood of tissue survival. Rewarming at a higher temperature may cause thermal burns and worsen the injury. Debridement occurs at 1-3 months, after demarcation.

    Answers 3 and 4: Debridement occurs at 1-3 months, after demarcation.

     

     

     

     

     

     

     

     

     

     

     

     

     

     

     

     

     

  116. A 75-year-old left-hand dominant man with hypertension presents complaining of night-time hand pain and numbness in his left thumb and index finger. On exam, he has a positive Tinel's sign at the

    left wrist with no thenar muscle atrophy. Nerve conduction velocities show motor and sensory latencies of 5 ms with sharp waves and fibrillations on electromyography. Which of the following in this patient would be most predictive of poor resolution of all symptoms after a carpal tunnel release?

    1. Male gender

    2. Positive Durkan's compression test

    3. Electromyography results

    4. Positive Tinel's sign

    5. Pillar pain CORRECT ANSWER: 3

    Severe disease indicative of median neuropathy on electrodiagnostic studies (EDS) is a significant predictor of poor prognosis after surgery for carpal tunnel syndrome (CTS).

     

    Nerve conduction studies and electromyography (EMG) testing can be used as a supplement to clinical examination in the diagnosis of CTS. Distal motor latency >4.5 ms or sensory > 3.5 ms is considered abnormal, as are sharp waves, fasciculations, and fibrillations. While surgery may be indicated based on clinical assessment alone, EDS are often used as a supplement to history and physical in confirming the diagnosis and/or assessing the likelihood of improvement with surgery. Demyelination of nerves may lead to increased latencies (slowing) of nerve conduction velocities (NCV).

     

    Kronlage et al. studied outcomes after mini-open carpal tunnel release in a cohort of 95 patients with moderate or severe CTS by EDS. They reported that patients with severe disease did eventually have similar postoperative pain scores at greater than one year, but had a prolonged course of worse symptoms during that first-year postoperatively as compared to the moderate disease group.

     

    Bland et al. queried over 1200 patients who underwent surgery for CTS. Using multiple variable logistic regression, they identified preoperative severity of disease on electrodiagnostic studies, greater age, lower symptom severity, longer disease duration, and male gender to be associated with poor outcomes. They found that patients with middle-range EDS improved the most after surgery, unlike patients with severely or no positive EDS findings.

     

    Incorrect answers:

    Answer 1: Male gender is associated with worse outcomes, but not as significantly as preop EDS.

    Answer 2: Positive Durkan's compression test is a good indicator of carpal tunnel syndrome but not as reflective of prognosis or resolution of symptoms after surgery.

    Answer 4: Positive Tinel's sign is not a strong prognostic indicator of poor outcome compared to EDS.

    Answer 5: Pillar pain occurs after carpal tunnel release but is not a preoperative predictor of outcome.

     

     

     

  117. A 31-year-old man transects both flexor tendons to each of his index and middle fingers while slicing vegetables. Wide-awake tumescent local anesthesia is employed for flexor tendon repair. What is one of the primary goals of this technique?

    1. Reduce tourniquet pain

    2. Active testing of surgical repair

    3. Detect bowstringing

    4. Assess neurovascular bundle integrity

    5. Compare 2-point sensibility with adjacent digits

    CORRECT ANSWER: 2

    The primary advantage is the ability to test the repair with full active flexion and extension by a comfortable, cooperative, unsedated, tourniquet-free patient.

     

    Other advantages of wide-awake flexor tendon repair include the ability to (1) evaluate if repairs will glide through pulleys, permitting the potential release of the entire A4 and venting of half the A2 pulley if indicated, (2) repair tendons inside tendon sheaths while demonstrating that the sheath has not been inadvertently caught, (3) confidently initiate early active motion if the patient can make a full fist during surgery (in contrast to passive or place-and-hold protocols).

     

    Lalonde reviewed wide-awake hand surgery. He states that a tourniquet is no longer required because of epinephrine hemostasis/tumescent local anesthesia, which is safe in the digit, and its vasoconstriction is reversible with phentolamine. Sedation is unnecessary because no tourniquet is used. Current literature suggests that Wide-awake flexor tendon repair decreases tenolysis and rupture rates. One study reported that maximal vasoconstriction occurs at 26min after injection.

     

    Lalonde and Martin reviewed wide-awake flexor tendon repair and early

    mobilization for zones 1 and 2. They state that historic finger necrosis was a result of of procaine, not epinephrine. The alpha blocker phentolamine is the antidote for vasoconstriction in the finger.

    Illustration A shows methods to decrease pain with local anesthetic injection. Incorrect Answers:

    Answer 1: No tourniquet is used in this technique.

    Answer 3: While bowstringing will be visible (if you cut the A2 and/or A4 pulleys), it is not one of the primary goals of this technique. The goal is to assess repair site excursion through pulleys and release or debulk the repair if necessary prior to skin closure, and thus reduce the chance of the need for late tenolysis.

    Answer 4: The neurovascular bundles can be assessed with the patient awake or asleep.

    Answer 5: Under local anesthesia, the infiltrated digit will be anesthetized. Testing of 2 point discrimination is not a goal of this technique.

     

     

     

     

     

  118. A 21-year-old male underwent percutaneous screw fixation of a nondisplaced scaphoid waist fracture 7 months ago and now presents with persistent radial sided wrist pain. His skin wounds are well-healed. Basic radiographs of the wrist show maintained hardware position. What is the next best step?

    1. Clinched fist stress radiograph

    2. Bone scan

    3. MR arthrography

    4. Fat-suppressed, T2-weighted MR Imaging

    5. Fine cut CT scan CORRECT ANSWER: 5

    After percutaneous screw fixation of a nondisplaced scaphoid waist fracture, the best imaging study to assess for union is fine cut CT scan of the scaphoid along the scaphoid axis.

     

    The etiology of chronic pain symptoms after dorsal percutaneous screw fixation of scaphoid waist fractures include fracture non-union, fracture pseudoarthrosis, loose, migrated, and/or broken implants, as well as extra-osseous factors (e.g. neuroma). Fine cut CT scans have shown to be the most specific imaging study to assess for fracture union, after technical errors have been excluded with basic radiographs (eg, inadequate screw size or poor screw position).

     

    Ring et al. reviewed acute scaphoid fractures. They state that surgical treatment is recommended for displaced and complex fractures (open fractures, perilunate fracture-dislocations, and scaphoid fractures associated with fracture of the distal radius), very proximal fractures, and fractures for which the diagnosis and treatment have been delayed.

     

    Lutsky et al. suggest that scaphoid nonunion is conventionally defined by no healing of the fracture greater than 6 months after fixation of the injury. They state that CT scans are most useful for diagnosing scaphoid union, but they are unreliable for ruling out nonunion.

     

    Yin et al. examined the cost effectiveness of multiple competing diagnostic strategies for suspected scaphoid fractures. They state that immediate CT and MRI were the most cost-effective strategies for diagnosing suspected scaphoid fractures.

     

    Illustration A shows a persistent fracture line with CT imaging. Illustration B shows axial view of the scaphoid with distal cortical hardware perforation of

    the screw.

     

    Incorrect Answers:

    Answer 1: Clinched fist stress radiographs assess for scapholunate dissociation and SL ligament injury.

    Answer 2: Bone scans are non-specific studies. It may show increased signal at the scaphoid, but this would not be specific for fracture union/non-union in the early time period.

    Answer 3-4: The amount of metal from MR imaging would not provide reliable findings of fracture union. However, MRI is considered the best imaging study to assess for nondisplaced, acute fractures when radiographs and CT images are negative.

     

     

     

     

     

     

     

     

     

  119. Closed rupture of the flexor tendons of the small finger has been reported secondary to:

    1. Internal fixation of a distal radius fracture with a volar plate

    2. Closed treatment of a distal radius fracture

    3. A large volar wrist ganglion

    4. Spurring of the scaphoid

    5. Hook of hamate nonunion

      CORRECT ANSWER: 5

      Nonunion of a hook of hamate fracture has been shown to lead to rupture of the flexor tendons to the small finger.

       

      Displaced hamate hook fractures are optimally treated with early excision to avoid sequelae such as flexor tendon and nerve injury, and to allow early return to activity. Nonunions are often a result of missed injuries, failure of nonoperative treatment, or failure of attempted ORIF. The hook acts as a fulcrum for the ulnar digital flexor tendons and it is believed that excision of the hook may lead to loss of grip strength, though this remains controversial. The proximity of the flexor tendons to the hook may also explain the reported occurrence of closed ruptures of these tendons in the presence of a nonunion of the hook.

       

      Yamazaki et al. present a case series of 6 patients with hook of hamate fractures that went on to nonunion and subsequently suffered closed ruptures of the flexor tendons to the small finger. They compared morphology of the fractures and found even small, rounded fracture fragments can lead to tendon rupture.

       

      Klausmeyer et al. provide a review of the treatment of hook of hamate fractures. They note that treatment of a hook of hamate fracture or nonunion is based on a reported 11% loss of flexor tendon excursion and case series of athletes who could not return to sport following closed treatment alone.

      Published series of open reduction internal fixation or excision of hook of hamate fractures both showed good results and return to sport.

       

      Illustration A is a CT image of a nonunion of a hook of hamate fracture. Illustration B is an intraoperative photograph of a ruptured flexor tendon about a hook of hamate fracture nonunion.

       

      Incorrect answers:

      Answer 1: Internal fixation of a distal radius fracture with a volar plate has been shown to potentially lead to rupture of the flexor pollicis longus, particularly when the plate placed distal to the watershed line.

      Answer 2: Closed treatment of distal radius fractures has been shown to potentially lead to closed rupture of the extensor pollicis longus tendon. Answer 3: Volar wrist ganglions are not known to lead to closed rupture of any flexor tendons.

      Answer 4: Spurring of the scaphoid has been associated with rupture of the

      radial flexor tendons.

       

       

       

       

       

       

  120. A 42-year-old woman presents with sudden left arm pain, cyanosis and paresthesia. Examination reveals mildly diminished grip strength, intact sensation over all digital pulps. The brachial, radial and ulnar pulses are not palpable, although Doppler examination detects a monophasic waveform. Treatment for the most likely diagnosis may include all of the following EXCEPT

    1. Aortic root replacement

    2. Intra-arterial thrombolysis

    3. Angioplasty

    4. Manual aspiration

    5. Stenting CORRECT ANSWER: 1

    This patient has non-occlusive acute left subclavian artery thrombosis (absent pulses but detectable doppler). Treatment can include percutaneous approaches (Answers 2-5). Aortic root replacement is not indicated for subclavian arterial disease alone. It is however indicated for ascending thoracic aortic dissection.

     

    Acute subclavian artery thrombosis is a rare cause of upper limb ischemia. Risks include aortic arch abnormality, aortic arch syndrome, trauma, arterial catheterization, pre-existing atheroma/stenosis at the subclavian artery ostium. Patterns include slow occlusion (atherosclerosis), blue finger syndrome (repeated focal microemboli) and acute global ischemia (large emboli).The diagnosis is made by conventional angiography, which has the added advantage of allowing percutaneous treatment. Treatment strategies include

    (1) medical therapy (heparin, aspirin, clopidogrel, IV thrombolysis, glycoprotein IIb/IIIa antagonists [abciximab, eptifibatide, tirofiban]), (2) open embolectomy and bypass grafting, and (3) percutaneous endovascular procedures.

     

    Pellerin et al. describe 2 cases of percutaneous endovascular management of acute left subclavian artery thrombosis. The 1st case was treated with thrombolysis, stenting and manual aspiration of the thrombus. The 2nd case was treated with anticoagulation, stenting, and long term anticoagulation.

     

    Rapp et al. reviewed patients with proximal lesions causing upper extremity ischemia. Fourteen had extrinsic compression, 17 had brachiocephalic atherosclerosis, 1 had an occluded graft, 1 had Takayasu's arteritis, 1 had radiation atherosclerosis. Some patients had ipsilateral vertebral arterial reversal of flow with neurologic symptoms characteristic of VERTEBROBASILAR insufficiency. Patients with occlusion had arterial reconstruction, bypass and endarterectomy. Five required amputation, 4 because of delayed diagnosis.

    They concluded that these lesions should not be overlooked as consequences are devastating.

     

    Illustration A shows acute non-occlusive left subclavian artery thrombosis. Panel A shows non-occlusive left subclavian artery thrombosis on aortic arch angiography (black arrow). Panel B shows the dilatation of the 2 self-expandable stents (white arrow). There is a protection balloon (black arrow) occluding the left vertebral artery preventing vertebral artery embolism. Panel C shows final aortic arch angiography demonstrating the crushed thrombus between the stent and the arterial wall (black arrow).

     

    Incorrect Answers:

    Answers 2-5: Percutaneous approaches may include thrombolysis, stenting, percutaneous transluminal ballon angioplasty, and manual aspiration.

     

     

     

     

     

     

  121. Which of the following carpal pathologies would be best treated with medial femoral condyle vascularized bone grafting and open fixation?

    1. Chronic lunate osteochondral defect

    2. Acute scaphoid fracture with humpback deformity

    3. Proximal scaphoid avascular necrosis

    4. Scaphoid nonunion advanced collapse with scaphocapitate arthrosis

    5. Stage I Kienböck’s disease

    CORRECT ANSWER: 3

    Out of the available options, vascularized medial femoral condyle graft and fixation is most commonly used in the treatment of proximal scaphoid avascular necrosis and scaphoid nonunion.

     

    Free vascularized bone grafts (VBGs) are often considered in the management of carpal bone nonunion. Various donor sites exist including the distal radius, iliac crest, ribs and femoral condyles. The benefits of VBG techniques include increased bone perfusion over time, accelerated graft consolidation, and rapid repopulation by cells. However, a relative contraindication for their use is in the setting of carpal bones without an intact cartilaginous shell, or advanced carpal collapse with degenerative changes.

    Al-Jabri et al. reviewed the use of the free vascularised bone graft for nonunion of the scaphoid. They noted that the union rate in medial femoral condyle VBGs to be 100% in 56 patients. They state the descending genicular vessels are most favoured due to their longer nature and their wider caliber.

     

    Jones et al. reported on the treatment of scaphoid waist nonunions with associated avascular proximal pole and carpal collapse, with a medial femoral condyle bone grafting. They showed better clinical results with medial femoral condyle bone grafting compared with distal radius VBGs.

     

    Illustration A shows vascularized medial femoral condyle graft harvesting using the superomedial geniculate artery pedicle.

     

    Incorrect Answers:

    Answer 1: Chronic lunate osteochondral defect may be treated non-operatively with observation only or arthroscopic osteochondral grafting with graft harvested from the lateral femoral condyle.

    Answer 2: Acute scaphoid fractures with humpback deformity rarely require free vascularized bone grafts with initial treatment.

    Answer 4: Scaphoid nonunion advanced collapse with scaphocapitate arthrosis is a relative contraindication for free vascularized bone grafts and wrist salvage procedures. Pan carpal arthritis is usually managed with arthrodesis. Answer 5: Stage I Kienböck’s disease does not require open fixation techniques. VBGs can be utilized for stage II disease and potentially for stage III(a) disease. The most commonly used VBG for this disease is based on the fourth and fifth extensor compartmental arteries (4 + 5 ECA), although many other techniques have been described.

     

     

     

     

     

     

  122. A 34-year-old man sustains a finger flexor tendon laceration and undergoes operative repair. Which of the following statements best describes the tendon motion rehabilitation protocol as depicted in Figures A where the splint holds the wrist at 45 degrees of flexion?

     

     

     

    1. Low force and low excursion

    2. Moderate force and potentially high tendon excursion

    3. Low force and high tendon excursion

    4. High force and high tendon excursion

    5. High force and low tendon excursion

    CORRECT ANSWER: 1

    The rehabilitation protocol depicted in Figure A is the Kleinert protocol which is categorized as a low force and low excursion rehabiliation. This uses a dorsal blocking splint with the wrist in 45° of flexion and elastic bands secured to the patient’s nails and a more proximal point on the splint. Once the interphalangeal (IP) joints are actively fully extended, recoil of the elastic bands flexes them down passively. The Duran protocol (Illustration A) is similar but the wrist is in 20° of flexion and relies on the patient to alternately passively extend the DIP and PIP joints with the other joints of the finger flexed. Early active motion protocols that include "place and hold" finger exercises are considered moderate force and potentially high excursion protocols.

     

    The review article by Lilly and Messer reports that synergistic motion protocols are low force and high tendon excursion and are the best at minimizing peritendonous adhesions. In this splint, passive digit flexion is combined with active wrist extension, followed by active digit extension coupled with active wrist flexion (Illustration B and C).

     

     

     

     

     

     

     

     

     

     

  123. Figure A is an arthroscopic image of a right wrist taken from the 3-4 portal. Which of the following correctly identifies the structures marked 1, 2, and 3?

     

     

     

     

    1. Radioscaphocapitate, long radiolunate, short radiolunate

    2. Radioscaphocapitate, short radiolunate, long radiolunate

    3. Long radiolunate, short radiolunate, Radioscaphocapitate

    4. Radiotriquetral, radiolunate, radioscaphiod

    5. Radioscaphoid, radiolunate, radiotriquetral

    CORRECT ANSWER: 1

    The three identified structures are the volar extrinsic ligaments of the wrist from radial to ulnar: radioscaphocapitate, long radiolunate, short radiolunate.

     

    During wrist arthroscopy the 3-4 portal is commonly used as a viewing portal. The portal is placed between the 3rd and 4th dorsal compartments (between the EPL and the EDC). Through this portal you can clearly visualize the volar extrinsic ligaments, which are the (from radial to ulnar) radioscaphocapitate, long radiolunate, and short radiolunate. When looking volarly one can also visualize the ligament of Testut, or the radioscapholunate ligament. It can be seen just radial to the scapholunate ligament and is considered a vestigial structure or vascular conduit rather than a true structural ligament.

     

    Bettinger et al. provide a review of arthroscopic anatomy of the wrist with a focus on what structures can be observed and evaluated during the procedure from radiocarpal and midcarpal portals.

    Berger provides a review of pertinent arthroscopic anatomy of the wrist. It starts with surface anatomy and landmarks, and goes on to describe arthroscopic views as well. It focuses on a systematic approach to a thorough evaluation of the wrist.

     

    In another review by Berger he describes the ligamentous anatomy of the wrist. He notes that the volar extrinsic ligaments from radial to ulnar are the radioscaphocapitate, long radiolunate, and short radiolunate ligaments. There are also volar ulnocarpal ligaments. He also describes the distal radioulnar joint and the triangular fibrocartilaginous complex.

     

    Figure A is an arthroscopic view of the volar extrinsic ligaments of the wrist. They are labeled 1-3 from radial to ulnar.

     

    Incorrect answers:

    Answers 2 and 3: These are not the proper order of the volar extrinsic ligaments of the wrist.

    Answers 4 and 5: These are DORSAL extrinsic ligaments.

     

     

     

  124. Which of the following images seen in figures A-E represents a "preaxial" deformity?

     

     

     

     

     

     

     

     

     

     

     

     

     

     

    1. Figure A

    2. Figure B

    3. Figure C

    4. Figure D

    5. Figure E CORRECT ANSWER: 5

    Figure E represents a preaxial polydactyly.

     

    Polydactyly about the hand can be classified as preaxial, postaxial, or central. Polydactyly about the radial side of the hand, such as a duplicate thumb is "preaxial." Polydactyly about the ulnar side of the hand, such as a duplicate small finger is "postaxial." Polydactyly about the central rays of the hand, commonly associated with syndactyly are "central." Preaxial polydactyly is commonly classified according to Wassel (See illustration A). Treatment varies depending on Wassel type, and typically includes excision of the smaller thumb with potential transfer or reconstruction of ligamentous and tendinous structures.

     

    Guo et al. present a review of polydactyly of the hand. He notes that preaxial

    polydactyly is the most common form (about 90% of all polydactylies) and occurs most often in Caucasians. He also notes that while it is normally an isolated anomaly, the presence of triphalangia does have a higher incidence with other disorders such as Holt-Oram and Fanconi Anemia.

     

    Al-Qattan presents a review of preaxial polydactyly. He notes that triphalangeal thumbs, preaxial polysyndactyly, and mirror hand deformities are all on the spectrum of "preaxial" anomalies. These occur due to an error in embryological development that disrupts the normal formation of the limb bud in the anteroposterior axis.

     

    Figure A depicts a "central" polysyndactyly. Figure B represents constriction band syndrome. Figure C represents macrodactyly. Figured D represents postaxial polydactyly. Figure E is a radiograph demonstrating a triphalangeal thumb. Illustration A illustrates the Wassel classification of preaxial polydactyly.

     

    Incorrect Answers:

    Answers A-D: These images do not represent preaxial polydactyly.

     

     

     

     

     

     

  125. A patient with skier's thumb presents with persistent pain and instability for 3 months. Examination reveals MCP laxity in 30° flexion, but not in extension. MRI demonstrates a Stener lesion. Which of the following is most likely true regarding this patients condition?

    1. The proper ulnar collateral ligament (UCL) is torn off the proximal phalanx.

    2. The proper UCL is torn off the metacarpal.

    3. The proper UCL and accessory UCL are torn off the proximal phalanx.

    4. The proper UCL and accessory UCL are torn off the metacarpal.

    5. The accessory UCL is torn off the proximal phalanx.

     

    CorreCt answer: 1

    If there is ulnar collateral laxity in flexion only, it implies isolated injury to the proper UCL, and the accessory UCL is intact. With a Stener lesion, the UCL is avulsed from its distal proximal phalanx (P1) attachment and trapped over the adductor aponeurosis.

     

    Skier's thumb is the most common upper extremity injury during skiing (7-32% prevalence) and arises from a hyperabduction load. Static stabilizers include proper UCL, accessory UCL, shape of the joint, dorsal capsule and volar plate. The adductor pollicis is the dynamic stabilizer. Most ruptures involve the DISTAL end. A Stener lesion occurs in 65-85% of all ruptures and are treated surgically. In flexion, the proper UCL and dorsal capsule are taut. In extension, the accessory UCL and volar plate are taut. Laxity in flexion implies isolated proper UCL rupture. Laxity in both flexion and extension implies both accessory UCL and proper UCL are ruptured. Swelling at the MCP suggests a Stener lesion, but is not specific.

     

    Bean et al. explored the effect of changing the proper UCL attachment site on range of motion in cadavers. For the metacarpal origin, they found that radial deviation increased with palmar placement and decreased with proximal placement. For the phalangeal insertion, they found that radial deviation increased with dorsal placement, and decreased with distal placement. Range of motion was decreased with both distal and palmar placements. They concluded that nonanatomic reconstruction alters normal MCP ROM and the insertion and origin landmarks should serve as guides for UCL repairs.

     

    Carlson et al. explored the attachment sites of the UCL and RCL of the thumb MCPJ. They found that the UCL metacarpal insertion was 4.2 mm and 5.3 mm from the dorsal and articular surfaces, respectively. The phalangeal insertion was 2.8 mm and 3.4 mm from the volar surface and articular surfaces, respectively. The RCL metacarpal origin was 3.5 mm and 3.3 mm from the dorsal and articular surfaces, respectively. The phalangeal insertion was 2.8 mm and 2.6 mm from the volar and articular surfaces, respectively. They concluded that accurate definition of the anatomical attachment sites will aid surgical repair.

     

    Illustration A shows the arrangement of the proper UCL and accessory UCL components of the UCL of the thumb MCP. Illustration B shows a bony UCL avulsion injury.

     

    Incorrect Answers:

    Answers 2-5: This is an isolated proper UCL injury. The Stener lesion is an avulsion of the distal (phalangeal) attachment.

     

     

     

     

     

     

     

     

     

  126. A healthy 12-year-old girl presents with right elbow pain after a fall off her bicycle 3-days ago. She has full, painless flexion and extension of the elbow with 30-degrees of fixed forearm pronation. Radiographs are shown in Figures A and B. Which of the following statements is true regarding this patient's condition?

     

     

     

     

     

     

    1. Caused by trauma

    2. Caused by failure of formation

    3. Caused by failure of segmentation

    4. Associated with transverse deficiency of the forearm

    5. Always requires surgical intervention with osteotomy and fusion

    CORRECT ANSWER: 3

    The patient has congenital proximal radioulnar synostosis (RUS), which is caused by a failure of segmentation.

     

    The proximal ends of the radius and ulna are temporarily united during elbow formation and share a common perichondrium. Congenital radioulnar synostosis is a primary developmental anomaly of radioulnar differentiation and segmentation in the 7th week of gestation. Familial cases of RUS tend to follow autosomal dominant inheritance and is bilateral in approximately 60% of cases. RUS has also been found to be associated with chromosomal abnormalities, most commonly sex chromosome duplications. The majority of patients can be treated nonoperatively. Patients with bilateral RUS and those with a pronation deformity of >60 degrees may be candidates for osteotomy and fusion.

     

    Elliot et al performed a retrospective cohort study evaluating patients with proximal radioulnar fusion and posterior radial head dislocations. The authors found suggestion of a developmental relationship between posterior dislocation of the radial head and proximal radioulnar fusion as supported by the fact that both anomalies were found to occur in the same patient. The anomalies are related primary developmental anomalies of radioulnar differentiation and segmentation.

     

    Cleary et al evaluated the natural history and functional status of 23 patients with congenital proximal radio-ulnar synostoses. The forearms were fixed at an average of 30 degrees of pronation and the position of the forearm was not related to subjective functional limitations, employment status or result of objective tests. The authors concluded that operative treatment is rarely indicated and more emphasis should be placed on objective functional tests rather than the position of the forearm.

     

    Figures A and B are AP and Lateral views of the right elbow demonstrating proximal radioulnar synostosis.

     

    Incorrect Answers:

    Answer 1: Although proximal radio-ulnar synostosis can be post-traumatic, the patient reports an acute history of trauma, which is unlikely to result in her presentation of synostosis.

    Answer 2: RUS is a result of failure of segmentation.

    Answer 4: RUS is associated with posterior dislocation of the radial head. Transverse deficiency of the forearm is a distinct entity.

    Answer 5: RUS rarely requires surgical intervention.

     

     

  127. A 22-year-old athlete presents with the injury shown in Figure A. What would be the recommended treatment?

     

     

     

     

    1. Observation only

    2. Removable wrist splint

    3. Long arm cast

    4. Open reduction internal fixation through a volar approach

    5. Open reduction internal fixation through a dorsal approach

    CORRECT ANSWER: 5

    This patient presents with a proximal scaphoid fracture. The most appropriate treatment would be open reduction internal fixation (ORIF) through a dorsal approach.

     

    ORIF is indicated in almost all cases of scaphoid proximal pole fractures as these is a high incidence of avascular necrosis (AVN) and non-union with these fracture patterns. Literature suggests AVN rates to be of 100% with proximal 5th fractures and and 33% with proximal 3rd frature.

     

    Raskin et al. disccused the dorsal open repair of proximal pole scaphoid

    fractures. They state that the ideal approach to manage these fractures is a 3-4 cm dorsal incision centered over Lister’s tubercle. Care should be taken to preserve the vessels around the dorsal ridge of the scaphoid, by avoiding capsule stripping in this area.

     

    Rettig et al. report their their outcomes with ORIF of displaced scaphoid waist fractures. Using internal fixation with K-wires or Herbert screws, 13 of the 14 (93%) fractures united with an average time to union of 11.5 weeks. They state that ORIF of acute displaced scaphoid waist fractures restores scaphoid alignment and leads to predictable union.

     

    Rettig et al. looked at retrograde screw fixation of acute unstable proximal pole scaphoid fractures for 17 patients. The operative technique consisted of a dorsal approach to the scaphoid, radius bone grafting, and freehand retrograde Herbert compression screw fixation. All fractures healed within 13 weeks (average, 10 weeks). No patients developed osteonecrosis or radioscaphoid arthritis.

    Figure A shows a minimally displaced proximal 1/3 scaphoid fracture. Incorrect Answers:

    Answers 1-3: Non-operative treatment of proximal scaphoid fractures have

    shown to have a high incidence of AVN and non-union. Operative fixation is preferred, especially in young athletes.

    Answer 4: Displaced proximal pole fractures require a dorsal approach because accurate placement of the screw has been shown to be more reliable with this approach.

     

     

     

  128. A 23-year-old football player falls on his right hand during a game. Figure A is an injury radiograph of his right hand. Figure B shows the main ligamentous restraints, ligaments 1 through 4 (L1 through L4). Which of the following is the MOST LIKELY scenario?

     

     

     

     

     

     

    1. This is most likely a volar dislocation. L2 is torn.

    2. This is most likely a volar dislocation. L3 is torn.

    3. This is most likely a dorsal dislocation. L2 is torn.

    4. This is most likely a dorsal dislocation. L3 is torn.

    5. This is most likely a dorsal dislocation. Both L2 and L3 are torn.

     

    CorreCt answer: 4

    This is a dorsal dislocation of the thumb CMC joint. The dorsoradial ligament (DRL, L3) is torn. The anterior oblique ligament (AOL, L2) remains intact but peels off the base of the 1st metacarpal (MC).

     

    Thumb CMC dislocation is rare. Dorsal dislocations are more common than volar dislocations. The mechanism for dorsal dislocation is axial loading with thumb MC flexed. (Another possible mechanism is a force driven into the 1st web e.g. handlebar driven into motorcyclist’s thumb on impact). The primary restraint to dorsal dislocation is the DRL. The AOL remains continuous is but peeled off the 1st metacarpal base. In contrast, in a Bennett/Rolando fracture, the AOL remains attached to a fragment of bone at the MC base and the MC translates because of the fracture through the base.

     

    Strauch et al. reviewed acute CMC dislocations in cadavers. They applied a dorsal dislocating force and progressively sectioned ligaments. They found that the primary restraint was the DRL with the AOL allowing dislocation after peeling off the base of the 1st metacarpal. The joint was more stable in pronation and extension which tightened the anterior oblique ligament.

     

    Bettinger and Berger reviewed the functional anatomy of the trapezium and TMCJ. They describe 16 ligaments that help stabilize the CMCJ and use geometric and mathematical models to describe and support ligament function. They state that the dorsoradial ligament is an important stabilizer against dorsal forces. The trapeziotrapezoid, trapezio-II metacarpal and trapezio-III metacarpal ligaments function as tension bands to support the trapezium against cantilever bending forces.

     

    Figure A shows a dorsal dislocation of the thumb CMC joint. Figure B shows the main ligamentous restraints of the CMC joint. L1 is the intermetacarpal ligament, L2 is the AOL, L3 is the DRL, L4 is the posterior oblique ligament. Illustration A shows dislocation of a cadaver thumb. Panel A shows the AOL (black arrow) and DRL (white arrow). With sectioning of the DRL (Panel B) and a dorsal directed force, the AOL strips of the 1st MC base (Panels C and D).

    Illustration B shows how both DRL and AOL are of equal laxity when reduced, but with dorsal subluxation, the DRL tightens and the AOL remains lax. Of note, the MC base is 34% wider than the trapezium (represented by spheres of differing sizes), which also contributes to differential laxity with dorsal translation.

     

    Incorrect Answers:

    Answers 1 and 2: This is a dorsal dislocation

    Answers 3 and 5: The AOL (L3) remains in continuity but is peeled/stripped off the 1st MC base.

     

     

     

     

     

     

     

     

     

  129. Demyelination of a nerve fiber will result in which of the following on a nerve conduction velocity study (NCS)?

    1. Increased amplitude

    2. Increased latency

    3. Normal evoked reponse

    4. No change in the conduction velocity

    5. No change in latency CORRECT ANSWER: 2

    Demyelination of a nerve fiber results in increased latency on a nerve conduction velocity test.

     

    A nerve conduction velocity test is a measurement of the speed of conduction of an electrical impulse through a large, myelinated nerve. In this test, evoked responses are measured after stimulation of a peripheral nerve. Typical measurements include the nerve conduction velocity, amplitude, and duration. When a nerve becomes demyelinated, or damaged, there is increased latency and overall decrease in the conduction velocity. By analyzing the results, one can differentiate between a normal nerve, demyelination, axonotmesis, neurotmesis, and neuropraxia. Common indications include carpal and cubital tunnel, radiculopathy, and nerve dysfunction of the shoulder. A thicker myelin sheath will affect nerve transmission by increasing the speed of wave propagation.

     

    Wilbourne reports that the electrodiagnostic exam (including nerve conduction studies) detect demyelination injuries along large, myelinated motor axons, as well as the sensory axons for position, vibration, and some light touch.

    Demyelination of the axon results in a block to conduction, focal slowing of signal transmission, and manifests as increased latencies on NCS stimulation proximal to the lesion. Clinically, focal demyelination manifests as decreased deep tendon reflexes, loss of vibrational sense, and intermittent pain and paresthesias. It does not, however, affect motor strength or sensibility.

     

    Illustration A shows a diagrammatic representation of a nerve conduction velocity test.

     

    Incorrect Answers:

    Answer 1: Demyelination results in decreased amplitude.

    Answer 3: Demyelination results in an absent or prolonged evoked response. Answer 4: Demyelination results in a decrease in conduction velocity.

    Answer 5: Demyelination results in increased latency.

     

     

     

     

     

     

  130. An 8-year-old boy's parents are concerned about the appearance of the child's middle finger. The child denies pain and his digital neurovascular status is normal. A clinical photograph and radiograph are provided in figures A and B. For children with this condition, which of the following is the best intervention to achieve a finger that is proportional to the rest of the hand?

     

     

     

     

     

     

    1. Epiphysiodesis now

    2. Epiphysiodesis when the finger reaches adult length of the father

    3. Compression wrapping until proportional size is achieved

    4. Resection of hypertrophic nerves

    5. Osteotomy at skeletal maturity.

     

    CorreCt answer: 2

     

    Clinical photograph and radiographs demonstrate macrodactyly of the middle finger, a rare congenital malformation enlarging all structures of the digit.

     

    Ishida et al reviews 23 cases of surgically treated macrodactyly finding favorable results with epiphysiodesis/epiphysiodectomy while resection of hypertrophic nerves was unsuccessful in preventing overgrowth. The epiphysiodesis is performed once the finger reaches the length of the same sex parent, using their digit as a template for final growth.

     

     

     

  131. Flexor tendons of the fingers within Zone 2 receive their primary nutritional supply from:

    1. Vinculae

    2. Phalangeal periosteum

    3. Musculotendon junction

    4. Tendon insertion

    5. Diffusion from the synovial sheath

      CORRECT ANSWER: 5

      The vascularity of tendon varies depending on the type of tendon (e.g. with or without a sheath) and the location. Sheathed tendons (e.g. flexor tendons of the hand) have a dual blood supply via both vascular perfusion but also have regions that are relatively avascular where they receive nutrition through synovial diffusion. This is the case in zone 2 of the digital flexor tendons where the primary nutritional supply is from synovial diffusion through the parietal paratenon which allows for passive nutrient delivery to the flexor tendon within the sheath. The digital flexor tendons also receive minor direct arterial perfusion in zone 2 through the vinicular system, osseous bony insertions, reflected vessels from the tendon sheath and longitudinal vessels from the palm, but this is not the major blood supply.

       

      Tendons not enclosed by a sheath receive their blood supply directly from vessels entering from the tendon surface or from the tendon-to-bone insertion.

       

       

       

       

       

  132. A 4-year-old child was born with bilateral congenital radial clubhands. Which of the following associated conditions is a contraindication to centralization of the hands on the ulna?

    1. Congenital scoliosis

    2. Hypoplastic thumb

    3. Tracheoesophageal fistula

    4. Imperforate anus

    5. Lack of elbow flexion CORRECT ANSWER: 5

    Patients born with bilateral radial clubhands may have difficulty getting their hands to their mouth. The centralization procedure would take away that ability if there is a lack of elbow flexion.

     

     

     

  133. A 6-year-old child has a fixed flexion deformity of the interphalangeal (IP) joint of the right thumb. The thumb is morphologically normal, with a nontender palpable nodule at the base of the metacarpophalangeal joint. Clinical photographs are shown in Figures 42a and 42b. Based on these findings, what is the treatment of choice?

     

     

     

     

     

     

    1. Complete release of the proximal annular pulley of the flexor sheath

    2. Removal of the nodule in the flexor pollicis longus

    3. Fractional lengthening of the flexor pollicis longus tendon at the musculotendinous junction

    4. Steroid injection into the palpable nodule

    5. No treatment because this condition normally spontaneously resolves

    CORRECT ANSWER: 1

    The child has a trigger thumb deformity. A trigger thumb is a developmental mechanical problem rather than a congenital deformity. The anomaly generally is not noted at birth. A fixed flexion deformity of the IP joint of the thumb most commonly occurs in children in the first 2 years of life. A stretching and splinting program may correct the deformity in the first year of life, but nonsurgical management after age 3 years results in a success rate of only

    50%. Release of the proximal annular pulley of the flexor sheath is recommended at this age.

     

     

     

  134. The attachments of the transverse carpal ligament include which of the following structures?

    1. Scaphoid and the ulna

    2. Trapezium and the hook of the hamate

    3. Trapezium and the triquetrum

    4. Trapezoid and the hook of the hamate

    5. Trapezoid and the pisiform

    CORRECT ANSWER: 2

    The transverse carpal ligament is the volar boundary of the carpal tunnel. It attaches to the scaphoid and trapezium radially and the pisiform and the hook of the hamate ulnarly. The ulna and trapezoid do not receive attachments of the transverse carpal ligament.

     

     

     

  135. A patient undergoes the procedure shown in Figure 19. An important part of this procedure is preservation of what wrist ligament?

     

     

     

    1. Radioscaphocapitate

    2. Scapholunate interosseous

    3. Ulnotriquetral

    4. Volar radioulnar

    5. Deep proximal capitohamate

    CORRECT ANSWER: 1

    Proximal row carpectomy is a salvage wrist procedure that yields a surprisingly stable construct. This has been attributed to two factors: 1) the congruency of the head of the capitate in the lunate fossa (this articulation is less congruent than the native lunate/lunate fossa relationship, but surprisingly stable), and

    2) preservation of the radioscaphocapitate ligament, the most radial of the palmar extrinsic ligaments, which prevents ulnar subluxation after proximal row carpectomy.

     

     

     

  136. The arrows in the axial T1-weighted MRI scan shown in Figure 25 show which of the following structures?

     

     

     

    1. Ulnar artery and accompanying vein

    2. Deep and superficial branches of the ulnar nerve

    3. Radial and ulnar digital nerves to the little finger

    4. Palmar cutaneous and thenar motor branch of the median nerve

    5. Dorsal cutaneous branch of the ulnar nerve and common digital artery to the fourth web

    CorreCt answer: 2

     

    The arrows in the figure show the deep branch of the ulnar nerve (more radial) and the superficial branch of the ulnar nerve within Guyon’s canal. Guyon’s canal is approximately 4 cm long beginning at the proximal extent of the transverse carpal ligament and ends at the aponeurotic arch of the hypothenar muscles. Many structures comprise the boundaries of Guyon’s canal. The floor, for example, consists of the transverse carpal ligament, the pisohamate and pisometacarpal ligaments, and the opponens digiti minimi. Within Guyon’s canal, the ulnar nerve bifurcates into the superficial and deep branches. The ulnar artery is immediately adjacent and radial to the ulnar nerve. The median nerve is visualized within the carpal tunnel, and the palmar cutaneous branch is more radial to Guyon’s canal and volar to the carpal tunnel. The radial and ulnar digital nerves to the little finger are branches off of the superficial branch of the ulnar nerve distal to its emergence from Guyon’s canal. The ulnar artery is the round structure located radial to the branches of the ulnar nerve within Guyon’s canal. Adjacent to the ulnar artery are two small veins. The dorsal cutaneous branch of the ulnar nerve branches from the ulnar nerve in the distal forearm, well proximal to Guyon’s canal. The common digital artery to the fourth web branches from the superficial palmar arch distal to Guyon’s canal. The hook of the hamate is clearly seen in the figure, orienting the observer to the ulnar side of the wrist

     

     

     

     

     

     

  137. A 37-year-old patient with type I diabetes mellitus has a flexor tenosynovitis of the thumb flexor tendon sheath following a kitchen knife puncture wound to the volar aspect of the thumb. Left unattended, this infection will likely first spread proximally creating an abscess in which of the following spaces of the palm?

    1. Central space

    2. Hypothenar space

    3. Carpal tunnel

    4. Posterior adductor space

    5. Thenar space CORRECT ANSWER: 5

    Flexor tenosynovitis of the thumb flexor tendon sheath can spread proximally and form an abscess within the thenar space of the palm. The flexor pollicis longus tendon does not pass through the central space of the palm or the hypothenar space of the palm. The flexor pollicis longus tendon does pass through the carpal tunnel, but this is not a palmar space. The three palmar spaces include the hypothenar space, the thenar space, and the central space. The posterior adductor space would likely only be involved secondarily after spread from a thenar space infection.

     

     

     

  138. New painful paresthesias near the site of the incision after an ulnar nerve transposition is the result of injury to what nerve?

    1. Medial antebrachial cutaneous

    2. Lateral antebrachial cutaneous

    3. Posterior antebrachial cutaneous

    4. Medial brachial cutaneous

    5. Dorsal antebrachial cutaneous

    CORRECT ANSWER: 1

    Branches of the medial antebrachial cutaneous nerve can often be identified during routine ulnar nerve surgery crossing the medial aspect of the elbow. It should be preserved to avoid development of painful paresthesias.

     

     

     

  139. A 21-year-old man who was injured in a snowboarding accident 18 months ago now reports wrist pain. An MRI scan is shown in Figure

    37. Based on the image findings, what is the most likely diagnosis?

     

     

     

     

    1. Preiser’s disease

    2. Scaphoid nonunion and osteonecrosis

    3. Kienbock’s disease

    4. Intraosseous ganglion

    5. Scapholunate dissociation

    CORRECT ANSWER: 2

    The coronal MRI scan of the wrist shows the scaphoid. There is a subtle fracture line with a step-off at the radial surface consistent with a nonunion. The signal intensity is markedly different between the two fragments of the scaphoid. This strongly suggests osteonecrosis. Preiser’s disease is osteonecrosis typically involving most or all of the scaphoid. Kienbock’s disease involves the lunate. Intraosseous ganglia are easily diagnosed on MRI but typically have a fluid-filled area surrounded by denser bone in the periphery. Scapholunate dissociation can be seen on MRI as an injury to the scapholunate ligament and widening of the scapholunate interval, neither of which is seen on this image.

     

     

     

  140. Which of the following describes the correct proximal to distal progression of the annular and cruciform pulleys of the digits?

    1. A1, C1, A2, C2, A3, A4, C3

    2. A1, A2, A3, C1, C2, C3, A4

    3. A1, C1, C2, A2, A3, A4, C3

    4. A1, A2, C1, A3, C2, A4, C3

    5. A1, A2, A3, A4, C1, C2, C3

     

    CorreCt answer: 4

     

    The correct progression of the annular and cruciform pulley in the digits is A1, A2, C1, A3, C2, A4, C3. The two cruciform pulleys are collapsible elements adjacent to the more rigid annular pulleys of the flexor tendon sheath. This arrangement enables unrestricted flexion of the proximal interphalangeal joint.

     

     

     

  141. In Dupuytren’s disease, the retrovascular cord typically displaces the radial proper digital nerve of the ring finger in what direction?

    1. Palmarly and radially

    2. Dorsally and ulnarly

    3. Palmarly and ulnarly

    4. Dorsally and radially

    5. Directly dorsal CORRECT ANSWER: 3

    Retrovascular cords are common in Dupuytren’s disease and commonly require surgical treatment. Nerve injury in Dupuytren’s surgery is an

    infrequent complication that occurs partly because the digital nerves can be displaced from their normal anatomic relationships by retrovascular cords. The nerves are displaced superficially, toward the center of the digit (palmarly and ulnarly). This displacement is typically seen at the level of the metacarpophalangeal joint.

     

     

     

  142. Ganglion cysts about the wrist most commonly arise from what structure?

    1. First carpometacarpal joint

    2. Second carpometacarpal joint

    3. Scapholunate interosseous ligament

    4. Radioscaphocapitate ligament

    5. Capitohamate interosseous ligament

    CORRECT ANSWER: 3

    Ganglion cysts are the most common mass or mass-like lesions seen in the hand and wrist. They arise in a variety of locations, including synovial joints or tendon sheaths. The most common location is the dorsal/radial wrist arising from the dorsal scapholunate interosseous ligament.

     

     

     

  143. Spontaneous entrapment of the posterior interosseous nerve most commonly occurs in which of the following locations?

    1. Lateral intermuscular septum

    2. Extensor carpi radialis brevis

    3. Arcade of Frohse

    4. Midsubstance of the supinator

    5. Leash of Henry CORRECT ANSWER: 3

    The extensor carpi radialis brevis, supinator muscle, arcade of Frohse, and leash of Henry are potential sites of compression for the posterior interosseous nerve. The most common location of spontaneous entrapment is the arcade of Frohse. The lateral intermuscular septum is a site of compression for the radial nerve.

     

     

  144. A 45-year-old carpenter complains of difficult gripping a hammer, which worsens with repeated use. On physical exam, he is able to passively flex the proximal interphalangeal (PIP) joint when the metacarpophalangeal (MCP) joint is flexed but not when the MCP joint is extended. What is the most likely explanation?

    1. Quadrigia effect

    2. Extrinsic tightness

    3. Intrinsic tightness

    4. Central slip rupture

    5. Lumbrical plus CORRECT ANSWER: 3

    The patient exhibits a positive Bunnell test, indicative of intrinsic tightness, which commonly affects those who use objects that require repetitive grip.

     

    Bunnell (intrinsic tightness) test is performed and is positive when there is a loss of PIP flexion when the MCP joint is extended; with the MCP flexed, PIP motion is typically normal.

     

    Espiritu et al. performed a cadaveric study to quantitatively assess the amount of release needed to overcome intrinsic tightness. Index, middle, ring, and little fingers regained flexion following release of 59%, 65%, 26%, and 33% of each extensor hood, respectively.

     

    Lee and Gellman review their preferred methods of operative treatment of specific intrinsic deformities caused by intrinsic tightness.

     

    Video 1. A clinical example on how to perform the Bunnell test for intrinsic tightness.

     

    Incorrect answers:

    Answer 1. Quadrigia effect is characterized by an active flexion lag in fingers adjacent to a digit with a previously injured or repaired flexor digitorum profundus tendon.

    Answer 2. Extrinsic tightness is limited MCP flexion secondary extensor tendon adhesion to bone.

    Answer 4. Central slip rupture would result in a Boutonniere deformity. Answer 5. Lumbrical plus is paradoxical PIP extension during active finger flexion.

     

     

  145. A 33-year-old woman reports a mass on the right hand that has been enlarging for 1 year. An intraoperative photograph is shown in Figure 28a, and a biopsy specimen is shown in Figure 28b. What is the most likely diagnosis?

     

     

     

     

     

     

     

    1. Ganglion cyst

    2. Abscess

    3. Hematoma

    4. Giant cell tumor of tendon sheath

    5. Synovial sarcoma CORRECT ANSWER: 4

    Giant cell tumor of the tendon sheath is the most common solid soft-tissue mass in the hand. These tumors are slow-growing and may be present for months or years before coming to medical attention. Patients usually report

    mechanical difficulties because of the size or position of the tumor. The gross appearance is that of a lobulated mass that may be multicolored; typically yellow, brown, red, and gray. Histologically the lesion consists of multinucleated giant cells, polygonal mononuclear cells, and histiocytes that may contain abundant hemosiderin or lipid.

     

     

     

  146. A 58-year-old woman has had a slowly progressing mass over the distal interphalangeal (DIP) joint of her dominant hand with a worsening deformity of her nail. She has no significant medical history but underwent bilateral knee arthroplasties 1 year ago. Radiographs reveal a small osteophyte at the DIP joint dorsally. A clinical photograph and a biopsy specimen are shown in Figures 76a and 76b. What is the most likely diagnosis?

     

     

     

     

     

     

     

    1. Metastatic lung carcinoma

    2. Mucous cyst

    3. Synovial sarcoma

    4. Inclusion cyst

    5. Felon abscess CORRECT ANSWER: 2

    A mucous cyst is thought to be a ganglion arising from the DIP joint in patients with osteoarthritis. They are frequently associated with nail deformities.

    Treatment involves removal of the cyst with debridement of DIP joint osteophytes.

     

     

     

  147. A patient has carpal tunnel syndrome that requires surgical release. The patient wants to know about the risks and benefits of the various surgical techniques for carpal tunnel release because they have read that an endoscopic technique is the best. What is the most significant advantage of endoscopic release over traditional open release?

    1. Decreased rate of nerve complications

    2. Decreased postoperative pain and analgesic use

    3. Decreased rate of recurrence of carpal tunnel symptoms

    4. Lower rate of vascular complications

    5. Lower infection rate CORRECT ANSWER: 2

    Carpal tunnel release is usually performed in one of two ways: open or endoscopically. There has always been great controversy and differing opinions about the advantages and disadvantages of either technique. Recent prospective randomized trials have shown that the primary advantage of the endoscopic technique is decreased pain in the postoperative period. A lower rate of vascular complications was not observed in these most recent studies. Likewise, nerve injury and infection rates were not different in these studies.

     

     

     

  148. All of the following structures are components of the triangular fibrocartilage complex. What structure is considered the most important restraint of dorsal/palmar translation of the radius on the ulna?

    1. Triangular articular disk

    2. Dorsal and palmar radioulnar ligaments

    3. Ulnolunate ligament

    4. Ulnotriquetral ligament

    5. Subsheath of the sixth extensor compartment

    CORRECT ANSWER: 2

    Clinical examination of the distal radioulnar joint is frequently described as translation of the ulnar head in reference to the radius. This description is incorrect, since the ulna is the bone that is fixed in space. Anatomically speaking, the radius moves in relation to the fixed ulna. The triangular fibrocartilage complex is composed of all of the structures listed above. The articular disk is a meniscal-like structure that serves a load-bearing function between the ulnar carpal bones and the ulnar head. About 20% of the load borne across the wrist passes through the disk. It has almost no ligamentous (stabilizing) function. At the dorsal and volar margins of the articular disk are thickened true ligamentous ligaments termed the radioulnar ligaments. They take origin from the foveal area of the ulnar head and styloid and insert into the dorsal and volar margins of the sigmoid notch. These two ligaments serve as primary restraints to dorsal and palmar translation of the radius on the ulna. The ulnolunate ligament, ulnotriquetral ligament, and the subsheath of the sixth extensor compartment are ligaments, but they serve to stabilize the carpus to the ulna and radius. They provide minimal stability to the distal radioulnar joint.

     

     

     

  149. Replantation is indicated for which of the following amputations.

     

    1. multiple level amputation of the small finger at the proximal and distal interphalangeal joints

    2. crushed amputation through the distal phalanx of the middle finger

    3. complete ring avulsion of the ring finger with tendon injury

    4. thumb amputation though the proximal phalanx shaft

    5. middle finger amputation though the proximal phalanx shaft

    CORRECT ANSWER: 4

    As outlined by Pederson, the contraindications to replantation are more relative than the indications, but they include the following: Single-finger replantations at the level of zone II (from the A1 pulley to the distal sublimis tendon insertion) are rarely indicated, with the notable exception of the thumb. Amputated parts that are severely crushed and those with multiple

    level injuries have poor function even if they survive replantation. While ring avulsion injuries with a vascular injury and no bone, tendon or nerve injury (Urbaniak type 2A ring avulsion injuries) should be repaired, ring avulsion injuries with bone, tendon or nerve injury (Urbaniak type 2B) or with complete degloving (Urbaniak type 3) have poor outcomes and Urbaniak and colleagues recommend amputation for such injuries. Very distal amputations at the level of the nail bed are marginally indicated as there needs to be approximately 4 mm of intact skin proximal to the nailfold for adequate veins to be present.

    Indications for replantation that rule out the other 4 choices of this question include the following: Overall, thumb replantation probably offers the best functional return. Even with poor motion and sensation, the thumb is useful to the patient as a post for opposition. A replanted thumb offers the best reconstruction available, toe transfers notwithstanding. Replantation beyond the level of the sublimis tendon insertion (zone I) usually results in good function. Multiple finger amputations present reconstructive difficulties that may be difficult to correct without replantation of one or all of the amputated digits. Any hand amputation from zone III (distally) to zone V (proximally) offers the chance of reasonable function after replantation, usually superior to available prostheses. Although usually indicated, the replantation of any hand or arm proximal to the level of the mid-forearm must be carefully considered.

     

     

     

  150. Figure 14a shows the radiograph of a 19-year-old man who hurt his little finger while falling off his bicycle. A closed reduction of the proximal interphalangeal (PIP) joint is performed in the emergency department. Postreduction radiographs are shown in Figures 14b and 14c. When the PIP joint is put through a range of motion, the middle phalanx redislocates when fully extended. What is the next most appropriate step in management?

     

     

     

    1. Extension block splinting for 3 weeks

    2. Volar plate arthroplasty

    3. Open reduction and internal fixation of the middle phalanx base

    4. Superficialis tendon repair through bone tunnels

    5. Dynamic external fixation

    CORRECT ANSWER:

    1

    The patient sustained a dorsal fracture-dislocation of the PIP joint. This is a very common injury and treatment options are variable. When there is no fracture of the base of the middle phalanx, extension block splinting is the best option. When a fracture is present, the stability of the fracture and the proportion of the base of the middle phalanx that is involved must be evaluated. If the fracture surface is less than 25% of the base and the joint reduces adequately in flexion, then extension block splinting is the best treatment option. When more than 25% of the joint is involved and/or the joint does not reduce completely in flexion, then surgical treatment is indicated. Dynamic external fixation is an effective and less invasive option for moderate injuries. Open repair or volar plate arthroplasty is typically reserved for severe injuries of more than 50% of the joint surface. In this patient, less than 25% of the joint surface is involved. The postreduction radiographs show an excellent congruent reduction. Because the PIP joint redislocates only in full

    extension, a PIP joint extension block splint should be applied (40-degree block) and the patient should be allowed to flex the finger. Over the next 3 weeks, the extension block can be reduced 10 degrees per week.

     

     

     

  151. A 21-year-old man reports wrist pain after a fall. He also reports that he sustained a wrist sprain the previous year. Radiographs reveal a waist scaphoid fracture with a nonunion, humpback deformity, but no evidence of osteonecrosis. What is the best choice for surgical treatment?

    1. Volar approach, standard bone graft, Kirschner wire fixation

    2. Volar approach, standard bone graft, screw fixation

    3. Volar approach, vascularized bone graft, Kirschner wire fixation

    4. Dorsal approach, vascularized bone graft, Kirschner wire fixation

    5. Dorsal approach, standard bone graft, screw fixation

    CORRECT ANSWER: 2

    The patient has a scaphoid nonunion with humpback deformity. Surgical treatment should include correction of the humpback deformity with bone graft. This is best accomplished with a volar approach, nonvascularized bone graft, and screw fixation. Since there is no prior surgical treatment and no evidence of proximal pole osteonecrosis, the use of a vascularized bone graft is not necessary or recommended. Whereas both dorsal and volar approaches have been described, the humpback deformity is most easily corrected via a volar approach. This exposure allows for insertion of Kirschner wire joy sticks to help correct the deformity and allow easy insertion of the bone graft. It also preserves the main dorsal blood supply to the scaphoid. Kirschner wire fixation of the fracture itself has been used in the past, but screw fixation is more secure. Fractures and nonunions close to the proximal pole may be better visualized with a dorsal approach, but correction of any deformity can be more difficult. Vascularized bone grafts are usually indicated for failed nonvascularized bone grafts or in the presence of osteonecrosis.

     

     

     

  152. A 62-year-old woman has long-standing left thumb pain and weakness when grasping objects. Examination reveals a positive grind test and dynamic thumb metacarpophalangeal hyperextension. Prolonged bracing, anti-inflammatories and oral analgesics has failed to relieve her symptoms. Clinical and radiographic images are seen in

    Figures A and B. What would be the best treatment option to relieve this patients pain and improve function?

     

     

     

     

     

     

     

    1. Extension osteotomy of the first metacarpal

    2. Carpometacarpal joint resection arthroplasty

    3. Carpometacarpal joint resection arthroplasty and metacarpophalangeal joint arthrodesis

    4. Carpometacarpal joint arthrodesis and metacarpophalangeal joint arthrodesis

    5. Carpometacarpal joint arthroscopy and debridement +/- ligament reconstruction

    CorreCt answer: 3

     

    This patient has end stage basilar thumb arthritis with thumb metacarpophalangeal hyperextension of 40 degrees. She has failed conservative management. The best treatment would be carpometacarpal joint resection arthroplasty and metacarpophalangeal joint arthrodesis.

     

    The mainstay of treatment for end-stage thumb carpometacarpal arthritis is excision of the trapezium, with or without ligament reconstruction and tendon interposition. However, if the patient has thumb metacarpophalangeal hyperextension > 30 degrees, concurrent metacarpophalangeal joint arthrodesis is recommend to avoid adduction contracture and poor outcomes.

     

    Armbruster et al. reviewed the treatment of MCP joint hyperextension with CMC arthroplasty. They recommend the following treatment algorithm:

    0° to 10°= Surgical intervention is not necessary when MCP hyperextension is less than 10°.

    10° to 20°= Percutaneous pinning of the MCP joint in 25° to 35° of flexion for 3-4 weeks may be performed independently or as an adjunct to EPB transfer. 20° to 40°= Capsulodesis of the volar aspect of the MCP joint is recommened to provide a check rein for hyperextension and Sesamoidesis has also been investigated as an adjunctive procedure.

     

    Mouton et al. determined if a hypermobile metacarpophalangeal joint was a causative factor in the development of primary osteoarthritis at the base of the thumb. Using twenty fresh-frozen cadaveric forearm specimens they recorded the motion at the metacarpophalangeal joint in flexion neutral and hyperextension. They showed that flexion>extension produce the most palmar surface pressure of the trapeziometacarpal joint which may contribute to osteoarthritis.

     

    Figure A and B are clinical photos showing significant metacarpophalangeal joint hyperextension. Figure C is a Ap radiograph showing advanced arthritis of the CMC and MCP joints.

     

    Incorrect Answers:

    Answer 1: This is best does for stage I or II disease to unload the solar arthritis. This is typically not appropriate for stage IV disease.

    Answer 2: If the patient has thumb metacarpophalangeal hyperextension > 30 degrees, concurrent metacarpophalangeal joint arthrodesis is recommend.

    Answer 4: Carpometacarpal joint arthrodesis and metacarpophalangeal joint arthrodesis would not be recommended. CMC arthrodesis is recommended in young male heavy laborers.

    Answer 5: Carpometacarpal joint arthroscopy and debridement +/- ligament reconstruction is recommended in early stage disease.

     

     

     

  153. Figures 19a and 19b show the clinical photographs of a 2-year-old child who cannot straighten the right thumb. Examination shows active flexion of the thumb, but the interphalangeal joint cannot be passively extended. Extension is not improved with flexion of the wrist or metacarpophalangeal joint of the thumb. What is the most appropriate management?

     

     

     

     

     

     

    1. Lengthening of the flexor pollicis at the wrist

    2. Volar plate release and pinning of the interphalangeal joint in extension

    3. Release of the A-1 pulley

    4. Extension splinting of the thumb for 30 days

    5. Injection of corticosteroids at the base of the thumb

    CORRECT ANSWER: 3

    Congenital trigger thumbs are common and are reliably treated by A-1 pulley release. A palpable nodule frequently is felt on the flexor tendon, confirming the diagnosis. The interphalangeal joint does not have intrinsic contracture and extends fully after pulley release. Corticosteroid injection is difficult and not beneficial in the child with fixed flexion. Extension splinting has been used by some but requires several months of splinting, not just 30 days. The flexor pollicis is of normal length and does not require lengthening.

     

     

     

  154. A 23-year-old man sustains a dorsal dislocation of the index finger metacarpophalangeal (MCP) joint while skateboarding. Which of the following anatomic structures is most likely to be interposed between the articular surfaces and account for the irreducibility of this joint by closed methods?

    1. Volar plate

    2. Flexor tendon

    3. Extensor tendon

    4. Lumbrical tendon

    5. Radial collateral ligament

    CORRECT ANSWER:

    1

    The most common structure preventing reduction of a MCP dislocation is the volar plate. It is most easily treated by a dorsal approach and a longitudinal split of the plate. Whereas it has been described that the metacarpal head can become entrapped between the radial lumbrical and the flexor tendon, replacement of the plate volar to the metacarpal head during surgery allows for the joint to be easily reduced with no attention paid to the lumbrical or flexor tendon. The collateral ligaments and the extensor tendon have not been described as structures that prevent reduction of the MCP joint.

     

     

     

  155. A 38-year-old man who works as an auto mechanic reports right wrist pain and intermittent paresthesias and numbness into the ulnar two digits of his hand. The pain symptoms begin after a day of particularly strenuous repetitive use of the right hand. He denies any color changes to the digits but does report a significant smoking history. Weakness of the interossei muscles are seen on manual testing. Both radial and ulnar pulses are palpable at the wrist. The patient states that sensation on the dorsum of his ring and little

    fingers is normal. Radiographs are shown in Figures 25a through 25c. What is the most likely diagnosis?

     

     

     

     

     

     

     

     

     

    1. Idiopathic compression of the ulnar nerve at Guyon's canal

    2. Ulnar hammer syndrome

    3. Cubital tunnel syndrome

    4. Pseudoaneurysm of the ulnar artery in the hand

    5. Nonunion of the hamate

    CORRECT ANSWER:

    5

    The anatomy of the distal ulnar tunnel is divided into three zones. Zone 1 is proximal to the bifurcation of the ulnar nerve and consists of both sensory and motor fibers of the nerve. Zone 2 represents the motor branch of the ulnar nerve distal to the bifurcation. Zone 3 represents the sensory branches of the ulnar nerve beyond its bifurcation. The site or zone of compression will predict the clinical presentation. The most common causes of compression in zones 1 and 2 are ganglions and fractures of the hook of the hamate, which can be seen on the radiographs. The patient has significant risk factors for ulnar vascular disease as a root cause of his symptoms. The most common cause of isolated sensory nerve branch compression in zone 3 is ulnar artery thrombosis, but this does not fit with this patient because he has both motor and sensory findings. The dorsum of the fingers have normal sensation; therefore, the cubital tunnel is unlikely as a cause of his symptoms. Idiopathic compression is ruled out by the nonunion of the hamate which is clearly seen on the oblique view, just distal to the pisiform.

     

     

     

  156. A 35-year-old man sustained the injury shown in Figure 37. Three months after closed management, the patient reports the sudden inability to extend his thumb. What is the next most appropriate step in management?

     

     

     

    1. Observation

    2. Primary repair of the extensor pollicis longus

    3. Primary repair of the extensor pollicis brevis

    4. Transfer of the extensor indicis proprius to the extensor pollicis longus

    5. Transfer of the extensor indicis proprius to the extensor pollicis brevis

    CORRECT ANSWER: 4

    Extensor pollicis longus rupture after nondisplaced or minimally displaced distal radius fractures occurs due to nutritional compromise and tendon attrition. Primary repair is almost never possible, and tendon transfer using the extensor indicis proprius, or tendon grafting, is indicated.

     

     

     

  157. A 15-year-old boy sustains an injury to his index finger when a basketball impacts the tip of the finger. He reports immediate pain and swelling. A radiograph is shown in Figure 56. Treatment should consist of

     

     

     

     

    1. buddy taping to the middle finger.

    2. extension block splinting of the proximal interphalangeal joint.

    3. full-time extension splinting of the distal interphalangeal joint.

    4. closed reduction of the displaced fragment and percutaneous pinning.

    5. open reduction and internal fixation of the fracture.

     

    CorreCt answer: 3

     

    The clinical history and radiographic findings are compatible with a Mallet fracture type of injury. When the fragment is displaced but the joint remains reduced, closed extension splinting remains the treatment of choice. Surgical treatment options should be reserved only for cases where volar subluxation of the intact distal phalanx has occurred because a number of studies have shown increased complication rates with surgical treatment.

     

     

     

  158. In obtaining informed consent for a percutaneous trigger finger release of the thumb, what structure is discussed as being most vulnerable to injury?

    1. Radial digital sensory nerve to the index finger

    2. Radial digital sensory nerve to the thumb

    3. Median motor branch to the thenar muscles

    4. Volar plate of the thumb metacarpophalangeal joint

    5. Ulnar digital sensory nerve to the thumb

    CORRECT ANSWER: 2

    Percutaneous release of the A1 pulley for trigger finger release has become increasingly popular. This procedure in the index, middle, and ring fingers can be carried out with little risk to adjacent neurovascular structures. However, percutaneous release in the thumb and little finger is controversial due to the close proximity of the digital nerves to the A1 pulley being released. It is generally recommended that percutaneous trigger release in the thumb and little finger be avoided. A cadaveric study by Bain and associates showed that the average distance of the digital nerves in the thumb from the pulley release point was 2.9 mm. The radial digital nerve was within 2 mm of the release point in 5 of 10 hands studied. Thus the margin of error is very small. The site of potential crossing of the digital nerves to the little finger was variable, increasing the risk of injury as well.

     

     

     

  159. An active 28-year-old man injured his middle finger playing flag football 4 months ago. Examination reveals that there is no active distal interphalangeal joint flexion, but there is full passive motion.

    The patient has near normal active proximal interphalangeal joint motion. Examination of the hand reveals a mass in the palm at the base of the middle finger. Radiographs are normal. What treatment option will best restore normal function?

    1. Insertion of an active silicone tendon rod implant

    2. Direct repair of the flexor digitorum profundus tendon

    3. Excision of the palmar mass and distal interphalangeal joint fusion

    4. One-stage flexor tendon grafting

    5. Two-stage flexor tendon grafting

    CORRECT ANSWER: 5

    These findings are most consistent with a late presentation of a flexor digitorum profundus avulsion with retraction of the tendon to the base of the finger. Whereas early repair of a flexor digitorum profundus insertional rupture is fairly straightforward, late reconstruction is significantly more complex. If the patient is seen much past 3 weeks after the original injury, it is virtually impossible to reattach the tendon stump because of retraction of the musculotendinous unit. After 6 weeks, the pulley system has generally collapsed and scarred down and will not accept either the original tendon or a tendon graft. The best surgical option to attempt to restore near normal function involves a two-stage tendon graft. Whereas an active silicone rod provides some short-term improvement, it does not represent a permanent solution because the silicone will eventually fatigue and fail. Distal interphalangeal joint fusion represents an acceptable salvage option but will obviously not restore near normal function.

     

     

     

  160. What ligament is the primary stabilizer of the wrist following a proximal row carpectomy?

    1. Dorsal radiocarpal

    2. Dorsal intercarpal

    3. Radioscaphocapitate

    4. Ulnocapitate

    5. Ulnotriquetral CORRECT ANSWER: 3

    The radioscaphocapitate ligament is the prime stabilizer between the radius and capitate, preventing ulnar translocation of the carpus. Its oblique orientation prevents the carpus from drifting ulnarly. This stout ligament must be protected when excising the scaphoid.

     

     

  161. A patient sustained a sharp laceration to the base of his left, nondominant thumb 4 months ago. Examination reveals no active flexion but full passive motion of the interphalangeal joint. What is the best treatment option?

    1. Interphalangeal joint fusion

    2. Intercalary tendon graft

    3. Silicone rod placement

    4. Primary flexor pollicis longus repair

    5. Flexor digitorum superficialis transfer

    CORRECT ANSWER: 5

    The patient has a chronic flexor tendon laceration. There are options to restore motion and strength; therefore, fusion is not necessary. Full range of motion is present so the soft tissues are suitable for a tendon transfer. A transfer of the flexor digitorum superficialis of the ring finger to the insertion of the flexor pollicis longus on the distal phalanx provides good results with a one-stage operation.

     

     

     

  162. A 17-year-old javelin thrower reports medial-sided elbow pain and diminished grip strength while throwing. He has decreased sensation in the little and ring fingers of his throwing hand only while throwing. The sensory deficits resolve at rest. Examination of the

    elbow reveals no instability and full motion. He has a positive Tinel’s sign over the cubital tunnel and a positive elbow flexion test.

    Radiographs are normal. What is the next most appropriate step in management?

    1. Anterior ulnar nerve transposition

    2. Cortisone injection

    3. Nighttime elbow extension splinting

    4. Medial collateral ligament reconstruction

    5. Ulnar nerve decompression in situ

    CORRECT ANSWER: 3

    The patient’s symptoms and examination findings are consistent with ulnar neuritis/cubital tunnel syndrome, most probably exacerbated by javelin throwing. The first step includes rest and extension splinting. Surgical

    intervention should only be considered after failure of nonsurgical management.

     

     

     

  163. A 17-year-old high school football player reports wrist pain after being tackled. Radiographs are shown in Figures 22a through 22c. What is the recommended intervention?

     

     

     

     

     

     

     

    1. Pedicled vascularized bone graft

    2. Long arm thumb spica cast

    3. Percutaneous screw fixation

    4. Corticocancellous bone grafting via a volar approach (Matti-Russe)

    5. Open reduction and differential pitch screw placement via a dorsal approach

    CorreCt answer: 5

     

    The patient has an acute fracture of the proximal pole. A 100% healing rate has been reported for open reduction and internal fixation of proximal pole fractures via a dorsal approach. This allows for direct viewing of the fracture line, facilitates reduction, and bone grafting can be done through the same incision if necessary. A vascularized or corticocancellous graft is reserved for nonunions. Proximal fractures are very slow to heal with a cast, if they heal at all. As a small fragment, percutaneous fixation is very difficult and has been reported for waist fractures.

     

     

     

  164. Which of the following flexor tendon annular pulleys originate from palmar plates overlying joints?

    1. A1, A3, A5

    2. A2, A4

    3. A1, A2, A4

    4. A1, A2, A3

    5. A2, A4, A5

     

    CorreCt answer: 1

     

    The pulley system governs the moment arm, excursion and joint rotation produced by the flexor tendons. The A2 and A4 pulleys are the most biomechanically important to these functions. A2 and A4 arise from the periosteum of the proximal half of the proximal phalanx, and the midportion of the middle phalanx, respectively. A1, A3 and A5 are joint pulleys arising from the palmar plates of the MP, PIP, and DIP joints respectively. C1, C2, and C3 are thin, condensable, cruciate sections of the flexor sheath which permit the annular pulleys to approximate each other during flexion.

     

     

     

     

     

  165. In surgically treating hand and finger infections in patients with diabetes mellitus, what factor is associated with higher amputation rates?

    1. Insulin dependence

    2. Gram-positive organisms

    3. Renal failure

    4. Retinopathy

    5. Peripheral neuropathy CORRECT ANSWER: 3

    Patients with diabetes mellitus are prone to infection, and surgical treatment of their infections frequently requires multiple procedures. The triad of poor wound healing, chronic neuropathy, and vascular disease contributes to the increased infection rate. Studies have demonstrated increased amputation rates in patients with diabetes mellitus who have renal failure or deep polymicrobial or gram-negative infections.

     

     

     

  166. What is the most common bacteria cultured from dog and cat bites to the upper extremity?

    1. Pasteurella

    2. Streptococcus

    3. Staphylococcus

    4. Bacteroides

    5. Moraxella CORRECT ANSWER: 1

    To define bacteria responsible for dog and cat bite infections, a prospective study yielded a median of five bacterial isolates per culture. Pasteurella is most common from both dog bites (50%) and cat bites (75%). Pasteurella canis was the most frequent pathogen of dog bites, and Pasteurella multocida was the most common isolate of cat bites. Other common aerobes included streptococci, staphylococci, moraxella, and neisseria.

     

     

  167. What is the most appropriate surgical treatment for a stage III symptomatic scapholunate advanced collapsed (SLAC) wrist?

    1. Radioscapholunate arthrodesis

    2. Scaphotrapeziotrapezoid arthrodesis

    3. Scaphocapitate arthrodesis

    4. Proximal row carpectomy

    5. Scaphoid excision and capitate-lunate-triquetrum-hamate arthrodesis

    CORRECT ANSWER: 5

    SLAC is the end result of chronic scapholunate instability. The arthritis follows a predictable pattern. Stage I disease involves cartilage loss between the waist of the scaphoid and the radial styloid. In stage II, the arthritis progresses to include the proximal pole of the scaphoid and the scaphoid fossa of the radius. Finally, stage III goes on to include arthritis of the capitolunate joint. The only treatment option that addresses all of the sites of arthritis is the scaphoid excision and four corner fusion.

     

     

     

  168. Which of the following is considered an important component in treating the lesion shown in Figure 56?

 

 

 

 

  1. Excision of the skin in addition to the cyst

  2. Resection of the nail plate

  3. Excision of bony osteophytes from the distal interphalangeal (DIP) joint

  4. Injection of corticosteroid into the DIP joint

  5. Resection of part of the collateral ligament and extensor mechanism

CORRECT ANSWER: 3

Mucoid cysts are commonly associated with DIP joint arthritis. Two treatment options are commonly used: (1) aspiration/drainage and injection of corticosteroid and (2) surgical excision. When performing the surgery, excision of the bony osteophytes about the DIP joint is helpful in achieving a cure.

There are no reports of significant benefit with nail removal or partial

ligament or extensor tendon resection. Some authors have advocated skin excision and rotational flaps for wound coverage, but this is somewhat controversial.